Anda di halaman 1dari 358

Civil Law Q&As (2007-2013)

hectorchristopher@yahoo.com

dbaratbateladot@gmail.com

A Compilation of the
Questions and Suggested Answers
In the
PHILIPPINE BAR EXAMINATIONS 2007-2013
In

CIVIL LAW
Compiled and Arranged By:
Baratbate-Ladot, Delight
Salise, Hector Christopher Jay-Arh Jr. M.
(University of San Jose-Recoletos School of Law)

ANSWERS TO BAR EXAMINATION QUESTIONS by


the UP LAW COMPLEX (2007-2013)
&
PHILIPPINE ASSOCIATION OF LAW SCHOOLS (2008)

Never Let The Odds Keep You From Pursuing What You Know In Your Heart You Were Meant To Do.Leroy Satchel Paige

Page 1 of 180

Civil Law Q&As (2007-2013)

hectorchristopher@yahoo.com

dbaratbateladot@gmail.com

FOREWORD
This work is a compilation of the ANSWERS TO BAR
EXAMINATION QUESTIONS by the UP LAW COMPLEX ,
Philippine Association of Law Schools from 2007-2010 and
local law students and lawyers forum sites from 2011-2013
and not an original creation or formulation of the author.
The authors were inspired by the work of Silliman Universitys
College of Law and its students of producing a very good
material to everyone involved in the legal field particularly the
students and the reviewees for free. Hence, this work is a
freeware.
Everyone is free to distribute and mass produce copies of this
work, however, the author accepts no liability for the content of
this reviewer, or for the consequences of the usage, abuse, or
any actions taken by the user on the basis of the information
given.
The answers (views or opinions) presented in this reviewer are
solely those of the authors in the given references and do not
necessarily represent those of the authors of this work.
The Authors.

Never Let The Odds Keep You From Pursuing What You Know In Your Heart You Were Meant To Do.Leroy Satchel Paige

Page 2 of 180

Civil Law Q&As (2007-2013)

hectorchristopher@yahoo.com

dbaratbateladot@gmail.com

TABLE OF CONTENTS
(Titles are based on Sillimans Compilation [Arranged by Topic])

Persons
Capacity: Juridical Capacity (2008).......................................................................................................... 12
Capacity; Juridical Capacity of Donee; Requisites for Acceptance (2012)...............................12
Capacity: Legal Capacity; Lex Rei Sitae (2007).....................................................................................13
Correction of Entries; Clerical Error Act (2008)..................................................................................14
Nationality Principle (2009)...........................................................................................................................14
Nationality Principle; Change of Name not Covered (2009).............................................................15

Conflict of Laws
Processual Presumption (2009).................................................................................................................... 16
Jurisdiction; Courts may Assume Jurisdiction over Conflict of Laws Cases (2010)...........17

Adoption
Adoption; Termination; Death of Adopter (2009).................................................................................17
Adoption; Illegitimate Child (2010)............................................................................................................18
Adoption; Illegitimate Child; Use of Mothers Surname as Middle Name (2012)..................19
Consent; Consent of the Adopters Heirs (2008)..................................................................................19
Qualifications of Adopter (2010).................................................................................................................20

Never Let The Odds Keep You From Pursuing What You Know In Your Heart You Were Meant To Do.Leroy Satchel Paige

Page 3 of 180

Civil Law Q&As (2007-2013)

hectorchristopher@yahoo.com

dbaratbateladot@gmail.com

Family Code
Marriage; Annulment; Grounds (2009).....................................................................................................20
Marriage; Annulment; Grounds (2007).....................................................................................................21
Marriage; Annulment; Parties (2012).........................................................................................................22
Marriage; Annulment; Support Pendente Lite (2010).........................................................................22
Marriage; Divorce Decrees; Filipino Spouse Becoming Alien (2009)..........................................23
Marriage; Divorce Decrees; Foreign Spouse Divorces Filipino Spouse (2012)........................24
Marriage; Divorce Decrees; Foreign Spouse Divorces Filipino Spouse (2010)........................25
Marriage; Legal Separation; Prescription (2012)..................................................................................25
Marriage; Legal Separation; Prescription (2007)..................................................................................26
Marriage; Psychological Incapacity (2013).............................................................................................26
Marriage; Psychological Incapacity (2012).............................................................................................28
Marriage; Requisites (2008)...........................................................................................................................28
Marriage; Subsequent Marriage (2008).....................................................................................................29
Marriage; Void Marriages; By Reason of Public Policy (2008)........................................................30
Marriage; Void Marriages; By Reason of Public Policy (2007)........................................................30
Marriage; Void Marriages; Property Relations (2009).........................................................................30
Marriage; Void Marriages; Status of Children (2009)..........................................................................31
Parental Authority; Illegitimate Minor Child (2009)..........................................................................32
Parental Authority; In Vitro Fertilization (2010).................................................................................32

Never Let The Odds Keep You From Pursuing What You Know In Your Heart You Were Meant To Do.Leroy Satchel Paige

Page 4 of 180

Civil Law Q&As (2007-2013)

hectorchristopher@yahoo.com

dbaratbateladot@gmail.com

Paternity & Filiation; Child Born Under a Void Marriage (2010)................................................33


Paternity & Filiation; Impugning Legitimacy (2010)..........................................................................34
Paternity & Filiation; In Vitro Fertilization;
Surrogate Mothers Remedy to Regain Custody (2010).........................34
Paternity & Filiation; Legitimacy; Presumption (2008)................................................................... 35
Paternity & Filiation; Legitimation of a Child from a Previous Valid Marriage (2008)......36
Paternity & Filiation; Legitimation of a Dead Child (2009)............................................................37
Paternity & Filiation; Support: Ascendants & Descendants;
Collateral Blood Relatives (2008).................................................................... 37
Paternity & Filiation; Use of Surname; Illegitimate Child (2009)................................................38
Paternity & Filiation; Who May Impugn Legitimacy (2009)...........................................................39
Property Relations; Adulterous Relationship (2009)..........................................................................39
Property Relations; Accession (2012)........................................................................................................40
Property Relations; Ante-Nuptial Debt (2007)...................................................................................... 40
Property Relations; Unions Without Marriage (2012)........................................................................41
Property Relations; Unions Without Marriage (2012)........................................................................42
Property Relations; Void Marriages (2010).............................................................................................43
Property Relations; Void Marriages (2010).............................................................................................45

Never Let The Odds Keep You From Pursuing What You Know In Your Heart You Were Meant To Do.Leroy Satchel Paige

Page 5 of 180

Civil Law Q&As (2007-2013)

hectorchristopher@yahoo.com

dbaratbateladot@gmail.com

Succession
Disposition; Mortis Causa vs. Intervivos; Corpse (2009)................................................................. 46
Heirs; Fideicommissary Substitution (2008).........................................................................................46
Heirs; Intestate Succession; Legitime; Computation (2010)..........................................................47
Heirs; Representation; Iron-Curtain Rule (2012).................................................................................49
Heirs; Reserva Troncal (2009)......................................................................................................................49
Intestate Succession (2008)...........................................................................................................................50
Intestate Succession (2008)...........................................................................................................................51
Intestate Succession; Rights of Representation:
Illegitimate, Adopted Child; Iron Curtain Rule (2007).......................... 51
Legitimes; Compulsory Heirs (2012).........................................................................................................53
Legitime; Compulsory Heirs (2008)............................................................................................................53
Preterition; Disinheritance (2008).............................................................................................................54
Succession; Proof of Death between persons called to succeed each other (2008).............55
Succession; Rule on Survivorship (2009)...............................................................................................56
Wills; Holographic Wills; Insertions & Cancellations (2012)...........................................................57
Wills; Holographic Wills; Probate (2009).................................................................................................. 57
Wills; Joint Wills (2008)...................................................................................................................................59
Wills; Joint Wills; Probate (2012)................................................................................................................59

Never Let The Odds Keep You From Pursuing What You Know In Your Heart You Were Meant To Do.Leroy Satchel Paige

Page 6 of 180

Civil Law Q&As (2007-2013)

hectorchristopher@yahoo.com

dbaratbateladot@gmail.com

Wills; Prohibition to Partition of a Co-Owned Property (2010).....................................................60


Wills; Notarial Wills; Blind Testator; Requisites (2008).....................................................................61
Wills; Testamentary Disposition; Period to Prohibit Partition (2008)........................................61
Wills; Witnesses to a Will, Presence required; Thumbmark as Signature (2007)..................62

Donation
Donations; Formalities; In Writing (2007).............................................................................................. 63
Donations; Illegal & Impossible Conditions (2007).............................................................................64
Donation; Inter Vivos (2013)..........................................................................................................................64

Property
Accretion; Alluvium (2008)............................................................................................................................65
Accretion; Rights of the Riparian Owner (2009)..................................................................................67
Builder; Good Faith; Requisites (2013).....................................................................................................68
Easement; Prescription; Acquisitive Prescription (2009)................................................................70
Easement; Right of Way (2013)....................................................................................................................70
Easement; Right of Way (2010)....................................................................................................................72
Hidden Treasure (2008)...................................................................................................................................73
Mortgage; Public or Private Instrument (2013).....................................................................................74
Occupation vs. Possession (2007)...............................................................................................................76
Ownership; Co-Ownership (2009)................................................................................................................76
Ownership; Co-Ownership (2008)................................................................................................................77

Never Let The Odds Keep You From Pursuing What You Know In Your Heart You Were Meant To Do.Leroy Satchel Paige

Page 7 of 180

Civil Law Q&As (2007-2013)

hectorchristopher@yahoo.com

dbaratbateladot@gmail.com

Property; Movable or Immovable (2007)...................................................................................................78

Land Titles and Deeds


Acquisition of Lands; Sale of Real Property to an Alien (2009)....................................................79
Non-Registrable Properties (2007)...............................................................................................................80
Prescription; Acquisitive Prescription (2008)....................................................................................... 81
Prescription; Judicially Foreclosed Real Property Mortgage (2012)...........................................82
Purchaser in Good Faith; Mortgaged Property (2008)....................................................................... 83
Registration; Governing Law (2007)...........................................................................................................84
Registration; Party Who First took Possession (2013)......................................................................85
Registration; Requisites; Proof (2013)...................................................................................................... 86
Remedies; Fraud; Rights of Innocent Purchaser (2009)...................................................................89

Contracts
Contract to Sell vs. Conditional Contract of Sale (2012)................................................................90
Rescission of Contract; Fortuitous Event (2008).................................................................................90
Stipulation; Arbitration Clause (2009).......................................................................................................91

Obligations
Extinguishment; Compensation (2009)....................................................................................................91
Extinguishment; Compensation (2008)....................................................................................................92
Extinguishment; Novation (2008)...............................................................................................................93
Extinguishment; Payment of Check (2013)............................................................................................94

Never Let The Odds Keep You From Pursuing What You Know In Your Heart You Were Meant To Do.Leroy Satchel Paige

Page 8 of 180

Civil Law Q&As (2007-2013)

hectorchristopher@yahoo.com

dbaratbateladot@gmail.com

Extinguishment; Payment of Check; Legal Tender (2008)..............................................................95


Liability; Solidary Liability (2008).............................................................................................................. 96
Obligations; Without Agreement (2007)..................................................................................................97

Trust
Trust De Son Tort (2007)................................................................................................................................98

Sales
Condominium Act; Partition of a Condominium (2009)...................................................................99
Mortgage; Equitable Mortgage (2012)........................................................................................................99
Option Contract; Liquor & Pulutan as Consideration (2013)...................................................100
Right of First Refusal; Lessee; Effect (2008).......................................................................................101

Lease
Builder; Good Faith; Useful Improvements (2013)...........................................................................103
Lease; Caveat Emptor (2009)......................................................................................................................104

Agency
Agency; Sale of a Real Property through an Agent (2010)............................................................104

Partnership
Liability; Liability of a Partner (2010)....................................................................................................105
Oral Partnership (2009)................................................................................................................................106
Share; Demand during the Existence of Partnership (2012)........................................................107

Never Let The Odds Keep You From Pursuing What You Know In Your Heart You Were Meant To Do.Leroy Satchel Paige

Page 9 of 180

Civil Law Q&As (2007-2013)

hectorchristopher@yahoo.com

dbaratbateladot@gmail.com

Commodatum & Mutuum


Mutuum; Interest; Solutio Indebiti (2012)............................................................................................107

Guaranty
Guaranty (2009).................................................................................................................................................108

Surety
Surety (2010)......................................................................................................................................................108

Pledge
Pledge; Pactum Commissorium (2009)..................................................................................................109

Torts and Damages


Damages (2012).................................................................................................................................................109
Damages; Moral & Exemplary (2009)......................................................................................................110
Damages; Public Officers acting in the Performance of their Duties (2012).......................111
Death Indemnity (2009)................................................................................................................................ 112
Doctrine of Discovered Peril (Last Clear Chance) (2007)...............................................................112
Liability; Owner of a Pet; Fortuitous Event (2010)..........................................................................113
Liability; Special Parental Authority (2010).......................................................................................115
Quasi-Delict; Claims; Requisites (2013).................................................................................................115
Quasi Tort (2010)..............................................................................................................................................116

Never Let The Odds Keep You From Pursuing What You Know In Your Heart You Were Meant To Do.Leroy Satchel Paige

Page 10 of 180

Civil Law Q&As (2007-2013)

hectorchristopher@yahoo.com

dbaratbateladot@gmail.com

MULTIPLE CHOICE QUESTIONS


2013 Civil Law Exam MCQ (October 13, 2013)..............118
2012 Civil Law Exam MCQ (October 14, 2012).................130
2011 Civil Law Exam MCQ (November 13, 2011)..............149
2010 Civil Law Exam MCQ (September 12, 2010).............176
2007 Civil Law Exam MCQ (September 09, 2007).............179

Never Let The Odds Keep You From Pursuing What You Know In Your Heart You Were Meant To Do.Leroy Satchel Paige

Page 11 of 180

Civil Law Q&As (2007-2013)

hectorchristopher@yahoo.com

dbaratbateladot@gmail.com

that "for civil purposes, the fetus shall


be considered born if it is alive at the

Persons

time it is completely delivered from the

Capacity: Juridical Capacity (2008)


No. II. At age 18, Marian found out that she
was pregnant. She insured her own life and
named her unborn

child

as her sole

beneficiary. When she was already due to


give birth, she and her boyfriend Pietro, the
father

of

her

unboarn

child,

were

kidnapped in a resort in Bataan where they


were vacationing. The military gave chase
and after one week, they were found in an
abandoned hut in Cavite. Marian and Pietro
were hacked with bolos. Marian and the
baby delivered were both found dead, with
the baby's umbilical cord already cut. Pietro
survived.
(A). Can Marian's baby be the beneficiary of
the insurance taken on the life of the
mother? (2%)
SUGGESTED ANSWER:
Yes, the baby can be the beneficiary of
the life insurance of Marian. Art. 40 NCC
provides

that

personality;

but

"birth
the

determines

conceived

child

shall be considered born for all purposes


that are favorable to it, provided that it
be

born

later

with

the

conditions

specified in Art. 41. Article 41 states

mother's womb. However, if the fetus


had an intra-uterine life of less than
seven months, it is not deemed born if it
dies within twenty-four (24) hours after
its complete delivery from the maternal

Capacity; Juridical Capacity of Donee;


Requisites for Acceptance (2012)
No.I. b) Ricky donated P 1 Million to the
unborn child of his pregnant girlfriend,

womb. The act of naming the unborn

which she accepted. After six (6) months of

child as sole beneficiary in the insurance

pregnancy, the fetus was born and baptized

is favorable to the conceived child and

as Angela. However, Angela died 20 hours

therefore the fetus acquires presumptive

after birth. Ricky sought to recover the P 1

or provisional personality. However, said

Million.

presumptive personality only becomes

Explain. (5%)

conclusive if the child is born alive. The


child need not survive for twenty-four

Is

Ricky

entitled

to

recover?

SUGGESTED ANSWER:

(24) hours as required under Art. 41 of

Yes, Ricky is entitled to recover the

the Code because "Marian was already

P1,000,000.00. The NCC considers a

due to give birth," indicating that the


child was more than seven months old.
Never Let The Odds Keep You From Pursuing What You Know In Your Heart You Were Meant To Do.Leroy Satchel Paige

Page 12 of 180

Civil Law Q&As (2007-2013)

hectorchristopher@yahoo.com

dbaratbateladot@gmail.com

the statement is FALSE, state the reason.


fetus a person for purposes favorable to

(2% each).

it provided it is born later in accordance

(1). Roberta, a Filipino, 17 years of age,

with the provision of the NCC. While the

without the knowledge of his parents, can

donation is favorable to the fetus, the


donation did not take effect because the
fetus was not born in accordance with
the NCC.
To be considered born, the fetus that
had an intrauterine life of less than
seven (7) months should live for 24
hours from its complete delivery from
the mothers womb. Since Angela had an
intrauterine life of less than seven (7)
months but did not live for 24 hours, she
was not considered born and, therefore,
did not become a person. Not being a
person, she has no juridical capacity to
be a donee, hence, the donation to her
did not take effect. The donation not
being effective, the amount donated may
be recovered. To retain it will be unjust
enrichment.

Capacity: Legal Capacity; Lex Rei Sitae


(2007)
No.VII. Write "TRUE" if the statement is
true or "FALSE" if the statement is false. If

contract in Australia is governed by


because

Philippine

Australian Laws allow aliens to acquire

ownership

property from the age of 16.

until the contract is annulled.

SUGGESTED ANSWER:

ALTERNATIVE ANSWER:

TRUE. Since Australian Law allows alien

FALSE. Laws relating to family rights

to acquire property from the age of 16,

and duties, or to the status, condition or

Roberta may validly own a house in

legal capacity of persons are binding

Australia, following the principle of lex

upon the citizens of the Philippines,

rei sitae enshrined in Art. 16, NCC,

even though living abroad (Art. 15, NCC).

which states "Real property as well as

The age of majority under Philippine law

personal property is subject to the law of

is 18 years (R.A. No. 6809); hence,

the

Roberta, being only 17 years old, has no

acquire

house

country

Moreover,

in

where

even

Australia

it

is

assuming

situated."
that

legal

Law,
over

she
the

will

property

acquire
bought

legal capacity to acquire and own land.

capacity of Roberta in entering the

Never Let The Odds Keep You From Pursuing What You Know In Your Heart You Were Meant To Do.Leroy Satchel Paige

Page 13 of 180

Civil Law Q&As (2007-2013)

hectorchristopher@yahoo.com

that
Correction of Entries; Clerical Error Act
(2008)
No. IV. Gianna was born to Andy and
Aimee, who at the time Gianna's birth were
not married to each other. While Andy was
single at the time, Aimee was still in the
process of securing a judicial declaration of
nullity on her marriage to her ex-husband.
Gianna's birth certificate, which was signed
by both Andy and Aimee, registered the
status

of

Gianna

as

"legitimate",

her

surname carrying that of Andy's and that


her parents were married to each other.
(A). Can a judicial action for correction of
entries in Gianna's birth certificate be
successfully maintained to:
a). Change her status from "legitimate" to
"illegitimate" (1%);
and
b). Change her surname from that of Andy's
to Aimee's maiden surname? (1%)
SUGGESTED ANSWER:
Yes, a judicial action for correction of
entries in Gianna's birth certificate can
be successfully maintained to change (a)
her

status

from

"legitimate"

to

"illegitimate," and (b) her surname from

of

Andy's

dbaratbateladot@gmail.com

to

Aimee's

maiden

surname in accordance with Rule 108 of

nickname in the civil register without


the Rules of Court because said changes

need of a judicial order. Errors that

are substantive corrections.

involve the change of nationality, age,


status, surname or sex of petitioner are

(B).

can

not included from the coverage of the

administrative proceedings be brought for

said Act (Silverio v. Republic, G.R. No.

the

Instead

purpose

of

of

judicial

making

action,

the

above

corrections? (2%)
SUGGESTED ANSWER:
No. An administrative proceeding cannot
be brought for the purpose of making the
above corrections. R.A. 9048, otherwise

174689, 22 Oct., 2007).

Nationality Principle (2009)


No.XII. Emmanuel and Margarita, American
citizens and employees of the U.S. State
Department, got married in the African

known as the Clerical Error Act, which

state of Kenya where sterility is a ground for

authorizes the city or municipal civil

annulment of marriage. Thereafter, the

registrar or the consul general to correct

spouses were assigned to the U.S. Embassy

a clerical or typographical error in an

in Manila. On the first year of the spouses

entry and/or change the first name or


Never Let The Odds Keep You From Pursuing What You Know In Your Heart You Were Meant To Do.Leroy Satchel Paige

Page 14 of 180

Civil Law Q&As (2007-2013)

hectorchristopher@yahoo.com

dbaratbateladot@gmail.com

Recio, 366 SCRA 437 [2001], Llorente v.


tour of duty in the Philippines, Margarita

Court of Appeals 345 SCRA 92 [2000],

filed an annulment case against Emmanuel

and Bayot v. Court of Appeals 570 SCRA

before a Philippine court on the ground of

472 [2008]).

her husbands sterility at the time of the


celebration of the marriage.
(A). Will the suit prosper? Explain your
answer. (3%)
SUGGESTED ANSWER:
No, the suits will not prosper. As applied
to foreign nationals with the respect to
family relations and status of persons,
the nationality principle set forth in
Article 15 of the Civil Code will govern
the

relations

Margarita.

of

Since

Emmanuel

they

are

and

American

citizens, the governing law as to the


ground for annulment is not Kenyan Law
which Magarita invokes in support of
sterility as such ground; but should be
U.S. Law, which is the national Law of
both

Emmanuel

and

Margarita

as

recognized under Philippine Law. Hence,


the Philippine court will not give due
course to the case based on Kenyan Law.
The nationality principle as expressed in
the application of national law of foreign
nationals

by

Philippine

courts

is

established by precedents (Pilapil v. IbaySomera, 174 SCRA 653[1989], Garcia v.

No.XX. (A). If Ligaya, a Filipino citizen


residing in the United States, files a petition

ALTERNATIVE ANSWER:
The

forum

has

jurisdiction

over

an

action for the annulment of marriage


solemnized elsewhere but only when the

for change of name before the District Court


of New York, what law shall apply? Explain.
(2%)

party bringing the actions is domiciled

SUGGESTED ANSWER:

in the forum. In this case, none of the

New York law shall apply. The petition of

parties to the marriage is domiciled in

change of name file din New York does

the

as

not concern the legal capacity or status

officials of the US Embassy whose stay in

of the petitioner. Moreover, it does nto

the country is merely temporary, lasting

affect the registry of any other country

only during their fixed tour of duty.

including the country of birth of the

Hence, the Philippine courts have no

petitioner.

jurisdiction over the action.

rendered in that petition will have effect

Philippines.

They

are

here

Whatever

judgment

is

only in New York. The New York court


Nationality Principle; Change of Name
not Covered (2009)

Never Let The Odds Keep You From Pursuing What You Know In Your Heart You Were Meant To Do.Leroy Satchel Paige

Page 15 of 180

Civil Law Q&As (2007-2013)

hectorchristopher@yahoo.com

dbaratbateladot@gmail.com

SUGGESTED ANSWER:
cannot, for instance, order the Civil
Registrar in the Philippines to change its
records. The judgment of the New York
court allowing a change in the name of
the petitioner will be limited to the
records of the petitioner in New York and
the

use

of

her

new

name

in

all

transactions in New York. Since the


records and processes in New York are
the only ones affected, the New York
court

will

apply

New

YorK

law

in

resolving the petition.


ALTERNATIVE ANSWER:
Philippine law shall apply (Art 15, NCC).
Status, conditions, family rights and
duties are governed by Philippine laws as
to

Filipinos

even

though

sojourning

abroad.
ALTENATIVE ANSWER:
If Ligaya, a Filipino, files a petition for
change of name with the District Court
of New YoRk, the laws of New York will
govern since change of name is not one
of those covered by the principles of
nationality.
(B). If Henry, an American citizen residing
in the Philippines, files a petition for change
of name before a Philippine court, what law
shall apply? Explain. (2%)

Philippine law will apply. The petition for


change of name in the Philippines will
affect only the records of the

principle, by which his legal status is


petitioner and his transactions in the

governed by national law, the matter of

Philippines. The Philippine court can

change of name being included in the

never

the

legal status. The Supreme Court has

custodian in the US of the records of the

reiterate in several cases, that the lex

petitioner. Moreover, change of name has

patriae as provided in Article 15 of the

nothing to do with the legal capacity or

Civil

status of the alien. Since Philippine

nationals

records and transactions are the only

status (supra).

acquire

jurisdiction

over

Code

ones affected, the Philippine court may

and transactions that law cannot be but

in

applicable

determining

to

foreign

their

legal

Conflict of Laws

effect the change only in accordance


with the laws governing those records

is

Processual Presumption (2009)

Philippine law.

No.I. TRUE or FALSE. Answer TRUE if the

ALTERNATIVE ANSWER:

statement

U.S. law shall apply as it is his national

statement is false. Explain your answer in

law. This is pursuant to the application

not more than two (2) sentences.

of

lex

patriae

or

the

is

true,

or

FALSE

if

the

nationality

Never Let The Odds Keep You From Pursuing What You Know In Your Heart You Were Meant To Do.Leroy Satchel Paige

Page 16 of 180

Civil Law Q&As (2007-2013)

hectorchristopher@yahoo.com

dbaratbateladot@gmail.com

ALTERNATIVE ANSWER:
(A).

The

doctrine

of

"processual

presumption" allows the court of the forum


to presume that the foreign law applicable
to the case is the same as the local or
domestic law. (1%)
SUGGESTED ANSWER:
TRUE. If the foreign law necessary to the
resolve an issue is not proven as a fact,
the court of the forum may presume that
the foreign law is the same as the law of
the forum.

Jurisdiction;

Courts

may

Assume

Jurisdiction over Conflict of Laws Cases


(2010)
No.III. Define, Enumerate or Explain. (2%
each)
(C) Give at least two reasons why a court
may assume jurisdiction over a conflict of
laws case.
SUGGESTED ANSWER:
(1) Statute theory. There is a domestic
law

authorizing

the

local

court

to

assume jurisdiction.
(2)

Comity

assumes

theory.

jurisdiction

The

local

based

principle of comity or courtesy.

on

court
the

(1) Public Order. To maintain peace and

No.XIII. Rafael, a wealthy bachelor, filed a


petition for the adoption of Dolly, a one-year

order, disputes that disturb the peace of

old foundling who had a severe heart ailment.

the forum should be settled by the court

During

of the forum even though the application

proceedings, Rafael died of natural causes.

of the foreign law is necessary for the

The Office of the Solicitor General files a

purpose.

motion to dismiss the petition on the ground

the

pendency

of

the

adoption

that the case can no longer proceed because

(2) Humanitarian Principle. An aggrieved

of the petitioners death.

party should not be left without remedy


in a forum even though the application
of the foreign law by the courts of the
forum is unavoidable in order to extend
relief.

(A). Should the case be dismissed? Explain.


(2%)
SUGGESTED ANSWER:
It

Adoption

depends

on

the

stage

of

the

proceedings when Rafael died. If he died


after all the requirements under the law

Adoption; Termination; Death of Adopter


(2009)
Never Let The Odds Keep You From Pursuing What You Know In Your Heart You Were Meant To Do.Leroy Satchel Paige

Page 17 of 180

Civil Law Q&As (2007-2013)

hectorchristopher@yahoo.com

dbaratbateladot@gmail.com

decree should be allowed in case it is the


have been complied with and the case is

adoptee who dies because adoption is

already submitted for resolution, the

primarily for his benefit.

court may grant the petition and issue a


decree of adoption despite the death of
the

adopter

(Section

13,

RA

8552).

Otherwise, the death of the petitioner


shall have the effect terminating the
proceedings.
(B). Will your answer be the same if it was
Dolly who died during the pendency of the
adoption proceedings? Explain. (2%)
SUGGESTED ANSWER:
No, if it was Dolly who died, the case
should

be

dismissed.

Her

death

terminates the proceedings (Art. 13,


Domestic Adoption Law).

ALTERNATIVE ANSWER:
It depends. If all the requirements under
the law have already been complied with
and the case is already submitted for
resolution, the death of the adoptee
should not abate the proceedings. The
court

should

issue

the

decree

of

adoption if will be for the best interest of


the adoptee. While RA8552 provides only
for the case where it is the petitioner
who dies before the decree is issued, it is
with more compelling reason that the

SUGGESTED ANSWER:
Adoption; Illegitimate Child (2010)
No.VIII. Spouses Rex and Lea bore two
children now aged 14 and 8. During the

The

consent

of

the

14-year-old

legitimate child, of the 10- year -old


illegitimate child and of the biological

subsistence of their marriage, Rex begot a

mother of the illegitimate child are

child by another woman. He is now 10

needed for the adoption (Section 7 and

years of age.

9, RA 8552). The consent of Lea is no

On Leas discovery of Rexs fathering a child


by another woman, she filed a petition for
legal separation which was granted.
Rex now wants to adopt his illegitimate
child.
(A) Whose consent is needed for Rexs
adoption of his illegitimate child? (2.5%)

longer

required

because

there

was

already a final decree of legal separation.


(B) If there was no legal separation, can Rex
still adopt his illegitimate child? Explain.
(2.5%)
SUGGESTED ANSWER:
Yes, he can still adopt his illegitimate
child but with the consent of his spouse,

Never Let The Odds Keep You From Pursuing What You Know In Your Heart You Were Meant To Do.Leroy Satchel Paige

Page 18 of 180

Civil Law Q&As (2007-2013)

hectorchristopher@yahoo.com

dbaratbateladot@gmail.com

law is silent as to what middle name an

of his 14-year-old legitimate child, of the


illegitimate child, and of the biological
mother of the illegitimate child (Section
7 and 9, RA 8552).

Adoption;
Mothers

Illegitimate
Surname

as

Child;

Use

Middle

of

Name

(2012)
No.IV.b) Honorato filed a petition to adopt
his

minor

illegitimate

child

Stephanie,

alleging that Stephanies mother is Gemma


Astorga Garcia; that Stephanie has been
using

her

mothers

middle

name

and

surname; and that he is now a widower and


qualified to be her adopting parent. He
prayed that Stephanies middle name be
changed from "Astorga" to "Garcia," which is
her

mothers

surname

and

that

her

surname "Garcia" be changed to "Catindig,"


which is his surname. This the trial court
denied. Was the trial court correct in
denying Hororatos request for Stephanies
use of her mothers surname as her middle
name? Explain. (5%)
SUGGESTED ANSWER:
No, the trial court was not correct. There
is no law prohibiting an illegitimate child
adopted by his natural father to use as
middle name his mothers surname. The

adoptee may use. In case of In re:

Adoption of Stephanie Nathy Astorga


Garcia, G.R. No. 148311, March 31,

Consent; Consent of the Adopters Heirs


(2008)

2005, the Supreme Court ruled that the

No.V. Despite several relationships with

adopted child may use the surname of

different

the natural mother as his middle name

unmarried.

because there is no prohibition in the

Brenda produced a daughter, Amy, now 30

law against it. Moreover, it will also be

years old. His second, with Carla, produced

for the benefit of the adopted child who


shall

preserve

his

lineage

on

his

mothers side and reinforce his right to


inherit from his mother and her family.
Lastly, it will make the adopted child

women,
His

Andrew

first

remained

relationship

with

two sons: Jon and Ryan. His third, with


Donna, bore him no children although
Elena has a daughter Jane, from a previous
relationship. His last, with Fe, produced no
biological

children

but

they

informally

adopted without court proceedings, Sandy's

conform with the time-honored Filipino

now 13 years old, whom they consider as

tradition

their own. Sandy was orphaned as a baby

of

carrying

the

mothers

surname as the persons middle name.

and was entrusted to them by the midwife


who attended to Sandy's birth. All the

Never Let The Odds Keep You From Pursuing What You Know In Your Heart You Were Meant To Do.Leroy Satchel Paige

Page 19 of 180

Civil Law Q&As (2007-2013)

hectorchristopher@yahoo.com

dbaratbateladot@gmail.com

including Amy and Elena obtains the


children, including Amy, now live with

written consent of Jane, if she is over

andrew in his house.

ten years old (Sec. 9(d), RA 8552).

(A). Is there any legal obstacle to the legal


adoption of Amy by Andrew? To the legal
adoption of Sandy by Andrew and Elena?
(2%)
SUGGESTED ANSWER:
Yes, there is a legal obstacle to the legal
adoption of Amy by Andrew. Under Sec.
9(d) of RA 8552, the New Domestic
Adoption

Act

consent

of

of

1998,
the

the

written

illegitimate

sons/daughters, ten (10) years of age or


over, of the adopter, if living with said
adopter and the latter's spouse, if any, is
necessary

to

the

adoption.

All

the

children of Andrew are living with him.


Andrew needs to get the written consent
of Jon, Ryan, Vina and Wilma, who are
all ten (10) years old or more. Sandy's
consent

to

Amy's

adoption

is

not

necessary because she was not legally


adopted by Andrew. Jane's consent is
likewise not necessary because she is
not a child of Andrew. Sandy, an orphan
since birth, is eligible for adoption under
Sec. 8(f) of RA 8552, provided that
Andrew obtains the written consent of
the other children mentioned above,

on certain situations enumerated in the


law. The case of John does not fall in any

Qualifications of Adopter (2010)

of the exceptions (R.A. 8552).


No.IX. Eighteen-year old Filipina Patrice
had a daughter out of wedlock whom she
named Laurie. At 26, Patrice married
American citizen John who brought her to
live with him in the United States of
America.

John

at

once

signified

his

willingness to adopt Laurie.

Marriage; Annulment; Grounds (2009)


No.XII. Emmanuel and Margarita, American
citizens and employees of the U.S. State
Department, got married in the African

Can John file the petition for adoption? If


yes, what are the requirements? If no, why?
(5%)

state of Kenya where sterility is a ground for


annulment of marriage. Thereafter, the
spouses were assigned to the U.S. Embassy
in Manila. On the first year of the spouses
tour of duty in the Philippines, Margarita

SUGGESTED ANSWER:
No, John cannot file the petition to
adopt

Family Code

alone.

Philippine

law

filed an annulment case against Emmanuel


before a Philippine court on the ground of

requires

husband and wife to adopt jointly except


Never Let The Odds Keep You From Pursuing What You Know In Your Heart You Were Meant To Do.Leroy Satchel Paige

Page 20 of 180

Civil Law Q&As (2007-2013)

hectorchristopher@yahoo.com

dbaratbateladot@gmail.com

the formal validity of such marriage is


her husbands sterility at the time of the

governed by Kenyan law and any issue as

celebration of the marriage.

to the formal validity of that marriage

(B). Assume Emmanuel and Margarita are


both

Filipinos.

Kenya,

they

After

come

their

back

wedding

and

take

in
up

residence in the Philippines. Can their


marriage be annulled on the ground of
Emmanuels sterility? Explain. (3%)
SUGGESTED ANSWER:
No, the marriage cannot be annulled
under the Philippine law. Sterility is not
a ground for annulment of marriage
under Article 45 of the Family Code.

ALTERNATIVE ANSWER:
No, the marriage cannot be annulled in
the Philippines.
The

Philippine

court

shall

have

jurisdiction over the action to annul the


marriage not only because the parties
are residents of the Philippines but
because they are Filipino citizens. The
Philippine court, however, shall apply
the law of the place where the marriage
was celebrated in determining its formal
validity (Article 26, FC; Article 17, NCC).

Since the marriage was celebrated in


Kenya in accordance with Kenyan law,

Since

sterility

does

not

constitute

shall be determined by applying Kenyan

absence or defect in the legal capacity of

law and not Philippine law.

the parties under Philippine law, there is


no

ground

to

avoid

or

annul

the

However, while Kenyan law governs the

marriage. Hence, the Philippine court

formal validity of the marriage, the legal

has to deny the petition.

capacity of the Filipino parties to the


marriage is governed not by Kenyan law
but by Philippine law (Article 15, NCC).
Sterility of a party as a ground for the

Marriage; Annulment; Grounds (2007)

annulment of the marriage is not a

No. VII. Write "TRUE" if the statement is

matter of form but a matter of legal

true or "FALSE" if the statement is false. If

capacity. Hence, the Philippine court

the statement is FALSE, state the reason.

must

(2% each).

apply

Phillippine

law

in

determining the status of the marriage


on the ground of absence or defect in the
legal capacity of the Filipino parties.

(4). The day after John and Marsha got


married,

John

told

her

that

he

was

impotent. Marsha continued to live with

Never Let The Odds Keep You From Pursuing What You Know In Your Heart You Were Meant To Do.Leroy Satchel Paige

Page 21 of 180

Civil Law Q&As (2007-2013)

hectorchristopher@yahoo.com

dbaratbateladot@gmail.com

declaration of nullity may only be filed


John for 2 years. Marsha is now estopped
from filing an annulment case against
John.
SUGGESTED ANSWER:
FALSE. Marsha is not estopped from
filing an annulment case against John on
the ground of his impotence, because
she learned of his impotence after the
celebration of the marriage and not
before.

Physical

incapacity

to

consummate is a valid ground for the


annulment of marriage if such incapacity
was existing at the time of the marriage,
continues and appears to be incurable.
The marriage may be annulled on this
ground

within

five

years

from

its

celebration.

Marriage; Annulment; Parties (2012)


No.IX.b) A petition for declaration of nullity
of a void marriage can only be filed by
either the husband or the wife? Do you
agree? Explain your answer. (5%)
SUGGESTED ANSWER:
Yes, I agree. Under the rules promulgated
by the Supreme Court, a direct action for

by any of the spouses.


ALTERNATIVE SUGGESTED ANSWER:

two business firms to G and their two


No, I do not agree. There are others who

children, aged 18 and 19.

may file a petition for declaration of

B also opened a bank account in the

nullity such as the other spouse in

amount of P3 million in the name of the two

bigamous marriages.

children to answer for their educational


expenses until they finish their college
degrees.

Marriage; Annulment; Support Pendente


Lite (2010)

day-to-day living expenses and upkeep of

No.V. G filed on July 8, 2000 a petition for


declaration of nullity of her marriage to B.
During the pendency of the case, the
couple

entered

agreement

For her part, G undertook to shoulder the

to

into
dissolve

compromise

their

absolute

community of property. B ceded his right to


their house and lot and all his shares in

the

children.

The

Court

approved

the

spouses agreement on September 8, 2000.


(A) Suppose the business firms suffered
reverses, rendering G unable to support
herself and the children. Can G still ask for
support pendente lite from B? Explain. (3%)

Never Let The Odds Keep You From Pursuing What You Know In Your Heart You Were Meant To Do.Leroy Satchel Paige

Page 22 of 180

Civil Law Q&As (2007-2013)

hectorchristopher@yahoo.com

Marriage;

SUGGESTED ANSWER:
If B acquiesces and does not file the

dbaratbateladot@gmail.com

Divorce

Decrees;

Filipino

Spouse Becoming Alien (2009)

action to impugn the legitimacy of the

No.IV. Harry married Wilma, a very wealthy

child within the prescriptive period for

woman. Barely five (5) years into the

doing so in Article 170 of the Family

marriage, Wilma fell in love with Joseph.

Code, G's daughter by another man shall

Thus, Wilma went to a small country in

be

Europe, became a naturalized citizen of

conclusively

presumed

as

the

that country, divorced Harry, and married

legitimate daughter of B by G.

Joseph.

year

thereafter,

Joseph

had squandered the P3 million fund for

permanent residence in the Philippines.

their college degrees, can they ask for more


support from B? Explain. (3%)

and

and

(B) Suppose in late 2004 the two children

their education before they could obtain

returned

Wilma

established

(A). Is the divorce obtained by Wilma from


Harry

recognized

in

the

Philippines?

Explain your answer. (3%)

SUGGESTED ANSWER:

SUGGESTED ANSRWER :

Yes, the two children can still ask for


support for schooling or training for

As to Wilma, the divorced obtained by

some

her

professions,

trade

or

vocation,

is

recognized

as

they shall have finished or completed

foreigner. Philippine personal laws do

their education (Article 194, Paragraph

not

2, Family Code; Javier v. Lucero, 94 Phil.

recognition of the divorce as regards

634 {1954}].Their having squandered the

Harry will depend on the applicability to

money given to them for their education

his case of the second paragraph of

will not deprive them of their right to

Article 26 of the Family Code. If it is

complete an education, or to extinguish

applicable, divorce is recognized as to

the obligation of the parents to ensure

him and, therefore, he can remarry.

the future of their children.

However, if it is not applicable, divorce is


not

recognized

is

the

Philippines

to

she

in

even beyond the age of majority until

apply

because

valid

foreigner.

as

to

now

However,

him

consequently, he cannot remarry.

and,

ALTERNATIVE ANSWER:

Never Let The Odds Keep You From Pursuing What You Know In Your Heart You Were Meant To Do.Leroy Satchel Paige

Page 23 of 180

Civil Law Q&As (2007-2013)

hectorchristopher@yahoo.com

dbaratbateladot@gmail.com

Wilma is already married to Joseph. Can


Yes , the divorce obtained by Wilma is
recognized as valid in the Philippines. At
the time she got the divorce, she was
already a foreign national having been
naturalized as a citizen of that small
country in Europe. Based on precedents
established

by

the

Supreme

Court

( Bayot v. CA, 570 SCRA 472 [2008]),


divorce

obtained

by

foreigner

is

recognized in the Philippines if validly


obtained in accordance with his or her
national law .
(B). If Harry hires you as his lawyer, what
legal recourse would you advise him to
take? Why? (2%)
SUGGESTED ANSWER:
I will advice Harry to:
(1) Dissolve and liquidate his property
relations with Wilma ; and
(2) If he will remarry, file a petition for
the recognition and enforcement of the
foreign

judgment

of

divorced

(Rule

39,Rules of Court ).

(C). Harry tells you that he has fallen in


love with another woman, Elizabeth, and
wants to marry her because, after all,

Harry legally marry Elizabeth? Explain.


(2%)

SUGGESTED ANSWER :

In Republic v. Obrecido, however, the


Supreme Court ruled that a Filipino

Yes, he can validly marry Elizabeth,

spouse is given the capacity to remarry

applying the doctrine laid down by the

even though the spouse who obtained

Supreme Court in Republic v. Obrecido

the divorce was a Filipino at the time of

(427 SCRA 114 [2005]). Under the second

the marriage, if the latter was already a

paragraph of Article 26 of the Family

foreigner when the divorce was already

Code, for the Filipino spouse to have

obtained abroad. According to the court,

capacity to remarry, the law expressly

to rule otherwise will violate the equal

requires the spouse who obtained the

protection clause of the Constitution.

divorce to be a foreigner at the time of


the marriage. Applying this requirement
to the case of Harry it would seem that
he is not given the capacity to remarry.
This is because Wilma was a Filipino at
the time of her marriage to Harry.

Marriage;

Divorce

Decrees;

Foreign

Spouse Divorces Filipino Spouse (2012)


(b) Cipriano and Lady Miros married each
other. Lady Miros then left for the US and
there, she obtained American citizenship.

Never Let The Odds Keep You From Pursuing What You Know In Your Heart You Were Meant To Do.Leroy Satchel Paige

Page 24 of 180

Civil Law Q&As (2007-2013)

hectorchristopher@yahoo.com

dbaratbateladot@gmail.com

(A). Under Article 26 of the Family Code,


Cipriano

later

learned

all

about

this

including the fact that Lady Miros has


divorced him in America and that she had
remarried there. He then filed a petition for
authority to remarry, invoking Par. 2, Art.
26

of

the

Family

Code.

Is

Cipriano

capacitated to re-marry by virtue of the


divorce decree obtained by his Filipino
spouse who was later naturalized as an
American citizen? Explain. (5%)
SUGGESTED ANSWER:
Yes, he is capacitated to remarry. While
the second paragraph of Art 26 of the
Family Code is applicable only to a
Filipino who married a foreigner at the
time of marriage, the Supreme Court
ruled in the case of Republic v. Orbecido,
G.R. No. 154380, 5 Oct, 2005, that the
said

provision

equally

applies

to

Filipino who married another Filipino at


the time of the marriage, but who was
already a foreigner when the divorce was
obtained.

Marriage;

Divorce

Decrees;

Foreign

Spouse Divorces Filipino Spouse (2010)


No.I. True or False.

when a foreign spouse divorces his/her


Filipino spouse, the latter may re-marry by

abroad

by

the

proving only that the foreign spouse has

sufficient

to

obtained a divorce against her or him

spouse to remarry.

foreigner

capacitate

spouse

the

is

Filipino

abroad. (1%)
SUGGESTED ANSWER :

Marriage; Legal Separation; Prescription

FALSE, In Garcia v. Recio , 366 SCRA

(2012)

437 (2001) , the SC held that for a


Filipino

spouse

to

have

capacity

to

contract a subsequent marriage, it must


also be proven that the foreign divorced
obtained abroad by the foreigner spouse
give such foreigner spouse capacity to
remarry.
ALTERNATIVE ANSWER:

No.IV.a)

After

they

got

married,

Nikki

discovered that Christian was having an


affair with another woman. But Nikki
decided to give it a try and lived with him
for two (2) years. After two (2) years, Nikki
filed an action for legal separation on the
ground of Christians sexual infidelity. Will
the action prosper? Explain. (5%)

TRUE, Art 26 (2) (FC), clearly provides


that the decree of divorce obtained

Never Let The Odds Keep You From Pursuing What You Know In Your Heart You Were Meant To Do.Leroy Satchel Paige

Page 25 of 180

Civil Law Q&As (2007-2013)

hectorchristopher@yahoo.com

dbaratbateladot@gmail.com

FALSE. The five-year prescriptive period


for filing legal separation runs from the

SUGGESTED ANSWER:
Although the action for legal separation
has not yet prescribed, the prescriptive
period being 5 years, if Obecidos affair
with another woman was ended when
Nikki decided to live with him again,
Nikkis

action

account

of

will

not

condonation.

prosper
However,

on
if

such affair is still continuing, Nikkis


action would prosper because the action
will surely be within five (5) years from
the commission of the latest act of
sexual infidelity. Every act of sexual
liaison is a ground for legal separation.

Marriage; Legal Separation; Prescription


(2007)
No.VII. Write "TRUE" if the statement is
true or "FALSE" if the statement is false. If
the statement is FALSE, state the reason.
(2% each).
(2). If a man commits several acts of sexual
infidelity, particularly in 2002, 2003, 2004,
2005, the prescriptive period to file for legal
separation runs from 2002.
SUGGESTED ANSWER:

occurrence

of

sexual

infidelity

committed in 2002 runs from 2002, for

drunkard, a womanizer, a gambler, and a


the sexual infidelity committed in 2003,
the prescriptive period runs from 2003
and

so

on.

The

action

for

legal

separation for the last act of sexual


infidelity in 2005 will prescribe in 2010.

mama's boy- traits that she never knew or


saw when Neil was courting her. Although
summoned, Neil did not answer Maria's
petition and never appeared in court.
To support her petition, Maria presented
three witnesses- herself, Dr. Elsie Chan,
and Ambrosia. Dr. Chan testified on the

Marriage;

Psychological

Incapacity

(2013)

psychological

report

on

Neil

that

she

prepared. Since Neil never acknowledged


n9r

responded

to

her

invitation

for

No.I. You are a Family Court judge and

interviews, her report is solely based on her

before you is a Petition for the Declaration

interviews with Maria and the spouses'

of Nullity of Marriage (under Article 36 of

minor children. Dr. Chan concluded that

the Family Code)filed by Maria against Neil.

Neil

Maria claims that Neil is psychologically

Personality Disorder, an ailment that she

incapacitated to comply with the essential

found to be already present since Neil's

is

suffering

from

Narcissistic

obligations of marriage because Neil is a


Never Let The Odds Keep You From Pursuing What You Know In Your Heart You Were Meant To Do.Leroy Satchel Paige

Page 26 of 180

Civil Law Q&As (2007-2013)

hectorchristopher@yahoo.com

dbaratbateladot@gmail.com

serious enough to prevent Neil from


early adulthood and one that is grave and

performing

incurable. Maria testified on the specific

obligations. Dr. Chans report contains

instances when she found Neil drunk, with


another

woman,

or

squandering

the

family's resources in a casino. Ambrosia,


the

spouses'

current

household

help,

corroborated Maria's testimony.


On the basis of the evidence presented, will
you grant the petition? (8%)
SUGGESTED ANSWER:
No. The petition should be denied.
The psychological incapacity under Art.
36

of

the

Family

Code

must

be

characterized by (a) gravity, (b) juridical


antecedence, and (c) incurability. It is
not enough to prove that the parties
failed to meet their responsibilities and
duties as married persons; it is essential
that they must be shown to be incapable
of doing so, due to some physiological
(not physical) illness (Republic v. CA and
Molina, G.R. No. 108763, Feb 13, 1997).
In this case, the pieces of evidence
presented are not sufficient to conclude
that

indeed

Neil

psychological
Personality
before

the

is

suffering

incapacity

Disorder]
marriage,

from

[Narcissistic

existing

already

incurable

and

his

essential

marital

mere conclusions. Being a drunkard, a


womanizer, a gambler and a mamas boy,
merely shows Neils failure to perform
his marital obligations. In a number of
cases, the Supreme Court did not find
the existence of psychological incapacity
in cases where the respondent showed
habitual

drunkenness

(Republic

v.

Melgar, G.R. No. 139676, 2006), blatant


display of infidelity and irresponsibility
(Dedel v. CA, 2004) or being hooked to
gambling and drugs (Republic v. TanyagSan Jose, G.R. No. 168328, 2007).
ALTERNATIVE ANSWER:

Yes. The petition should be granted.


The personal medical or psychological
examination

of

requirement

respondent is
for

psychological

not

declaration

incapacity.

It

is

a
of

the

totality of the evidence presented which


shall

determine

psychological

the

incapacity

existence

of

(Marcos

v.

Marcos, G.R. No. 136490, Oct 19, 2000).


Dr. Chans report corroborated by Marias
and Ambrosias testimonies, therefore,
sufficiently prove Neils psychological
incapacity

to

assume

his

marital

obligations.

Never Let The Odds Keep You From Pursuing What You Know In Your Heart You Were Meant To Do.Leroy Satchel Paige

Page 27 of 180

Civil Law Q&As (2007-2013)

hectorchristopher@yahoo.com

dbaratbateladot@gmail.com

cannot be annulled on the ground of


Marriage;

Psychological

Incapacity

(2012)
No.II.b) The petitioner filed a petition for
declaration of nullity of marriage based
allegedly on the psychological incapacity of
the respondent, but the psychologist was
not

able

to

personally

examine

the

respondent and the psychological report


was

based

petitioner.

only

on

Should

the

the

narration

annulment

of
be

granted? Explain. (5%)


SUGGESTED ANSWER:
The annulment cannot be guaranteed
solely on the basis of the psychological
report. For the report to prove the
psychological
respondent,

it

incapacity

of

the

is

that

the

required

psychologist should personally examine


the respondent and the psychological
report

should

be

based

on

the

psychologists independent assessment


of the facts as to whether or not the
respondent

is

psychologically

incapacitated.
Since,

the

psychologist

did

not

personally examine the respondent, and


his report is based solely on the story of
the petitioner who has an interest in the
outcome of the petition, the marriage

respondents psychological incapacity if

When Faye was 25 years old, Brad discovered

the said report is the only evidence of


respondents psychological incapacity.

her continued liason with Roderick and in one


of their heated arguments, Faye shot Brad to
death. She lost no time in marrying her true
love Roderick, without a marriage license,

Marriage; Requisites (2008)

claiming that they have been continuously


cohabiting for more than 5 years.

No. III. Roderick and Faye were high school


sweethearts. When Roderick was 18 and Faye,

(A). Was the marriage of Roderick and Faye

16 years old, they started to live together as

valid? (2%)

husband and wife without the benefit of


marriage. When Faye reached 18 years of age,
her parents forcibly took her back and
arranged for her marriage to Brad. Although
Faye lived with Brad after the marriage,
Roderick continued to regularly visit Faye
while Brad was away at work. During their
marriage, Faye gave birth to a baby girl, Laica.

SUGGESTED ANSWER:
No. The marriage of Roderick and Faye is
not valid. Art. 4, FC provides that the
absence of any of the essential or formal
requisites renders the marriage void ab
initio. However, no license shall be

Never Let The Odds Keep You From Pursuing What You Know In Your Heart You Were Meant To Do.Leroy Satchel Paige

Page 28 of 180

Civil Law Q&As (2007-2013)

hectorchristopher@yahoo.com

dbaratbateladot@gmail.com

a divorced German national born of a


necessary for the marriage of a man and
a woman who have lived together as
husband and wife for at least 5 years and
without any legal impediment to marry
each other. In Republic v. Dayot, G.R.
No. 175581, 28 March 2008, reiterating
the doctrine in Nial v. Bayadog, G.R.
No. 133778, 14 March 2000, this fiveyear

period

exclusivity

is

and

characterized
continuity.

In

by
the

present case, the marriage of Roderick


and Faye cannot be considered as a
marriage

of

exceptional

character,

because there were 2 legal impediments


during their cohabitation: minority on
the part of Faye, during the first two
years of cohabitation; and, lack of legal
capacity, since Faye married Brad at the
age of 18. The absence of a marriage
license made the marriage of Faye and
Roderick void ab initio.

Marriage; Subsequent Marriage (2008)


No. I. Ana Rivera had a husband, a Filipino
citizen

like

her,

who

was

among

the

passengers on board a commercial jet plane


which crashed in the Atlantic Ocean ten
(10) years earlier and had never been heard
of ever since. Believing that her husband
had died, Ana married Adolf Cruz Staedtler,

German

father

and

Filipino

residing in Stuttgart. To avoid being

mother

subsistence of his previous marriage


reqiured to submit the required certificate

provided that: (a) his prior spouse in the

of capacity to marry from the German

first marriage had been absent for four

Embassy in Manila, Adolf stated in the

consecutive years; (b) that the spouse

application for marriage license that he was

present has a well-founded belief that

a Filipino citizen. With the marriage license

the absent spouse was already dead, and

stating that Adolf was a Filipino, the couple

(C) present spouse instituted a summary

got married in a ceremony officiated by the


Parish Priest of Calamba, Laguna in a
beach in Nasugbu, Batangas, as the local
parish

priest

refused

to

solemnize

marriages except in his church. Is the

proceeding for the declaration of the


presumptive death of absent spouse.
Otherwise, the second marriage shall be
null and void. In the instant case, the
husband

marriage valid? Explain fully. (5%)

of

Ana

was

among

the

passengers on board a commercial jet


SUGGESTED ANSWER:

plane which crashed in the Atlantic

No. The marriage is not valid. Art. 41 FC


allows the present spouse to contract a
subsequent

marriage

during

the

Ocean.

The

body

of

the

deceased

husband was not recovered to confirm


his death. Thus, following Art. 41, Ana

Never Let The Odds Keep You From Pursuing What You Know In Your Heart You Were Meant To Do.Leroy Satchel Paige

Page 29 of 180

Civil Law Q&As (2007-2013)

hectorchristopher@yahoo.com

dbaratbateladot@gmail.com

Yes. Jon and Jane can marry each other;


should

have

first

secured

judicial

declaration of his presumptive death


before she married Adolf. The absence of
the

said

judicial

declaration

incapacitated Ana from contracting her


second marriage, making it void ab
initio.

Marriage; Void Marriages; By Reason of


Public Policy (2008)
No.V. Despite several relationships with
different
unmarried.

women,
His

Andrew

first

remained

relationship

with

Brenda produced a daughter, Amy, now 30


years old. His second, with Carla, produced
two sons: Jon and Ryan. His third, with
Donna, bore him no children although
Elena has a daughter Jane, from a previous
relationship. His last, with Fe, produced no
biological

children

but

they

informally

adopted without court proceedings, Sandy's


now 13 years old, whom they consider as
their own. Sandy was orphaned as a baby
and was entrusted to them by the midwife
who attended to Sandy's birth. All the
children, including Amy, now live with
andrew in his house.
(D). Can Jon and Jane legally marry? (1%)
SUGGESTED ANSWER:

Jon is an illegitimate child of Andrew

20 years old. David had a son, Julian, with


while Jane is a child of Elena from a
previous

relationship.

Thus,

their

marriage is not one of the prohibited

his ex-girlfriend Sandra. Julian and Thelma


can get married.
SUGGESTED ANSWER:

marriages enumerated under Art. 38 of


the FC.

TRUE.

Julian

and

Thelma

can

get

married. Marriage between stepbrothers


and
Marriage; Void Marriages; By Reason of
Public Policy (2007)

stepsisters

are

not

among

the

marriages prohibited under the Family


Code.

No. VII. Write "TRUE" if the statement is


true or "FALSE" if the statement is false. If
the statement is FALSE, state the reason.
(2% each).
(5). Amor gave birth to Thelma when she
was 15 years old. Thereafter, Amor met

Marriage;

Void

Marriages;

Property

Relations (2009)
No. III. In December 2000, Michael and
Anna, after obtaining a valid marriage

David and they got married when she was


Never Let The Odds Keep You From Pursuing What You Know In Your Heart You Were Meant To Do.Leroy Satchel Paige

Page 30 of 180

Civil Law Q&As (2007-2013)

hectorchristopher@yahoo.com

dbaratbateladot@gmail.com

of the Family Code. This is on the


license, went to the Office of the Mayor of
Urbano, Bulacan, to get married. The
Mayor

was

not

there,

but

the

Mayors

secretary asked Michael and Anna and their


witnesses to fill up and sign the required
marriage contract forms. The secretary then
told them to wait, and went out to look for the
Mayor who was attending a wedding in a
neighboring municipality.

When the secretary caught up with the


Mayor at the wedding reception, she showed
him the marriage contract forms and told
him that the couple and their witnesses
were waiting in his office. The Mayor
forthwith signed all the copies of the
marriage

contract,

gave

them

to

the

secretary who returned to the Mayors


office. She then gave copies of the marriage
contract to the parties, and told Michael
and Anna that they were already married.
Thereafter, the couple lived together as
husband and wife, and had three sons.
(C). What property regime governs the
properties acquired by the couple? Explain.
(2%)
SUGGESTED ANSWER:
The marriage being void, the property
relationship that governed their union is
special co-ownership under Article 147

assumption that there was no

impediment for them to validity marry


each other.

When the secretary caught up with the


Mayor at the wedding reception, she showed
him the marriage contract forms and told
him that the couple and their witnesses

Marriage;

Void

Marriages;

Status

of

Children (2009)

Anna, after obtaining a valid marriage


license, went to the Office of the Mayor of
Urbano, Bulacan, to get married. The
was

not

forthwith signed all the copies of the


marriage

No. III. In December 2000, Michael and

Mayor

were waiting in his office. The Mayor

there,

but

the

Mayors

secretary asked Michael and Anna and their


witnesses to fill up and sign the required
marriage contract forms. The secretary then
told them to wait, and went out to look for the
Mayor who was attending a wedding in a

contract,

gave

them

to

the

secretary who returned to the Mayors


office. She then gave copies of the marriage
contract to the parties, and told Michael
and Anna that they were already married.
Thereafter, the couple lived together as
husband and wife, and had three sons.
(A). Is the marriage of Michael and Anna
valid,

voidable,

or

void?

Explain

your

answer. (3%)

neighboring municipality.
Never Let The Odds Keep You From Pursuing What You Know In Your Heart You Were Meant To Do.Leroy Satchel Paige

Page 31 of 180

Civil Law Q&As (2007-2013)

hectorchristopher@yahoo.com

dbaratbateladot@gmail.com

representing that he was single when, in


fact, he was not; and exemplary, to teach a

SUGGESTED ANSWER :

lesson to like-minded Lotharios.

The marriage is void because the formal


requisite

of

marriage

ceremony

was

absent ( Art.3, F.C. 209, Family Code).


ALTERNATIVE ANSWER:
The

marriage

is

void

because

an

essential requisite was absent: consent


of

the

parties

freely

given

in

the

presence of the solemnizing officer (Art


.2, FC).
(B). What is the status of the three children
of Michael and Anna? Explain your answer.
(2%)
SUGGESTED ANSWER:
The children are illegitimate, having
been born outside a valid marriage.

Parental Authority; Illegitimate Minor


Child (2009)
No.XIV. Rodolfo, married to Sharon, had an
illicit affair with his secretary, Nanette, a
19-year old girl, and begot a baby girl,
Rona. Nanette sued Rodolfo for damages:
actual, for hospital and other medical
expenses

in

delivering

the

child

by

caesarean section; moral, claiming that


Rodolfo

promised

to

marry

her,

and even though he is giving support for


(C). When Rona reaches seven (7) years old,

the child. To acquire custody over Rona,

she tells Rodolfo that she prefers to live

Rodolfo should first deprive Nanette of

with

off

parental authority if there is ground

financially than Nanette. If Rodolfo files an

under the law, and in a proper court

action for the custody of Rona, alleging that

proceedings. In the same action, the

he is Ronas choice as custodial parent, will

court may award custody of Rona to

him,

because

he

is

better

the court grant Rodolfos petition? Why or


why not? (2%)
SUGGESTED ANSWER:
No, because Rodolfo has no parental
authority over Rona. He who has the
parental

authority

has

the

right

Rodolfo if it is for her best interest.

Parental Authority; In Vitro Fertilization


(2010)

to

No.VI. Gigolo entered into an agreement

custody. Under the Family Code, the

with Majorette for her to carry in her womb

mother alone has parental authority over

his baby via in vitro fertilization. Gigolo

the illegitimate child. This is true even if

undertook to underwrite Majorettes pre-

illegitimate father recognized the child

natal expenses as well as those attendant

Never Let The Odds Keep You From Pursuing What You Know In Your Heart You Were Meant To Do.Leroy Satchel Paige

Page 32 of 180

Civil Law Q&As (2007-2013)

hectorchristopher@yahoo.com

dbaratbateladot@gmail.com

The relationship produced one offspring,


to her delivery. Gigolo would thereafter pay
Majorette P2 million and, in return, she
would give custody of the baby to him.
After Majorette gives birth and delivers the
baby to Gigolo following her receipt of P2
million, she engages your services as her
lawyer to regain custody of the baby.
(C) Who of the two can exercise parental
authority over the child? Explain. (2.5%)
SUGGESTED ANSWER:
Majorette,

the

mother,

can

exercise

parental authority. Since the child was


born

out

of

wedlock,

the

child

is

illegitimate and the mother has the


exclusive parental authority and custody
over the child.
ALTERNATIVE ANSWER:
Gigolo can exercise parental authority
over the child. Majorette has no blood
relation to the child. She is just a
carrier of the child.

Paternity & Filiation; Child Born Under a


Void Marriage (2010)
No.X. In 1997, B and G started living
together without the benefit of marriage.

Venus. The couple acquired a residential lot

legitimated by the subsequent marriage


in Paraaque. After four (4) years or in

of her parents, such legitimation was

2001,

rendered

having

completed

her

4-year

ineffective

when

the

said

college degree as a fulltime student, she

marriage was later on declared null and

and

void

contracted

marriage

without

due

to

absence

of

marriage

license.

license.

The marriage of B and G was, two years

Under Article 178 of the Family Code,

later, declared null and void due to the

legitimation

absence of a marriage license.

subsequent

(B). Is Venus legitimate, illegitimate, or


legitimated? Explain briefly. (3%)

shall
valid

marriage

underscored

Venus is illegitimate. She was conceived


and born outside a valid marriage. Thus,
she is considered illegitimate (Art 165,
Family

Code).

While

Venus

place

marriage

by

between

parents. The annulment of a voidable


shall

legitimation.
SUGGESTED ANSWER:

take

not

The
portion

affect

inclusion
in

the

the

of

the

Article

necessarily implies that the Article's


application

is

limited

to

voidable

marriages. It follows that when the

was

Never Let The Odds Keep You From Pursuing What You Know In Your Heart You Were Meant To Do.Leroy Satchel Paige

Page 33 of 180

Civil Law Q&As (2007-2013)

hectorchristopher@yahoo.com

dbaratbateladot@gmail.com

presumed as the child of B under Article


subsequent marriage is null or void, the
legitimation must also be null and void.
In the present problem, the marriage
between B and G was not voidable but
void. Hence, Venus has remained an
illegitimate child.

Paternity

&

Filiation;

Impugning

Legitimacy (2010)
No.IV.

Spouses

offsprings.

Albeit

personality

and
they

differences,

begot

had
the

two

serious
spouses

continued to live under one roof. B begot a


son by another woman. G also begot a
daughter by another man.
(A). If G gives the surname of B to her
daughter by another man, what can B do to
protect their legitimate children's interests?
Explain. (5%)
SUGGESTED ANSWER:
B can impugn the status of G's daughter
by

another

daughter

on

man
the

as

his

ground

legitimate
that

for

biological reason he could not have been


the father of the child, a fact that may
be proven by the DNA test. Having been
born during the marriage between B and
G, G's daughter by another man is

164 of the Family Code. In the same

action to impugn, B can pray for the


correction of the status of the said

Paternity

&

daughter in her record of birth.

Fertilization;

Filiation;
Surrogate

In

Vitro

Mothers

Remedy to Regain Custody (2010)


(B). If B acquiesces to the use of his
surname by Gs daughter by another man,

No.VI. Gigolo entered into an agreement

what is/are the consequence/s? Explain.

with Majorette for her to carry in her womb

(5%)

his baby via in vitro fertilization. Gigolo


undertook to underwrite Majorettes pre-

SUGGESTED ANSWER:

natal expenses as well as those attendant

If B acquiesces and does not file the


action to impugn the legitimacy of the
child within the prescriptive period for

to her delivery. Gigolo would thereafter pay


Majorette P2 million and, in return, she
would give custody of the baby to him.

doing so in Article 170 of the Family

After Majorette gives birth and delivers the

Code, G's daughter by another man shall

baby to Gigolo following her receipt of P2

be

conclusively

presumed

as

the

legitimate daughter of B by G.

Never Let The Odds Keep You From Pursuing What You Know In Your Heart You Were Meant To Do.Leroy Satchel Paige

Page 34 of 180

Civil Law Q&As (2007-2013)

hectorchristopher@yahoo.com

dbaratbateladot@gmail.com

(B) Can Gigolo demand from Majorette the


million, she engages your services as her
lawyer to regain custody of the baby.
(A) What legal action can you file on behalf
of Majorette? Explain. (2.5%)
SUGGESTED ANSWER:
As her lawyer, I can file a petition for
habeas corpus on behalf Majorette to
recover custody of her child. Since she is
the mother of the child that was born
out

of

wedlock,

she

has

exclusive

parental authority and custody over the


child. Gigolo, therefore, has no right to
have custody of the child and his refusal
to give up custody will constitute illegal
detention for which habeas corpus is the
proper remedy.
ALTERNATIVE ANSWER:
The action to regain custody will not
prosper. In the first place Majorette
cannot regain custody of the baby. As
surrogate mother she merely carries the
child in her womb for its development.
The child is the child of the natural
parents- Gigolo and his partner. The
agreement between Gigolo and Majorette
is a valid agreement.

return of the P2 million if he returns the


baby? Explain. (2.5%)
SUGGESTED ANSWER:

If Gigolo voluntarily recognized the child


No, he cannot. Both he and Majorette are

as his illegitimate child in accordance

guilty of violating the provision of the

with Article 175 in relation to Article

Anti-Child Abuse Law (RA7610) on child

172 of the Family Code, the child is

trafficking. Being in pari delicto, the

entitled to support and inheritance from

partners shall be left where they are and

Gigolo.

Gigolo cannot demand the return of what


he paid.
ALTERNATIVE ANSWER:

ALTERNATIVE ANSWER:
Yes, because Gigolo is the natural and
biological parent of the baby.

Yes. The agreement between Gigolo and


Majorette is a valid agreement.
Paternity

&

Filiation;

Legitimacy;

(D) Is the child entitled to support and

Presumption (2008)

inheritance from Gigolo? Explain. (2.5%)

No. III. Roderick and Faye were high school

SUGGESTED ANSWER:

sweethearts. When Roderick was 18 and


Faye, 16 years old, they started to live
together as husband and wife without the

Never Let The Odds Keep You From Pursuing What You Know In Your Heart You Were Meant To Do.Leroy Satchel Paige

Page 35 of 180

Civil Law Q&As (2007-2013)

hectorchristopher@yahoo.com

dbaratbateladot@gmail.com

No. Laica cannot bring an action to


benefit of marriage. When Faye reached 18

impugn her own status. In Liyao Jr. v.

years of age, her parents forcibly took her

Tanhoti-Liyao, G.R. No. 138961, 07

back and arranged for her marriage to Brad.


Although Faye lived with Brad after the
marriage, Roderick continued to regularly
visit Faye while Brad was away at work.
During their marriage, Faye gave birth to a
baby girl, Laica. When Faye was 25 years old,
Brad discovered her continued liason with
Roderick

and

in

one

of

their

heated

arguments, Faye shot Brad to death. She lost


no time in marrying her true love Roderick,
without a marriage license, claiming that they
have been continuosly cohabiting for more
than 5 years.

(B). What is the filiation status of Laica?


(2%)
SUGGESTED ANSWER:
Laica

is

legitimate

because

children

conceived or born during the marriage of


the

parents

are

presumed

to

be

legitimate (Art. 164, FC).


(C).Can Laica bring an action to impugn her
own status on the ground that based on
DNA results, Roderick is her biological
father? (2%)
SUGGESTED ANSWER:

No. Laica cannot be legitimated by the


March 2002, the Supreme Court ruled

marriage

that impugning the legitimacy of the

because only children conceived and

child is a strictly personal right of

born outside of wedlock of parents who

husband, except: (a) when the husband

at the time of the conception of the

died before the expiration of the period

former were not disqualified by any

fixed for bringing the action; (b) if he

impediment to marry each other may be

should

legitimated (Art. 177, FC).

die

complaint,

after

the

without

filing

having

of

the

of

her

biological

parents

desisted

therefrom, or (c) if the child was born


after the death of the husband. Laica's

Paternity & Filiation; Legitimation of a

case does not fall under any of the

Child from a Previous Valid Marriage

exceptions.

(2008)

(D).

Can

Laica

be

legitimated

by

marriage of her biological parents? (1%)


SUGGESTED ANSWER:

the

No. IV. Gianna was born to Andy and


Aimee, who at the time Gianna's birth were
not married to each other. While Andy was
single at the time, Aimee was still in the

Never Let The Odds Keep You From Pursuing What You Know In Your Heart You Were Meant To Do.Leroy Satchel Paige

Page 36 of 180

Civil Law Q&As (2007-2013)

hectorchristopher@yahoo.com

process of securing a judicial declaration of


nullity on her marriage to her ex-husband.
Gianna's birth certificate, which was signed
by both Andy and Aimee, registered the
status

of

Gianna

as

"legitimate",

her

surname carrying that of Andy's and that


her parents were married to each other.
(C). Assuming that Aimee is successful in
declaring her former marriage void, and
Andy and Aimee subsequently married each
other, would Gianna be legitimated? (1%)
SUGGESTED ANSWER:
Gianna cannot be legitimated by the
subsequent marriage of Andy and Aimee.
Art. 177 of the FC provides that "only
children conceived and born outside of
wedlock of parents who, at the time of
the conception of the former, were not
disqualified by any impediment to marry
each other may be legitimated." In the
present case, a legal impediment was
existing at the time of the conception of
Gianna. Her mother, Aimee, was still
alive in the process of securing judicial
declaration of nullity on her marriage to
her ex-husband.

Paternity & Filiation; Legitimation of a


Dead Child (2009)

dbaratbateladot@gmail.com

No. I. TRUE or FALSE. Answer TRUE if the


statement is true, or FALSE if the

statement is false. Explain your answer in


not more than two (2) sentences.

Paternity

&

Ascendants &

(E). A dead child can be legitimated. (1%)

Filiation;

Support:

Descendants; Collateral

Blood Relatives (2008)


No.V. Despite several relationships with

SUGGESTED ANSWER:

different

women,

unmarried.

require a child to be alive at the same

Brenda produced a daughter, Amy, now 30

time of the marriage of his / her parents

years old. His second, with Carla, produced

181 of the Family Code which states


that [Th]e llegitimation of children who
died before the celebration of marriage
will benefit their descendants, does not
preclude

instances

where

such

first

remained

TRUE To be legitimated, the law does not

( Article 177, FC ). Furthermore, Art.

His

Andrew

relationship

with

two sons: Jon and Ryan. His third, with


Donna, bore him no children although
Elena has a daughter Jane, from a previous
relationship. His last, with Fe, produced no
biological

children

but

they

informally

adopted without court proceedings, Sandy's


now 13 years old, whom they consider as

legitimation will benefit no one but the


child's ascendants ,or other relatives .
Never Let The Odds Keep You From Pursuing What You Know In Your Heart You Were Meant To Do.Leroy Satchel Paige

Page 37 of 180

Civil Law Q&As (2007-2013)

hectorchristopher@yahoo.com

dbaratbateladot@gmail.com

their own. Sandy was orphaned as a baby

Paternity & Filiation; Use of Surname;

and was entrusted to them by the midwife

Illegitimate Child (2009)

who attended to Sandy's birth. All the


children, including Amy, now live with
andrew in his house.

No.XIV. Rodolfo, married to Sharon, had an


illicit affair with his secretary, Nanette, a
19-year old girl, and begot a baby girl,

(B). In his old age, can Andrew be legally

Rona. Nanette sued Rodolfo for damages:

entitled to claim support from Amy, Jon,

actual, for hospital and other medical

Ryan, Vina, Wilma, and Sandy assuming

expenses

that all of them have the means to support

caesarean section; moral, claiming that

him? (1%)

Rodolfo

in

delivering

promised

to

the

child

marry

by

her,

representing that he was single when, in


SUGGESTED ANSWER:
Andrew, in his old age, cannot be legally

fact, he was not; and exemplary, to teach a


lesson to like-minded Lotharios.

entitled to claim support because Art.

(B). Suppose Rodolfo later on acknowledges

195, par 2 of the FC limits the giving of

Rona and gives her regular support, can he

support to "legitimate ascendants and

compel her to use his surname? Why or

descendants."

why not? (2%)

(C). Can Amy, Jon, Ryan, Vina, Wilma, and

SUGGESTED ANSWER:

Sandy legally claim support from each

No. he has no right to compel Rona to

other? (2%)

use his surname. The law does not give

SUGGESTED ANSWER:
Amy, Jon, Ryan, Vina, Wilma and Sandy
cannot legally claim support from each
other because Art. 195, par 5 limits the
giving of support to "legitimate brothers
and sisters, whether full or half blood."

him the right simply because he gave her


support (RA 9255).

Under the Family Code, an illegitimate


child was required to use only the
surname of the mother. Under RA 9255,
otherwise known as the Revilla law,
however, the illegitimate child is given
the option to use the surname of the
illegitimate father when the latter has

recognized the former in accordance

illegitimate child, Rodolfo cannot

with law. Since the choice belongs to the

Never Let The Odds Keep You From Pursuing What You Know In Your Heart You Were Meant To Do.Leroy Satchel Paige

Page 38 of 180

Civil Law Q&As (2007-2013)

hectorchristopher@yahoo.com

compel Rona, if already of age, to use


the surname against her will. If Rona is
still a minor, to use the surname of
Rodolfo

will

require

the

consent

of

Rona's mother who has sole parental


authority over her.

Paternity & Filiation; Who May Impugn


Legitimacy (2009)
No.V.

Four

children,

namely:

Alberto,

Baldomero, Caridad, and Dioscoro, were


born to the spouses Conrado and Clarita de
la Costa. The childrens birth certificates
were duly signed by Conrado, showing them
to be the couples legitimate children.
Later, one Edilberto de la Cruz executed a
notarial document acknowledging Alberto
and Baldomero as his illegitimate children
>with

Clarita.

Edilberto

died

leaving

substantial properties. In the settlement of


his

estate,

Alberto

intervened

claiming

deceaseds

illegitimate

and
shares

Baldomero
as

the

children.

The

legitimate family of Edilberto opposed the


claim.
Are Alberto and Baldomero entitled to share
in the estate of Edilberto? Explain. (4%)
SUGGESTED ANSWER:

No,

Alberto

dbaratbateladot@gmail.com

and

Baldomero

are

entitled to share in Edilbertos estate.

not

illegitimate children. Before they can be


They are not related at all to Edilberto.

conferred

They were born during the marriage of

illegitimate

Conrado

first

and

Clarita,

hence,

are

the

status

children,

impugn

their

of

Edilbertos

Condrado

must

legitimacy.

Since

considered legitimate children of the

Condrado has not initiated any action to

said spouses. This status is conferred on

impugn their legitimacy, they continue

them at birth by law.

to be the legitimate of Condrado. They


cannot be the illegitimate children of

Under Philippine law, a person cannot

Edilberto at the same time. Not being

have more than one natural filiation.

the illegitimate children of Edilberto,

The legitimate filiation of a person can

they have no right to inherit from him.

be changed only if the legitimate father


will successfully impugn such status.

In the problem, therefore, the filiation of


Alberto and Baldomero as legitimate
children of Condrado cannot be changed

Property

Relations;

Adulterous

Relationship (2009)
No. XI. TRUE or FALSE. Answer TRUE if
the statement is true, or FALSE if the

by their recognition by Edilberto as his


Never Let The Odds Keep You From Pursuing What You Know In Your Heart You Were Meant To Do.Leroy Satchel Paige

Page 39 of 180

Civil Law Q&As (2007-2013)

hectorchristopher@yahoo.com

dbaratbateladot@gmail.com

building conjugal or paraphernal? Reasons.


statement is false. Explain your answer in
not more than two (2) sentences.
(B). If there is no marriage settlement, the
salary of a "spouse" in an adulterous
marriage

belongs

to

the

conjugal

partnership of gains. (1%)


SUGGESTED ANSWER:
False. In adulterous relationship, the
salary of a married partner belongs to
the absolute community, or conjugal
partnership, of such married partner
with his or her lawful spouse. Under
Articles 148 of the Family Code, the
property
partner

relations
and

governed

between

his/her

by

ordinary

married

paramour

is

co-ownership

where the partners become co-owners


only

when

they

acquisition
paramour

of
is

contributed
the

deemed

to

property.
to

have

the
The
not

contributed in the earning of the salary


of the married partner.

Property Relations; Accession (2012)


No.III.(a) Maria, wife of Pedro, withdrew P 5
Million from their conjugal funds. With this
money, she constructed a building on a lot
which she inherited from her father. Is the

(5%)

Note: The rule on reverse accession is


SUGGESTED ANSWER:

applicable only to the regime of conjugal

It depends. If the value of the building is


more than the value of the land, the
building

is

conjugal

and

the

land

becomes conjugal property under Art.

partnership of gains in both the Family


Code and the New Civil Code. The foregoing
answer assumes that CPG is the regime of
the property relations of the spouses.

120 of the Family Code. This is a case of


reverse accession, where the building is
considered as the principal and the land,
the accessory. If, on the other hand, the

Property Relations; Ante-Nuptial Debt


(2007)

value of the land is more than the value

No. VII. Write "TRUE" if the statement is

of the building, then the ordinary rule of

true or "FALSE" if the statement is false. If

accession applies where the land is the

the statement is FALSE, state the reason.

principal

and

(2% each).

accessory.

In

the
such

building,
case,

the

the
land

remains paraphernal property and the

(3). An individual, While single, purchases a


house and lot in 1990 and borrows money

building becomes paraphernal propery.


Never Let The Odds Keep You From Pursuing What You Know In Your Heart You Were Meant To Do.Leroy Satchel Paige

Page 40 of 180

Civil Law Q&As (2007-2013)

in

1992

to

repair

it.

hectorchristopher@yahoo.com

In

1995,

such

individual gets married while the debt is

Property

dbaratbateladot@gmail.com

Relations;

Unions

Without

Marriage (2012)

still being paid. After the marriage, the debt


is still the responsibility of such individual.

No.V. a) Spouses Primo and Monina Lim,


childless, were entrusted with the custody

SUGGESTED ANSWER:

of two (2) minor children, the parents of


whom were unknown. Eager of having

FALSE.

The

absolute

Community

of

children of their own, the spouses made it

property is liable for the ante-nuptial

appear

debts of either spouse in so far as the

parents by naming them Michelle P. Lim

same redounded to the benefit of the

and Michael Jude Lim. Subsequently,

family (Art. 94 par.7, FC).

Monina married Angel Olario after Primos

that

they

were

the

childrens

death.
ALTERNATIVE ANSWER:
She decided to adopt the children by
FALSE.

The

the

availing the amnesty given under R.A. 8552

community

to those individuals who simulated the

property, because the property already

birth of a child. She filed separate petitions

constitutes

community

for the adoption of Michelle, then 25 years

property under Art. 91 of FC which took

old and Michael, 18. Both Michelle and

responsibility

debt

is

of

the

absolute

already

effect in 1988 while the house and lot

Michael gave consent to the adoption.

here involved was purchased in 1990.

The trial court dismissed the petition and

There is no indication that the spouse

ruled that Monina should have filed the

who bought the property had legitimate

petition jointly with her new husband.

descendants by a former marriage, which

Monina, in a Motion for Reconsideration

would exclude the house and lot from

argues that mere consent of her husband

the community property, Art. 92 par 3,

would suffice and that joint adoption is not

FC). If the spouses established a conjugal

needed,

partnership, the property belongs to the

for

the

adoptees

are

already

emancipated.

individual spouse if full ownership was

Is the trial court correct in dismissing the

vested before marriage (Art. 118, FC).

petitions for adoption? Explain. (5%)

SUGGESTED ANSWER:

Never Let The Odds Keep You From Pursuing What You Know In Your Heart You Were Meant To Do.Leroy Satchel Paige

Page 41 of 180

Civil Law Q&As (2007-2013)

hectorchristopher@yahoo.com

dbaratbateladot@gmail.com

rights and interests in the Agro-Macro


Yes, the trial court was correct. At the
time the petitions for adoptions were
filed, petitioner had already remarried.
Under the law, husband and wife shall
adopt

jointly,

except

in

the

cases

enumerated in the law. The adoption


cases of Michelle and James do not fall
in any of the exceptions provided in the
law where a spouse is permitted to adopt
alone.

Hence,

Monina

should

adopt

jointly with her husband Angel (Adoption


of Michelle P. Lim, G.R. Nos. 168992-93,
May 21, 2009).

Property

Relations;

Unions

Without

Marriage (2012)
No.V. b) Jambrich, an Austrian, fell in-love
and lived together

with Descallar and

bought their houses and lots at Agro-Macro


Subdivision.

In

the

Contracts

to

Sell,

Jambrich and Descallar were referred to as


the buyers. When the Deed of Absolute Sale
was presented for registration before the
Register of Deeds, it was refused because
Jambrich was an alien and could not
acquire
domain.

alienable
After

lands

Jambrich

of

the

and

public

Descallar

separated, Jambrich purchased an engine


and some accessories for his boat from
Borromeo. To pay for his debt, he sold his

properties to Borromeo.

apply to their property relations and the


Borromeo discovered that titles to the three

properties in question are owned by

(3) lots have been transfereed in the name

them in equal shares even though all the

of Descallar. Who is the rightful owner of

funds used in acquiring the properties

the properties? Explain. (5%)

came only from the salaries or wages, or

SUGGESTED ANSWER:

the

income

of

Jambrich

from

his

business or profession. In such case,

It depends. On the assumption that the

while Jambrich is disqualified to own

Family Code is the applicable law, the

any

ownership of the properties depends on

subsequent transfer of all his interest

whether or not, Jambrich and Descallar

therein to Borromeo, a Filipino, was

are capacitated to marry each other

valid as it removed the disqualification.

during their cohabitation, and whether

In such case, the properties are owned

or not both have contributed funds for

by Borromeo and Descallar in equal

the acquisition of the properties.

shares.

If both of them are capacitated to marry

If, on the other hand, Jambrich and

each other, Art 147- co-ownership will

Descallar were not capacitated to marry

part

of

the

properties,

his

Never Let The Odds Keep You From Pursuing What You Know In Your Heart You Were Meant To Do.Leroy Satchel Paige

Page 42 of 180

Civil Law Q&As (2007-2013)

hectorchristopher@yahoo.com

dbaratbateladot@gmail.com

which bore no offspring, was declared void


each

other

Art.

148-co-ownership

governs their property relations. Under


this regime, Jambrich and Descallar are
co-owners of the properties but only if
both

of

them

contributed

in

their

acquisition. If all the funds used in


acquiring

the

properties

in

question

came from Jambrich, the entire property


is his even though he is disqualified from
owning it. His subsequent transfer to
Borromeo,

however,

is

valid

as

it

removed the disqualification. In such


case, all the properties are owned by
Borromeo.
Descallar

If,

on

the

other

hand

to

their

contributed

acquisition, the properties are co-owned


by Descallar and Borromeo in proportion
to the respective contributions of the
Descallar and Jambrich.
Note: The facts of the problem are not
exactly the same as in the case of
Borromeo v. Descallar, G.R. NO. 159310,
Feb 24, 2009, hence, the difference in
the resulting answer.

Property

Relations;

Void

Marriages

(2010)
No.VII. G and B were married on July 3,
1989. On March 4, 2001, the marriage,

ab initio under Article 36 of the Family


Code. At the time of the dissolution of the

SUGGESTED ANSWER:

marriage, the couple possessed the

Since the marriage was declared void ab

following properties:
a house and lot acquired by B on
August 3, 1988, one third (1/3) of
the

purchase

price

(representing

downpayment) of which he paid; one

initio in 2001, no Absolute Community


or

Conjugal

Partnership

was

ever

established between B and G. Their


property relation is governed by a

G on

special co-ownership under Article 147

February 14, 1990 out of a cash gift

of the Family Code because they were

given to her by her parents on her

capacitated to marry each other.

third

(1/3) was paid by

graduation on April 6, 1989; and


the balance was paid out of the
spouses joint income; and
an apartment unit donated to B by
an uncle on June 19, 1987.

Under

that

Article

147,

wages

and

salaries of the former spouses earned


during their cohabitation shall be owned
by them in equal shares while properties
acquired thru their work for industry

(A) Who owns the foregoing properties?


Explain. (5%)

shall be owned by them in proportion to

Never Let The Odds Keep You From Pursuing What You Know In Your Heart You Were Meant To Do.Leroy Satchel Paige

Page 43 of 180

Civil Law Q&As (2007-2013)

hectorchristopher@yahoo.com

dbaratbateladot@gmail.com

during the cohabitation, hence, it is his


their respective contributions. Care and
maintenance of the family is recognized
as

valuable

contribution.

In

the

absence of proof as to the value of their


respective

contributions,

they

shall

share equally.
If ownership of the house and lot was
acquired by B on August 3, 1988 at the
time he bought it on installment before
he got married, he shall remain owner of
the house and lot but he must reimburse
G for all the amounts she advanced to
pay the purchase price and for one-half
share in the last payment from their
joint income. In such case, the house
and lot were not acquired during their
cohabitation, hence, are not co-owned by
B and G.
But if the ownership of the house and lot
was acquired during the cohabitation,
the house and lot will be owned as
follows:
(1) 1/3 of the house and lot is owned by
B. He is an undivided co-owner to that
extent

for

his

contributions

in

its

acquisition in the form of the down


payment he made before the celebration
of the marriage. The money he used to
pay the down payment was not earned

exclusive property.

(2) 1/3 of the house and lot is owned by

G. She is an undivided co-owner to the


extent

for

her

contribution

in

its

acquisition when she paid 1/3 of the

After

summing

up

their

prospective

shares, B and G are undivided co-owners


of the house and lot in equal shares.

purchase price using the gift from her

As

parents. Although the gift was acquired

exclusive by B because he acquired it

by G during her cohabitation with B, it is

before their cohabitation. Even if he

her exclusive property. It did not consist

acquired it during their cohabitation, it

of wage or salary or fruit of her work or

will

industry.

because it did not come from his wage or

to

the

still

be

apartment,

his

it

exclusive

is

owned

property

salary, or from his work or industry. It


(3) 1/3 of the house is co-owned by B

was acquired gratuitously from his uncle.

and G because the payment came from


their co-owned funds, i.e., their joint

(B) If G and B had married on July 3, 1987

income during their cohabitation which

and their marriage was dissolved in 2007,

is shared by them equally in the absence


of any proof to the contrary.

who owns the properties? Explain. (5%)


SUGGESTED ANSWER:

Never Let The Odds Keep You From Pursuing What You Know In Your Heart You Were Meant To Do.Leroy Satchel Paige

Page 44 of 180

Civil Law Q&As (2007-2013)

hectorchristopher@yahoo.com

The answer is the same as in letter A.

dbaratbateladot@gmail.com

No.X. In 1997, B and G started living


together without the benefit of marriage.

Since the parties to the marriage which

The relationship produced one offspring,

was later declared void ab initio were

Venus. The couple acquired a residential lot

capacitated to marry each other, the


applicable law under the New Civil Code
was

Article

144.This

Article

is

substantially the same as Article 147 of


the Family Code.
Hence, the determination of ownership
will remain the same as in question A.
And

even

assuming

that

the

two

provisions are not the same, Article 147


of the Family Code is still the law that
will govern the property relations of B
and G because under Article 256, the
Family

Code

has

retroactive

effect

insofar as it does not prejudice or impair


vested or acquired rights under the new
Civil Code or other laws. Applying Article
147 retroactively to the case of G and B
will not impair any vested right. Until
the declaration of nullity of the marriage
under the Family Code, B and G have not
as yet acquired any vested right over the
properties

acquired

during

their

cohabitation.

Property
(2010)

Relations;

Void

Marriages

and G. Their properties are governed by


in Paraaque. After four (4) years or in

the special co-ownership provision of

2001,

4-year

Article 147 of the Family Code because

college degree as a fulltime student, she

both B and G were capacitated to marry

and

each other. The said Article provides

having

contracted

completed

marriage

her

without

license.

that when a man and a woman who are

The marriage of B and G was, two years


later, declared null and void due to the

capacitated to marry each other, live


exclusively with each other as husband
and wife without the benefit of marriage,

absence of a marriage license.

or under a void marriage: (1) their wages


(A). If you were the judge who declared the

and salaries shall be owned by them in

nullity of the marriage, to whom would you

equal shares; and (2) property acquired

award the lot? Explain briefly. (3%)

by both of them through their work or


industry shall be governed by the rules

SUGGESTED ANSWER:

on co-ownership. In co-ownership, the


Since the marriage was null and void, no
Absolute

Community

or

Conjugal

parties are co-owners if they contributed


something of value in the acquisition of

Partnership was established between B


Never Let The Odds Keep You From Pursuing What You Know In Your Heart You Were Meant To Do.Leroy Satchel Paige

Page 45 of 180

Civil Law Q&As (2007-2013)

the

property.

proportion

Their

to

contributions.

hectorchristopher@yahoo.com

share

their

In

an

is

respective
ordinary

co-

the family is not recognized as a valuable


contribution for the acquisition of a
property. In the Article 147 special cohowever,

care

and

maintenance is recognized as a valuable


contribution
contributor

which
to

half

will
of

entitle
the

the

property

acquired.
Having

Succession

in

ownership the care and maintenance of

ownership

dbaratbateladot@gmail.com

Disposition; Mortis Causa vs. Intervivos;


Corpse (2009)
No. XI. TRUE or FALSE. Answer TRUE if
the statement is true, or FALSE if the
statement is false. Explain your answer in
not more than two (2) sentences.
(E). A person can dispose of his corpse
through an act intervivos. (1%)
SUGGESTED ANSWER:

been

acquired

during

their

False. A persons cannot dispose of his

is

corpse through an act inter vivos, i.e.,

presumed acquired through their joint

an act to take effect during his lifetime.

work and industr under Article 147,

Before his death there is no corpse to

hence, B and G are co-owners of the said

dispose. But he is allowed to do so

property in equal shares.

through an act mortis causa, i.e., an act

cohabitation,

the

residential

lot

Article 147 also provides that when a

to take effect upon his death.

party to the void marriage was in bad


faith, he forfeits his share in the coownership

in

favor

of

the

common

children or descendants, the default of

Heirs;

Fideicommissary

Substitution

(2008)

children or descendants, the forfeited

No. XIII. Raymond, single, named his sister

share shall belong to the innocent party.

Ruffa in his will as a devisee of a parcel of

In the foregoing problem, there is no

land which he owned. The will imposed

showing that one party was in bad faith.

upon Ruffa the obligation of preseving the

Hence, both shall be presumed in good

land and transferring it, upon her death, to

faith and no forfeiture shall take place.

her illegitimate daughter Scarlet who was


then only one year old. Raymond later died,

leaving behind his widowed mother, Ruffa


and Scarlet.
Never Let The Odds Keep You From Pursuing What You Know In Your Heart You Were Meant To Do.Leroy Satchel Paige

Page 46 of 180

Civil Law Q&As (2007-2013)

hectorchristopher@yahoo.com

dbaratbateladot@gmail.com

illegitimate child, cannot inherit the


(A). Is the condition imposed upon Ruffa, to

property by intestate succession from

preserve the property and to transmit it

Raymond who is a legitimate relative of

upon her death to Scarlet, valid? (1%)


SUGGESTED ANSWER:
Yes, the condition imposed upon Ruffa
to preserve the property and to transmit
it upon her death to Scarlet is valid
because

it

is

tantamount

to

fideicommissary substitution under Art.


863 of the Civil Code.
(B).

If Scarlet

predeceases

Ruffa, who

inherits the property? (2%)


SUGGESTED ANSWER:
Ruffa

will

inherit

the

property

as

Scarlet's heir. Scarlet acquires a right to


the

succession

Raymond's

from

death,

the

even

time

though

of
she

should predecease Ruffa (Art. 866, Civil


Code).
(C). If Ruffa predeceases Raymond, can
Scarlet inherit the property directly from
Raymond? (2%)
SUGGESTED ANSWER:
If

Ruffa

Raymond's

predeceases
widowed

mother

Raymond,
will

be

entitled to the inheritance. Scarlet, an

each
Ruffa (Art. 992, Civil Code). Moreover,

of

their

children

as

their

presumptive legitimes.

Scarlet is not a compulsory heir of

Peter later re-married and had two (2)

Raymond, hence she can inherit only by

children by his second wife Marie. Peter and

testamentary

Marie,

succession.

Since

having

successfully

engaged

in

Raymond executed a will in the case at

business, acquired real properties. Peter

bar, Scarlet may inherit from Raymond.

later died intestate.


(A). Who are Peters legal heirs and how will
his estate be divided among them? (5%)

Heirs; Intestate

Succession; Legitime;

Computation (2010)

SUGGESTED ANSWER:

No.XI. The spouses Peter and Paula had

The legal heirs of Peter are his children

three (3) children. Paula later obtained a

by the first and second marriages and

judgment of nullity of marriage. Their

his surviving second wife.

absolute community of property having


been dissolved, they delivered P1 million to

Their shares in the estate of Peter will


depend, however, on the cause of the

Never Let The Odds Keep You From Pursuing What You Know In Your Heart You Were Meant To Do.Leroy Satchel Paige

Page 47 of 180

Civil Law Q&As (2007-2013)

hectorchristopher@yahoo.com

nullity of the first marriage. If the


nullity

of

the

first

marriage

was

psychological incapacity of one or both


spouses, the three children of that void
marriage are legitimate and all of the
legal heirs shall share the estate of Peter
in equal shares. If the judgment of
nullity was for other causes, the three
children are illegitimate and the estate
shall

be

distributed

such

that

an

illegitimate child of the first marriage


shall receive half of the share of a
legitimate child of the second marriage,
and the second wife will inherit a share
equal to that of a legitimate child. In no
case may the two legitimate children of
the second marriage receive a share less
than one-half of the estate which is their
legitime.

When

the

estate

is

not

sufficient to pay all the legitimes of the


compulsory heirs, the legitime of the
spouse is preferred and the illegitimate
children suffer the reduction.
Computation:
(A)

If

the

ground

of

nullity

is

psychological incapacity:
3 children
marriage

by first 1/6 of the estate


for each

2 children by second 1/6 of the estate


marriage
for each

Surviving
spouse

dbaratbateladot@gmail.com

second 1/6 of the estate

the
(B) If the

ground

of nullity is not

estate

is

left

after

paying

the

legitime of the surviving spouse which is

psychological capacity:

preferred.

2
legitimate of the estate for
children
each
of
second

Hence, the remaining of the estate


shall be divided among the 3 illegitimate

marriage
Surviving
second spouse

children.

of the estate

3
illegitimate 1/12 of
estate for
children
each of first marriage

(B). What is the effect of the receipt by


Peters 3 children by his first marriage of
their presumptive legitimes on their right to
inherit following Peters death? (5%)

Note: The legitime of an illegitimate


child is supposed to be the legitime of

SUGGESTED ANSWER:

a legitimate child or 1/8 of the estate.

In the distribution of Peters estate, of

But the estate will not be sufficient to

the

pay

children of the first marriage shall be

the

said

legitime

of

the

illegitimate children, because only of

presumptive

received

by

the

collated to Peters estate and shall be

Never Let The Odds Keep You From Pursuing What You Know In Your Heart You Were Meant To Do.Leroy Satchel Paige

Page 48 of 180

Civil Law Q&As (2007-2013)

hectorchristopher@yahoo.com

dbaratbateladot@gmail.com

heir of the decedent he seeks to inherit


imputed

as

an

advance

of

their

respective inheritance from Peter. Only


half

of

the

presumptive

legitime

is

collated to the estate of Peter because


the other half shall be collated to the
estate of his first wife.

Heirs; Representation; Iron-Curtain Rule


(2012)
No.VIII.a) Ricky and Arlene are married.
They begot Franco during their marriage.
Franco had an illicit relationship with
Audrey and out of which, they begot Arnel.
Frnaco

predeceased

Ricky,

Arlene

and

Arnel. Before Ricky died, he executed a will


which when submitted to probate was
opposed by Arnel on the ground that he
should be given the share of his father,
Franco. Is the opposition of Arnel correct?
Why? (5%)
SUGGESTED ANSWER:
No, his opposition is not correct. Arnel
cannot

inherit

from

Ricky

in

the

representation of his father Franco. In


representation, the representative must
not only be a legal heir of the person he
is representing, he must also be a legal

from.
While Arnel is a legal heir of Franco, he
is not a legal heir of Ricky because under

(B).In reservatroncal, all reservatarios (reser


Art 992 of the NCC, an illegitimate child
has no right to inherit ab intestato from
the legitimate children and relatives of
his

father

disqualified

or
to

mother.
inherit

Arnel
from

is

vees) inherit as a class and in equal shares


regardless of their proximity in degree to
the prepositus. (1%)
SUGGESTED ANSWER:

Ricky

FALSE. Not all the relatives within the

because Arnel is an illegitimate child of

third degree will inherit as reservatario ,

Franco and Ricky is a legitimate relative

and not all those who are entitled to

of Franco.

inherit will inherit in the equal shares .


The

applicable

laws

of

intestate

succession will determine who among


Heirs; Reserva Troncal (2009)

the relatives will inherit as reservatarios

No. I. TRUE or FALSE. Answer TRUE if the


statement

is

true,

or

FALSE

if

the

statement is false. Explain your answer in


not more than two (2) sentences.

and what shares they will tak, i.e., the


direct line excludes the collateral, the
descending

direct

line

excludes

the

ascending ,the nearer excludes the more


remote, the nephews and nieces exclude

Never Let The Odds Keep You From Pursuing What You Know In Your Heart You Were Meant To Do.Leroy Satchel Paige

Page 49 of 180

Civil Law Q&As (2007-2013)

hectorchristopher@yahoo.com

dbaratbateladot@gmail.com

blood, a half blood sibling will receive


the uncles and the aunts, and half blood
relatives inherit half the share of fullblooded relatives.

Intestate Succession (2008)


No. VII. Ramon Mayaman died intestate,
leaving a net estate of P10,000,000.00.
Determine how much each heir will receive
from the estate:
(A). If Ramon is survived by his wife, three
full-blood brothers, two half-brothers, and
one nephew (the son of a deceased fullblood brother)? Explain. (3%)
SUGGESTED ANSWER:
Having died intestate, the estate of
Ramon shall be inherited by his wife and
his full and half blood siblings or their
respective representatives. In intestacy,
if the wife concurs with no one but the
siblings of the husband, all of them are
the

intestate

heirs

of

the

deceased

husband. The wife will receive half of the


intestate estate, while the siblings or
their

respective

representatives,

will

inherit the other half to be divided


among them equally. If some siblings are
of the full-blood and the other of the half

half the share of a full-blood sibling.

The wife will receive one half (1/2) of the


(1). The wife of Ramon will, therefore,

estate or P5,000,000.00. The other half

receive one half () of the estate or the

shall be inherited by (1) the full-blood

amount of P5,000,000.00.

brother,

(2). The three (3) full-blood brothers, will,

children, and (2) the half-sister. They will

therefore, receive P1,000,000.00 each.

divide the other half between them such

(3).

receive

that the share of the half-sister is just

of

half the share of the full-blood brother.

The

nephew

P1,000,000.00

will

by

right

represented

by

his

three

representation.

The share of the full-blood brother shall

(4). The two (2) half-brothers will receive

in

P500,000.00 each.

nephews in equal shares by right of

turn

be

inherited

by

the

three

presentation.
(B). If Ramon is survived by his wife, a halfsister,

and three

nephews

(sons

of a

deceased full-blood brother)? Explain. (3%)


SUGGESTED ANSWER:

Therefore, the three (3) nephews will


receive P1,111,111.10 each the halfsister

will

receive

the

sum

of

P1,666,666.60.

Never Let The Odds Keep You From Pursuing What You Know In Your Heart You Were Meant To Do.Leroy Satchel Paige

Page 50 of 180

Civil Law Q&As (2007-2013)

hectorchristopher@yahoo.com

Intestate Succession (2008)

dbaratbateladot@gmail.com

(D). How should the house and lot, and the


cash be distributed? (1%)

No.X. Arthur executed a will which contained


only: (i) a provision disinheriting his daughter

SUGGESTED ANSWER:

Bernica for running off with a married man,


and (ii) a provision disposing of his share in

Since the probate of the will cannot be

the family house and lot in favor of his other

allowed,

children Connie and Dora. He did not make

succession apply. Under Art. 996 of the

any provisions in favor of his wife Erica,

Civil Code, if a widow or widower and

because as the will stated, she would anyway

legitimate children or descendants are

get of the house and lot as her conjugal

left, the surviving spouse has the same

share.

The

will

was

very

brief

and

straightforward and both the above provisions


were contained in page 1, which Arthur and
his

instrumental

witness,

signed

at

the

bottom. Page 2 contained the attestation


clause and the signatures, at the bottom
thereof, of the 3 instrumental witnesses

the

rules

on

intestate

share as of the children. Thus, ownership


over the house and lot will be created
among

wife

Erica

and

her

children

Bernice, Connie and Dora. Similarly, the


amount of P 1 million will be equally
divided among them.

which included Lambert, the driver of Arthur;


Yoly, the family cook, and Attorney Zorba, the
lawyer who prepared the will. There was a 3rd

Intestate

page, but this only contained the notarial

Representation:

acknowledgement.

Child; Iron Curtain Rule (2007)

The

attestation

clause

Succession;

Rights

Illegitimate,

of

Adopted

stated the will was signed on the same


occasion by Arthur and his instrumental

No. X. For purpose of this question, assume

witnesses who all signed in the presence of

all formalities and procedural requirements

each

have been complied with.

other,

and the

notary

public

who

notarized the will. There are no marginal


signatures or pagination appearing on any of
the 3 pages. Upon his death, it was discovered
that apart from the house and lot, he had a P
1 million account deposited with ABC bank.

In 1970, Ramon and Dessa got married.


Prior to their marriage, Ramon had a child,
Anna. In 1971 and 1972, Ramon and Dessa
legally

adopted

Cherry

and

Michelle

respectively. In 1973, Dessa died while


giving birth to Larry Anna had a child, Lia.

Anna never married. Cherry, on the other


hand, legally adopted Shelly. Larry had
Never Let The Odds Keep You From Pursuing What You Know In Your Heart You Were Meant To Do.Leroy Satchel Paige

Page 51 of 180

Civil Law Q&As (2007-2013)

hectorchristopher@yahoo.com

adopted.

The

dbaratbateladot@gmail.com

law

on

representation

twins, Hans and Gretel, with his girlfriend,

requires the representative to be a legal

Fiona. In 2005, Anna, Larry and Cherry

heir of the person he is representing and

died in a car accident. In 2007, Ramon


died. Who may inherit from Ramon and
who may not? Give your reason briefly.
(10%)
SUGGESTED ANSWER:
The following may inherit from Ramon:
(1). Michelle, as an adopted child of
Ramon, will inherit as a legitimate child
of Ramon. As an adopted child, Michelle
has all the rights of a legitimate child
(Sec 18, Domestic Adoption Law).
(2). Lia will inherit in representation of
Anna. Although Lia is an illegitimate
child, she is not barred by Articles 992,
because

her

mother

Anna

is

an

illegitimate herself. She will represent


Anna as regards Anna's legitime under
Art. 902, NCC and as regards Anna's
intestate share under Art. 990, NCC.
The following may not inherit from
Ramon:
(1). Shelly, being an adopted child, she
cannot represent Cherry. This is because
adoption creates a personal legal relation
only

between

the

adopter

and

the

time, adoption was governed by the New


also of the person from whom the person

Civil Code. Under the New Civil Code,

being

to

husband and wife were allowed to adopt

inherit. While Shelly is a legal heir of

separately or not jointly with the other

Cherry, Shelly is not a legal heir of

spouse. And since the problem does not

Ramon.

Adoption

purely

specifically and categorically state, it is

personal

legal

between

possible to construe the use of the word

represented

was

supposed

created

relation

only

Cherry and Shelly.

"respectively"

in

the

problem

as

indicative of the situation that Cherry


(2). Hans and Gretel are barred from

was

inheriting from Ramon under Art. 992,

Michelle was adopted by Dessa alone. In

NCC. Being illegitimate children, they

such

cannot inherit ab intestao from Ramon.

alternative answer to the problem will be

ALTERNATIVE ANSWER:

adopted

case

of

by

Ramon

separate

alone

adoption

and

the

as follows: Only Lia will inherit from


Ramon in representation of Ramon's

The problem expressly mentioned the

illegitimate daughter Anna. Although Lia

dates of the adoption of Cherry and

is an illegitimate child, she is not barred

Michelle as 1971 and 1972. During that

from inheriting from Ramon because her

Never Let The Odds Keep You From Pursuing What You Know In Your Heart You Were Meant To Do.Leroy Satchel Paige

Page 52 of 180

Civil Law Q&As (2007-2013)

hectorchristopher@yahoo.com

dbaratbateladot@gmail.com

No.VIII.b) How can RJP distribute his estate


mother is herself illegitimate. Shelly

by will, if his heirs are JCP, his wife; HBR

cannot

and RVC, his parents; and an illegitimate

inherit

in

representation

of

Cherry because Shelly is just an adopted


child of Cherry. In representation, the
representative must not only be a legal
heir of the person he is representing but
also of the decedent from whom the
represented

person

is

supposed

to

inherit. In the case of Shelly, while she


is a legal heir of Cherry by virtue of
adoption, she is not a legal heir of
Ramon.

Adoption

creates

personal

legal relation only between the adopting


parent and the adopted child (Teotico v.
Del Val, 13 SCRA 406, 1965. Michelle
cannot inherit from Ramon, because she
was adopted not by Ramon but by Dessa.
In the eyes of the law, she is not related
to Ramon at all. Hence, she is not a legal
heir of Ramon. Hans and Gretel are not
entitled to inherit from Ramon, because
they are barred by Art. 992 NCC. Being
illegitimate
cannot

children

inherit

from

of

Larry,

the

they

legitimate

relatives of their father Larry. Ramon is


a legitimate relative of Larry who is the
legitimate father.

Legitimes; Compulsory Heirs (2012)

child, SGO?

Middle East. He had saved P100.000 in his


saving account in Manila which intended to

SUGGESTED ANSWER:

use to start a business in his home

A testator may dispose of by will the free

country. On his flight home, Ernesto had a

portion of his estate. Since the legitime

fatal heart attack. He left behind his

of JCP is 1/8 of the estate, SGO is of

widowed mother, his common-law wife and

the estate and that of HBR and RVC is

their twins sons. He left no will, no debts,

of the hereditary estate under Art 889 of

no other relatives and no other properties

the NCC, the remaining 1/8 of the estate


is the free portion which the testator
may dispose of by will.

except the money in his saving account.


Who are the heirs entitled to inherint from
him and how much should each receive?
(3%)
SUGGESTED ANSWER:

Legitime; Compulsory Heirs (2008)


No.

XII.

Ernesto,

an

overseas

Filipino

worker, was coming home to the Philippines

The mother and twin sons are entitled to


inherit from Ernesto. Art. 991 of the
Civil Code, provides that if legitimate

after working for so many years in the


Never Let The Odds Keep You From Pursuing What You Know In Your Heart You Were Meant To Do.Leroy Satchel Paige

Page 53 of 180

Civil Law Q&As (2007-2013)

hectorchristopher@yahoo.com

dbaratbateladot@gmail.com

lawyer who prepared the will. There was a 3rd

ascendants are left, the twin sons shall


divide the inheritance with them taking
one-half of the estate. Thus, the widowed
mother gets P50,000.00 while the twin
sons shall receive P25,000.00 each. The
common-law wife cannot inherit from
him because when the law speaks "widow
or widower" as a compulsory heir, the
law refers to a legitimate spouse (Art.
887, par 3, Civil Code).

Preterition; Disinheritance (2008)


No.X. Arthur executed a will which contained
only: (i) a provision disinheriting his daughter
Bernica for running off with a married man,
and (ii) a provision disposing of his share in
the family house and lot in favor of his other
children Connie and Dora. He did not make
any provisions in favor of his wife Erica,
because as the will stated, she would anyway
get of the house and lot as her conjugal
share.

The

will

was

very

brief

and

straightforward and both the above provisions


were contained in page 1, which Arthur and
his

instrumental

witness,

signed

at

the

bottom. Page 2 contained the attestation


clause and the signatures, at the bottom
thereof, of the 3 instrumental witnesses
which included Lambert, the driver of Arthur;
Yoly, the family cook, and Attorney Zorba, the

page, but this only contained the notarial


acknowledgement.

The attestation clause stated the will was


signed on the same occasion by Arthur and
his instrumental witnesses who all signed

(B). What other defects of the will, if any,


can cause denial of probate? (2%)
SUGGESTED ANSWER:

in the presence of each other, and the


notary public who notarized the will. There

The other defects of the will that can

are no marginal signatures or pagination

cause its denial are as follows: (a) Atty.

appearing on any of the 3 pages. Upon his

Zorba, the one who prepared the will was

death, it was discovered that apart from the

one of the three witnesses, violating the

house and lot, he had a P 1 million account

three-witnesses rule; (b) no marginal

deposited with ABC bank.

signature at the last page; (c ) the


attestation did not state the number of

(A). Was Erica preterited? (1%)

pages upon which the will is written;


SUGGESTED ANSWER:

and,

Erica cannot be preterited. Art. 854 of


the

Civil

Code

provides

that

only

(d)

no

pagination

appearing

correlatively in letters on the upper part


of the three pages (Azuela v. C.A., G.R.

compulsory heirs in the direct line can


be preterited.
Never Let The Odds Keep You From Pursuing What You Know In Your Heart You Were Meant To Do.Leroy Satchel Paige

Page 54 of 180

Civil Law Q&As (2007-2013)

hectorchristopher@yahoo.com

dbaratbateladot@gmail.com

No. 122880, 12 Apr 2006 and cited cases

(B). Between Marian and the baby, who is

therein, Art 805 and 806, Civil Code).

presumed to have died ahead? (1%)

(C). Was the disinheritance valid? (1%)

SUGGESTED ANSWER:

SUGGESTED ANSWER:

Marian is presumed to have died ahead


of the baby. Art. 43 applies to persons

Yes, the disinheritance was valid. Art.


919, par 7, Civil Code provides that
"when a child or descendant leads a
dishonorable

or

disgraceful

life,

like

running off with a married man, there is


sufficient cause for disinheritance."

who are called to succeed each other.


The proof of death must be established
by positive or circumstantial evidence
derived from facts. It can never be
established from mere inference. In the
present case, it is very clear that only
Marian and Pietro were hacked with
bolos. There was no showing that the

Succession;

Proof

of

Death

between

baby was also hacked to death. The

persons called to succeed each other

baby's death could have been due to lack

(2008)

of nutrition.

No. II. At age 18, Marian found out that she

ALTERNATIVE ANSWER:

was pregnant. She insured her own life and


as her sole

The baby is presumed to have died ahead

beneficiary. When she was already due to

of Marian. Under Par. 5, rule 131, Sec. 5

give birth, she and her boyfriend Pietro, the

(KK) of the Rules of Court, if one is

father

under 15 or above 60 and the age of the

named her unborn

of

her

child

unboarn

child,

were

kidnapped in a resort in Bataan where they


were vacationing. The military gave chase
and after one week, they were found in an
abandoned hut in Cavite. Marian and Pietro
were hacked with bolos. Marian and the
baby delivered were both found dead, with

other is in between 15 and 60, the latter


is presumed to have survived. In the
instant case, Marian was already 18
when

she

found

out

that

she

was

pregnant. She could be of the same age

the baby's umbilical cord already cut. Pietro

or maybe 19 years of age when she gave

survived.

birth.

(C). Will Pietro, as surviving biological


father of the baby, be entitled to claim the

Never Let The Odds Keep You From Pursuing What You Know In Your Heart You Were Meant To Do.Leroy Satchel Paige

Page 55 of 180

Civil Law Q&As (2007-2013)

hectorchristopher@yahoo.com

proceeds of the life insurance on the life of


Marian? (2%)
SUGGESTED ANSWER:
Pietro, as the biological father of the
baby, shall be entitled to claim the
proceeds of life insurance of the Marian
because he is a compulsory heir of his
child.

Succession; Rule on Survivorship (2009)


No. II. Dr. Lopez, a 70-year old widower, and
his son Roberto both died in a fire that
gutted their home while they were sleeping
in their air-conditioned rooms. Robertos
wife, Marilyn, and their two children were
spared because they were in the province at
the time. Dr. Lopez left an estate worth
P20M and a life insurance policy in the
amount of P1M with his three children --one of whom is Roberto --- as beneficiaries.
Marilyn is now claiming for herself and her
children her husbands share in the estate
left by Dr. Lopez, and her husbands share
in the proceeds of Dr. Lopezs life insurance
policy. Rule on the validity of Marilyns
claims with reasons. (4%)
SUGGESTED ANSWER :

dbaratbateladot@gmail.com

As to the Estate of Dr. Lopez:

three children of Dr. Lopez . Marilyn


cannot represent her husband Roberto
Marilyn is not entitled to a share in the

because the right is not given by the law

estate of Dr. Lopez. For purpose of

to a surviving spouse.

succession,

Dr.

Lopez

and

his

son

Roberto are presumed to have died at

As to the proceeds of the insurance on

the same time, there being no evidence

the life of Dr. Lopez:

to prove otherwise, and there shall be no


transmission of rights from one to the
other (Article 43, NCC). Hence, Roberto,
inherited nothing from his father that
Marilyn

would

Roberto

.The

in

turn

children

inherit
of

from

Roberto,

however, will succeed their grandfather,


Dr. Lopez ,in representation of their
father Roberto and together Roberto will
receive 1/3 of the estate of Dr. Lopez
since their father Roberto was one of the

Since
regards

succession is
the

not

insurance

involved as

contract,

the

provisions of the Rules of Court (Rule


131, Sec. 3 , [jj] [5] ) on survivorship
shall apply. Under the Rules, Dr. Lopez,
who was 70 years old, is presumed to
have died ahead of Roberto who is
presumably between the ages 15 and 60.
Having survived the insured, Roberto's
right as a beneficiary became vested

Never Let The Odds Keep You From Pursuing What You Know In Your Heart You Were Meant To Do.Leroy Satchel Paige

Page 56 of 180

Civil Law Q&As (2007-2013)

hectorchristopher@yahoo.com

dbaratbateladot@gmail.com

testator himself, Rosas claimed that the


upon the death of Dr. Lopez. When
Roberto died after Dr. Lopez, his right to
receive the insurance became part of his
hereditary estate, which in turn was
inherited in equal shares by his legal
heirs, namely, his spouse and children.
Therefore, Roberto's children and his
spouse are entitled to Roberto's onethird share in the insurance proceeds.

Wills; Holographic Wills; Insertions &


Cancellations (2012)
No.VII.a) Natividads holographic will, which
had only one (1) substantial provision, as
first written, named Rosa as her sole heir.
However, when Gregorio presented it for
probate, it already contained an alteration,
naming Gregorio, instead of Rosa, as sole
heir, but without authentication by
Natividads signature. Rosa opposes the
probate

alleging

authentication.

such
She

lack

claims

of

proper

that

the

unaltered form of the will should be given


effect. Whose claim should be granted?
Explain. (5%)
SUGGESTED ANSWER:
It depends. If the cancellation of Rosas
name in the will was done by the

holographic will in its original tenor

The writing of Gregorios name as sole


should be given effect must be denied.

heir was ineffective, even though written

The said cancellation has revoked the

by the testator himself, because such is

entire will as nothing remains of the will

an

after the name of Rosa was cancelled.

authentication by the full signature of

Such cancellation is valid revocation of

the testator to be valid and effective. Not

the

require

having an authenticated, the designation

authentication by the full signature of

of Gregorio as an heir was ineffective,

the testator to be effective.

(Kalaw v. Relova, G.R. No. L-40207, Sept

will

and

does

not

alteration

that

requires

28, 1984).
However, if the cancellation of Rosas
name was not done by the testator
himself, such cancellation shall not be
effective and the will in its original tenor

Wills; Holographic Wills; Probate (2009)

shall remain valid. The effectively of the

No.VI. On December 1, 2000, Dr. Juanito

holographic will cannot be left to the

Fuentes

mercy of unscrupulous third parties.

wherein he gave nothing to his recognized

executed

holographic

will,

illegitimate son, Jay. Dr. Fuentes left for the

Never Let The Odds Keep You From Pursuing What You Know In Your Heart You Were Meant To Do.Leroy Satchel Paige

Page 57 of 180

Civil Law Q&As (2007-2013)

hectorchristopher@yahoo.com

dbaratbateladot@gmail.com

Civil Code (Art. 816, NCC). Since Dr.


United States, passed the New York medical

Fuentes executed his will in accordance

licensure examinations, resided therein,

with the Philippine law, the Philippine

and became a naturalized American citizen.


He died in New York in 2007. The laws of
New York do not recognize holographic wills
or compulsory heirs.
(A). Can the holographic will of Dr. Fuentes
be admitted to probate in the Philippines?
Why or why not? (3%)
SUGGESTED ANSWER:
Yes, the holographic will of Dr. Fuentes
may be admitted to probate in the
Philippines because there is no public
policy violated by such probate. The only
issue at probate is the due execution of
the

will

which

includes

the

formal

validity of the will. As regards formal


validity, the only issue the court will
resolve at probate is whether or not the
will was executed in accordance with the
form prescribed by the law observed by
the testator in the execution of his will.
For

purposes

Philippines,

of

an

probate

alien

in

the

testator

may

observe the law of the place where the


will was executed (Art 17, NCC), or the
formalities of the law of the place where
he

resides,

or

according

to

the

formalities of the law of his own country,


or in accordance with the Philippine

No, Jay cannot insist because under New


court shall apply the New Civil Code in

York law he is not a compulsory heir

determining the formal validity of the

entitled to a legitime.

holographic will. The subsequent change


in the citizenship of Dr. Fuentes did not
affect the law governing the validity of
his will. Under the new Civil Code, which
was the law used by Dr. Fuentes, the law
enforced at the time of execution of the
will shall govern the formal validity of
the will (Art. 795, NCC).

The

national

law

of

the

testator

determines who his heirs are, the order


that

they

succeed, how

much

their

successional rights are, and whether or


not a testamentary disposition in his will
is valid (Art 16, NCC). Since, Dr. Fuentes
was a US citizen, the laws of the New
York determines who his heirs are. And

(B). Assuming that the will is probated in

since

the Philippines, can Jay validly insist that

recognize the concept of compulsory

he be given his legitime? Why or why not?

heirs, Jay is not a compulsory heir of Dr.

(3%)

the

New

York

law

does

not

Fuentes entitled to a legitime.

SUGGESTED ANSWER:

Never Let The Odds Keep You From Pursuing What You Know In Your Heart You Were Meant To Do.Leroy Satchel Paige

Page 58 of 180

Civil Law Q&As (2007-2013)

hectorchristopher@yahoo.com

(B).

valid? (2%)

Wills; Joint Wills (2008)


No. XI. John and Paula, British citizens at
birth, acquired Philippine citizenship by
naturalization after their marriage. During
their

marriage

the

couple

acquired

substanial landholdings in London and in


Makati. Paula bore John three children,
Peter, Paul and Mary. In one of their trips to
London, the couple executed a joint will
appointing each other as their heirs and
providing that upon the death of the
survivor between them the entire estate
would go to Peter and Paul only but the two
could not dispose of nor divide the London
estate as long as they live. John and Paul
died

tragically

in

the

London

Subway

terrorist attack in 2005. Peter and Paul filed


a petition for probate of their parent's will
before a Makati Regional Trial Court.
(A). Should the will be admitted to probate?
(2%)
SUGGESTED ANSWER:
No. The will cannot be admitted to
probate because a joint will is expressly
prohibited under Art. 818 of the Civil
Code. This provision applies John and
Paula

became

their marriage.

Are

Filipino

citizens

after

the

dbaratbateladot@gmail.com

testamentary

dispositions

No.VII.b) John Sagun and Maria Carla


Camua, British citizens at birth, acquired

SUGGESTED ANSWER:

Philippine

citizenship

by

naturalization

No. The testamentary dispositions are

after their marriage. During their marriage,

not valid because (a) omission of Mary, a

the

legitimate

landholdings in London and in Makati.

preterition

child,

is

which

tantamount

shall

annul

to
the

institution of Peter and Paul as heirs


(Art.

854,

Civil

Code);

and,

(b)

the

disposition that Peter and Paul could not


dispose of nor divide the London estate

Maria

couple

begot

acquired

three

(3)

substantial

children,

Jorge,

Luisito, and Joshur. In one of their trips to


London, the couple executed a joint will
appointing each other as their heirs and
providing that upon the death of the
survivor between them, the entire estate

for more than 20 years is void (Art. 870,

would go to Jorge and Luisito only but the

Civil Code).

two (2) could not dispose of nor divide the


London estate as long as they live. John
and Maria died tragically in the London

Wills; Joint Wills; Probate (2012)

subway terrorist attack in 2005. Jorge and


Luisito filed a petition for probate of their
parents will before a Makati Regional Trial

Never Let The Odds Keep You From Pursuing What You Know In Your Heart You Were Meant To Do.Leroy Satchel Paige

Page 59 of 180

Civil Law Q&As (2007-2013)

hectorchristopher@yahoo.com

Court. Joshur vehemently objected because


he was preterited.

Explain. (2%)
SUGGESTED ANSWER:
No, the will should not be admitted to
probate. Since the couples are both
Filipino citizens, Art 818 and 819 of the
NCC shall apply. Said articles prohibits
the execution of joint wills and make
them void, even though authorized of
the country where they were executed.
(2) Are the testamentary dispositions valid?
Explain. (2%)
SUGGESTED ANSWER:
Since the joint will is void, all the
testamentary disposition written therein
are also void. However, if the will is
valid, the institutions of the heirs shall
be

annulled

because

Joshur

was

preterited. He was preterited because he


will receive nothing from the will, will
receive nothing in testacy, and the facts
do not show that he received anything as
an advance on his inheritance. He was
totally excluded from the inheritance of
his parents.

(3) Is the testamentary prohibition against


the division of the London estate valid?
Explain. (1%)

(1) Should the will be admitted to probate?

dbaratbateladot@gmail.com

cannot be partitioned for as long as the


SUGGESTED ANSWER:
Assuming the will of John and Maria was

youngest of his four children desires to stay


there. As coheirs and co-owners, the other
three may demand partition anytime. (1%)

valid, the testamentary prohibition on the


division of the London estate shall be valid
but only for 20 years. Under Arts 1083 and
494 of the NCC, a testamentary disposition
of the testator cannot forbid the partition of
all or part of the estate for a period longer
than twenty (20) years.

SUGGESTED ANSWER:
FALSE, The other three co heirs may
not anytime demand the partition of the
house and lot since it was expressly
provided by the decedent in his will that
the same cannot be partitioned while his
youngest child desires to stay there.

Wills; Prohibition to Partition of a CoOwned Property (2010)

Article 1083 of the New Civil Code allows


a decedent to prohibit, by will, the
partition of a property and his estate for

No.I. True or False.

a period not longer than 20 years no

(B) X, a widower, died leaving a will stating


that the house and lot where he lived
Never Let The Odds Keep You From Pursuing What You Know In Your Heart You Were Meant To Do.Leroy Satchel Paige

Page 60 of 180

Civil Law Q&As (2007-2013)

hectorchristopher@yahoo.com

dbaratbateladot@gmail.com

of Arts. 804, 805 and 806 of the Civil


matter what his reason maybe. Hence,
the three co-heir cannot demand its
partition at anytime but only after 20
years from the death of their father.
Even if the deceased parent did not leave
a will, if the house and lot constituted
their family home, Article 159 of the
Family Code prohibits its partition for a
period of ten (10) years, or for as long as
there is a minor beneficiary living in the
family home.

Wills; Notarial

Wills;

Blind

Testator;

Requisites (2008)
No. XIV. Stevie was born blind. He went to
school for the blind, and learned to read in
Baille

Language.

He

Speaks

English

fluently. Can he:


(A). Make a will? (1%)
SUGGESTED ANSWER:
Assuming that he is of legal age (Art.
797, Civil Code) and of sound mind at
the time of execution of the will (Art.
798, Civil Code), Stevie, a blind person,
can make a notarial will, subject to
compliance with the "two-reading rule"
(Art. 808, Civil Code) and the provisions

Code.

subscribing witnesses, and again, by the


(B). Act as a witness to a will? (1%)

notary public before whom the will is


acknowledged (Art. 808, Civil Code).

SUGGESTED ANSWER:
Stevie cannot be a witness to a will. Art.
820 of the Civil Code provides that "any

Wills; Testamentary Disposition; Period

person of sound mind and of the age of

to Prohibit Partition (2008)

eighteen years or more, and not blind,


deaf or dumb, and able to read and write,

No. XI. John and Paula, British citizens at


birth, acquired Philippine citizenship by

may be a witness to the execution of a

naturalization after their marriage. During

will.

their

(C). In either of the above instances, must


the will be read to him? (1%)
SUGGESTED ANSWER:

marriage

the

couple

acquired

substanial landholdings in London and in


Makati. Paula bore John three children,
Peter, Paul and Mary. In one of their trips to
London, the couple executed a joint will
appointing each other as their heirs and

If Stevie makes a will, the will must be


read to him twice, once by one of the
Never Let The Odds Keep You From Pursuing What You Know In Your Heart You Were Meant To Do.Leroy Satchel Paige

Page 61 of 180

Civil Law Q&As (2007-2013)

hectorchristopher@yahoo.com

dbaratbateladot@gmail.com

Coming from the hospital, Clara insisted on


providing that upon the death of the
survivor between them the entire estate
would go to Peter and Paul only but the two
could not dispose of nor divide the London
estate as long as they live. John and Paul
died

tragically

in

the

London

Subway

terrorist attack in 2005. Peter and Paul filed


a petition for probate of their parent's will
before a Makati Regional Trial Court.
(C). Is the testamentary prohibition against
the division of the London estate valid? (2%)
SUGGESTED ANSWER:
No. the testamentary prohibition against
the division of the London estate is void
(Art.

870,

Civil

Code).

testator,

however, may prohibit partition for a


period which shall not exceed twenty
(20) years (Art. 870 in relation to Art.
494, par 3, Civil Code).

Wills; Witnesses to a Will, Presence


required;

Thumbmark

as

Signature

(2007)
No.VI. Clara, thinking of her mortality,
drafted a will and asked Roberta, Hannah,
Luisa

and

Benjamin

to

be

witnesses.

During the day of signing of her will, Clara


fell down the stairs and broke her arms.

signing her will by thumb mark and said

Probate

should

be

denied.

The

that she can sign her full name later. While

requirement that the testator and at

the

Roberta

least three (3) witnesses must sign all in

experienced a stomach ache and kept going

the "presence" of one another was not

to the restroom for long periods of time.

complied with. Benjamin who notarized

Hannah, while waiting for her turn to sign

the will is disqualified as a witness,

will

was

being

signed,

the will, was reading the 7

th

Harry Potter

hence he cannot be counted as one of

book on the couch, beside the table on

the three witnesses (Cruz v. Villasor, 54

which everyone was signing. Benjamin,


aside from witnessing the will, also offered
to notarize it. A week after, Clara was run
over by a drunk driver while crossing the
street in Greenbelt.

SCRA 31, 1973). The testatrix and the


other witnesses signed the will not in
the presence of Roberta because she was
in the restroom for extended periods of
time. Inside the restroom, Roberta could

May the will of Clara be admitted to

not have possibly seen the testatrix and

probate? Give your reasons briefly. (10%)

the other witnesses sign the will by

SUGGESTED ANSWER:

merely casting her eyes in the proper


direction (Jaboneta v. Gustilo, 5 Phil
541, 1906; Nera v. Rimando, 18 Phil

Never Let The Odds Keep You From Pursuing What You Know In Your Heart You Were Meant To Do.Leroy Satchel Paige

Page 62 of 180

Civil Law Q&As (2007-2013)

hectorchristopher@yahoo.com

dbaratbateladot@gmail.com

Jennifer acquired it and placed it in his


451,

1914).

Therefore,

the

testatrix

signed the will in the presence of only


two witnesses, and only two witnesses
signed the will in the presence of the
testatrix and of one another.
It is to be noted, however, that the
thumb mark intended by the testator to
be his signature in executing his last will
and

testament

is

valid

(Payad

v.

Tolentino, 62 Phil 848, 1936; Matias v.


Salud, L-104 Phil 1046, 23 June, 1958).
The problem, however, states that Clara
"said that she can sign her full name
later;" Hence, she did not consider her
thumb mark as her "complete" signature,
and intended further action on her part.
The testatrix and the other witness
signed

the

will

in

the

presence

of

Hannah, because she was aware of her


function and role as witness and was in a
position to see the testatrix and the
other witnesses sign by merely casting
her eyes in the proper direction.

Donation
Donations; Formalities; In Writing (2007)
No. VIII. In 1986, Jennifer and Brad were
madly in love. In 1989, because a certain
Picasso painting reminded Brad of her,

bedroom. In 1990, Brad and Jennifer broke


up. While Brad was mending his broken
heart, he met Angie and fell in love.

cannot validly bequeath the same to


Because the Picasso painting reminded

Angie (Art. 930, NCC). Even assuming

Angie of him, Brad in his will bequeathed

that the painting was impliedly given or

the painting to Angie. Brad died in 1995.

donated

Saddened by Brad's death, Jennifer asked

donation is nevertheless void for not

for the Picasso painting as a remembrance

being in writing. The Picasso painting

of him. Angie refused and claimed that

must be worth more than 5,000 pesos.

Brad, in his will, bequeathed the painting


to her. Is Angie correct? Why or why not?
(10%)
SUGGESTED ANSWER:

by

Jennifer

to

Brad,

the

Under Art. 748, NCC, the donation and


acceptance of a movable worth more
than 5,000 pesos must be in writing,
otherwise the donation is void. The
donation being void, Jennifer remained

NO. Angie is not correct. The Picasso


painting is not given or donated by
Jennifer to Brad. She merely "placed it

the owner of the Picasso painting and


Brad could not have validly disposed of
said painting in favor of Angie in his will.

in his bedroom." Hence, she is still the


owner of the painting. Not being the

ALTERNATIVE ANSWER:

owner of the Picasso painting, Brad


Never Let The Odds Keep You From Pursuing What You Know In Your Heart You Were Meant To Do.Leroy Satchel Paige

Page 63 of 180

Civil Law Q&As (2007-2013)

hectorchristopher@yahoo.com

dbaratbateladot@gmail.com

written. Such conditions, shall therefore,


YES. Angie is correct. Even assuming

be disregarded but the donation remains

that there was void donation because the

valid (Art. 727, NCC). On the other hand,

same was not in writing, Brad was in


uninterrupted possession of the Picasso
painting from 1989 to 1995, lasting for
six (6) years prior to his death. Brad has
already

acquired

ownership

painting

through

prescription.

Under

ownership

of

of

the

acquisitive

Art.

movables

1132,

NCC,

prescribes

through continuous possession for four


(4) years in good faith and for eight (8)
years without need of other conditions.
A void donation may be the basis of
possession in the concept of owner and
of just title for purposes of acquisitive
prescription.

Donations;

Illegal

&

Impossible

Conditions (2007)
No.I. Distinguish the following concepts:
(B). Illegal and impossible conditions in a
simple donation v. illegal and impossible
conditions in an onerous donation. (5%)
SUGGESTED ANSWER:
Illegal and impossible conditions in a
simple donation are considered as not

given

to

me,

hereby

freely,

voluntarily and irrevocably donate to

illegal and impossible donations imposed

her my one-hectare rice land covered

in an onerous donation shall annul the

by TCT No. 11550, located in San

donation (Art. 1183, NCC). This is so,

Fernando, Pampanga. This donation

because onerous donations are governed

shall take effect upon my death."

by the law on contracts (Art. 733, NCC).

The deed also contained Jennifer's signed


acceptance, and an attached notarized
Donation; Inter Vivos (2013)

declaration by Josefa and Jennifer that the


land will remain in Josefa's possession and

No.V. Josefa executed a deed of donation

cannot be alienated, encumbered, sold or

covering a one-hectare rice land in favor of

disposed of while Josefa is still alive.

her daughter, Jennifer. The deed specifically


provides that:

Advise Jennifer on whether the deed is a


donation inter vivos or mortis causa and

"For and in consideration of her love

explain the reasons supporting your advice.

and service Jennifer has shown and

(8%)

Never Let The Odds Keep You From Pursuing What You Know In Your Heart You Were Meant To Do.Leroy Satchel Paige

Page 64 of 180

Civil Law Q&As (2007-2013)

hectorchristopher@yahoo.com

dbaratbateladot@gmail.com

donation which state that the same will


only take effect upon the death of the

SUGGESTED ANSWER:

donor and that there is a prohibition to


The donation is a donation inter vivos.
When

the

donor

intends

that

alienate, encumber, dispose, or sell the

the

donation shall take effect during the


lifetime

of

the

donor,

though

the

property shall not be delivered till after


the

donors

death,

this

shall

be

donation inter vivos (Art. 729, Civil


Code).
The

Civil

Code

transmissions.

prefers

Moreover,

inter

vivos

mortis

causa

donations should follow the formalities of a


will (Art. 728, Civil Code). Here there is no
showing

that

such

formalities

were

followed. Thus, it is favorable to Jennifer


that the deed is a donation inter vivos.

Furthermore, what is most significant in


determining the type of donation is the
absence of stipulation that the donor
could

revoke

the

donation;

on

the

contrary, the deeds expressly declare


them

to

be

irrevocable,

quality

absolutely incompatible with the idea of


conveyances

mortis

causa

where

revocability is the essence of the act, to


the

extent

that

testator

cannot

lawfully waive or restrict his right of


revocation. The provisions of the deed of

As the donation is in the nature of a


same should be harmonized with its

mortis causa disposition, the formalities

express irrevocability (Austria-Magat v.

of a will should have been complied with

CA, G.R. No. 106755, Feb 1, 2002).

under

Art.

728

of

the

Civil

Code,

otherwise, the donation is void and


ALTERNATIVE ANSWER:

would produce no effect (The National

The donation is donation mortis causa.


The deed clearly states that the donation
shall take effect upon the death of the
donor,

Josefa.

retained

The

ownership

donor,
of

moreover,

the

subject

property as it was declared that the


property

cannot

be

Treasure of the Philippines v. Vda. de


Meimban, G.R. No. L-61023, Aug 22,
1984).

Property
Accretion; Alluvium (2008)

alienated,

No. IX. The properties of Jessica and Jenny,

encumbered, sold or disposed of while

who are neighbors, lie along the banks of

the donor is still alive.

the Marikina River. At certain times of the


year, the river would swell and as the water
recedes, soil, rocks and other materials are

Never Let The Odds Keep You From Pursuing What You Know In Your Heart You Were Meant To Do.Leroy Satchel Paige

Page 65 of 180

Civil Law Q&As (2007-2013)

hectorchristopher@yahoo.com

dbaratbateladot@gmail.com

current of the water." Where the land is


deposited
properties.

on

Jessica's

This

pattern

and
of

Jenny's
the

river

swelling, receding and depositing soil and


other materials being deposited on the
neighbors' properties have gone on for
many years. Knowing his pattern, Jessica
constructed a concrete barrier about 2
meters

from

her

property

line

and

extending towards the river, so that when


the water recedes, soil and other materials
are trapped within this barrier. After several
years, the area between Jessica's property
line to the concrete barrier was completely
filled

with

soil,

effectively

increasing

Jessica's property by 2 meters. Jenny's


property, where no barrier was constructed,
also increased by one meter along the side
of the river.
(A). Can Jessica and Jenny legally claim
ownership over the additional 2 meters and
one meter, respectively, of land deposited
along their properties?(2%)
SUGGESTED ANSWER:
Only Jenny can claim ownership over
the

additional

one

meter

of

land

deposited along her property. Art. 457 of


the Civil Code provides that "to the
owners of lands adjoining the banks of
river belong the accretion which they
gradually receive from the effects of the

not formed solely by the natural effect of


the water current of the river bordering

(B). If Jessica's and Jenny's properties are


land but is also the consequences of the
direct and deliberate intervention of
man, it is man-made accretion and a
part of the public domain (Tiongco v.

registered,

will

the

benefit

of

such

registration extend to the increased area of


their properties? (2%)
SUGGESTED ANSWER:

Director of Lands, 16 C.A. Rep 211, cited


in Nazareno v. C.A., G.R. No. 98045, 26
June 1996). Thus, Jessica cannot legally
claim ownership of the additional 2
meters
because

of

land

she

along

her

constructed

property
concrete

barrier about 2 meters from her property


causing

deposits

of

soil

and

other

materials when the water recedes. In


other words, the increase in her property
was not caused by nature but was manmade.

If the properties of Jessica and Jenny


are

registered,

registration
increased
Accretion

the

does
area
does

benefit

not
of

extend

their
not

of
to

such
the

properties.

automatically

become registered land because there is


a specific technical description of the lot
in its Torrens title. There must be a
separate application for registration of
the alluvial deposits under the Torrens
System (Grande v. CA, G.R. No. L-17652,
30 June, 1962).

Never Let The Odds Keep You From Pursuing What You Know In Your Heart You Were Meant To Do.Leroy Satchel Paige

Page 66 of 180

Civil Law Q&As (2007-2013)

hectorchristopher@yahoo.com

dbaratbateladot@gmail.com

up in the mountains and forests so that


(C). Assume the two properties are on a cliff
adjoining the shore of Laguna Lake. Jessica
and Jenny had a hotel built on the
properties. They had the erath and rocks
excavated from the properties dumped on
the adjoining shore, giving rise to a new
patch of dry land. Can they validly lay claim
to the patch of land? (2%)
SUGGESTED ANSWER:
No. Jessica and Jenny cannot validly lay
claim to the patch of land because in
order to acquire land by accretion, there
should

be

natural

and

actual

continuity of the accretion to the land of


the riparian owner caused by natural ebb
and flow of the current of the river
(Delgado v. Samonte, CA-G.R. No. 34979R, 10 Aug 1966).

Accretion; Rights of the Riparian Owner


(2009)
No.XVI. Marciano is the owner of a parcel of
land through which a river runs out into
the sea. The land had been brought under
the Torrens System, and is cultivated by
Ulpiano and his family as farmworkers
therein. Over the years, the river has
brought silt and sediment from its sources

gradually the land owned by Marciano


increased in area by three hectares.

Ulpiano built three huts on this additional

SUGGESTED ANSWER:

married

Marcianos contention is correct. Since

children live. On this same area, Ulpiano

that accretion was deposited on his land

and his family planted peanuts, monggo

by the action of the waters of the river

beans

also

and he did not construct any structure

regularly paid taxes on the land, as shown

to increase the deposition of soil and

by tax declarations, for over thirty years.

silt, Marciano automatically owns the

area,

where

and

he

and

his

vegetables.

two

Ulpiano

When Marciano learned of the increase in


the size of the land, he ordered Ulpiano to

accretion. His real right of ownership is


enforceable

against

the

whole

world

demolish the huts, and demanded that he

including Ulpiano and his two married

be paid his share in the proceeds of the

children. Although Marcianos land is

harvest. Marciano claims that under the

registered, the three (3) hectares land

Civil Code, the alluvium belongs to him as a

deposited through accretion was not

registered riparian owner to whose land the

automatically

accretion attaches, and that his right is

unregistered

enforceable against the whole world.

registered.
land,

it

is

As

an

subject

to

acquisitive prescription by third persons.

(A). Is Marciano correct? Explain. (3%)


Never Let The Odds Keep You From Pursuing What You Know In Your Heart You Were Meant To Do.Leroy Satchel Paige

Page 67 of 180

Civil Law Q&As (2007-2013)

hectorchristopher@yahoo.com

dbaratbateladot@gmail.com

accounting of the fruits he has gathered,


he has the right to deduct from the
Although Ulpiano and his children live in
the three (3) hectare unregistered land
owned

by

Marciano,

they

are

farm

workers; therefore, they are possessors


not in the concept of owners but in the
concept of mere holders. Even if they
possess the land for more than 30 years,
they cannot become the owners thereof
through

extraordinary

acquisitive

prescription, because the law requires


possession in the concept of the owner.
Payment of taxes and tax declaration are
not enough to make their possession one
in the concept of owner. They must
repudiate the possession in the concept
of holder by executing unequivocal acts
of repudiation amounting to ouster of
Marciano, known to Marciano and must
be

proven

evidence.

by
Only

clear

and

then

convincing
would

his

possession become adverse.


(B). What rights, if any, does Ulpiano have
against Marciano? Explain. (3%)
SUGGESTED ANSWER:
Although Ulpiano is a possessor in bad
faith, because he knew he does not own
the land, he will lose the three huts he
built

in

bad

faith

and

make

an

value of the fruits the expenses for

CRC likewise sold to the spouses Rodriguez,


production, gathering and preservation
of the fruits (Art 443, NCC).
He may also ask for reimbursement of

a 700-square meter land (Lot B) which is


adjacent to Lot A. Lot B has a present fair
market value of P1,500,000.
The spouses Dela Cruz constructed a house

the taxes he has paid, as these are

on Lot B, relying on their presentation of

charges on the land owned by Marciano.

the CRC sales agent that it is the property

This obligation is based on a quasi-

they purchased. Only upon the completion

contract (Art 2175, NCC).

of their house did the spouses Dela Cruz


discovered that they had built on Lot B
owned by the spouses Rodriguez, not on Lot

Builder; Good Faith; Requisites (2013)


No.VIII. Ciriaco Realty Corporation (CRC)
sold to the spouses Del a Cruz a500-square
meter land (Lot A) in Paranaque. The land
now has a fair market value of Pl,200,000.

A that they purchased. They spent P 1


000,000 for the house.
As their lawyer, advise the spouses Dela
Cruz on their rights and obligations under
the given circumstances, and the recourses

Never Let The Odds Keep You From Pursuing What You Know In Your Heart You Were Meant To Do.Leroy Satchel Paige

Page 68 of 180

Civil Law Q&As (2007-2013)

hectorchristopher@yahoo.com

dbaratbateladot@gmail.com

(1) to appropriate as his own the works


and options open to them to protect their

after payment of the indemnity provided

interests. (8%)

for in Art 546 and 548, or

SUGGESTED ANSWER:

(2) to oblige the one who built to pay the

Based on the fact as stated, the spouses


Dela Cruz as builders and the spouses
Rodriguez as land owners, are both in
good faith. The spouses Dela Cruz are
builder in good faith because before
constructing the house they exercised
due diligence by asking the Agent of CRC
the location of the lot A, and they relied
on the information given by the agent
who is presumed to know the identity of
the lot purchased by the Dela Cruz
spouses (Pleasantville v. CA, 253 SCRA
10, 1996). On the other hand, there is no
showing that the land owners, spouse
Rodriguez acted in bad faith. The facts
do not show that the building was done
with

their

knowledge

and

without

opposition on their part (Art 453, Civil


Code). The good faith is always presumed
(Art. 527, Civil Code).
The owner of the land on which anything
has been built, sown, or planted in good
faith shall have the right:

price of the land.

entitled to the right of retention pending


However, the builder cannot be obliged

reimbursement of the expenses they

to

is

incurred or the increase in value which

the

the thing may have acquired by reason of

building.. In such case, he shall pay

the improvement (Art 546, Civil Code).

reasonable rent of the owner of the land

Thus,

does

the

demand P1,000,000.00 as payment of

building or trees after proper indemnity

the expenses in building the house or

(Art 448, Civil Code).

increase in value of the land because of

buy

the

considerable

not

land
more

choose

if

its

than

to

value

that

of

appropriate

the

spouses

Dela

Cruz

may

the house as a useful improvement, as


The house constructed by the spouses

may be determined by the court form

Dela Cruz is considered as a useful

the evidence presented during the trial

expense, since it increased the value of

(Depra v. Dumlao, 136 SCRA 475, 1985;

the lot. As such, should the spouses

Technogas Phils v. CA, 268 SCRA 5,

Rodriguez decides to appropriate the

1997).

house,

the

spouses

Dela

Cruz

are

Never Let The Odds Keep You From Pursuing What You Know In Your Heart You Were Meant To Do.Leroy Satchel Paige

Page 69 of 180

Civil Law Q&As (2007-2013)

hectorchristopher@yahoo.com

dbaratbateladot@gmail.com

been using this pathway (pathway A) since


Easement;

Prescription;

Acquisitive

Prescription (2009)
No. XI. TRUE or FALSE. Answer TRUE if
the statement is true, or FALSE if the
statement is false. Explain your answer in
not more than two (2) sentences.
(C). Acquisitive prescription of a negative
easement runs from the time the owner of
the dominant estate forbids, in a notarized
document, the owner of the servient estate
from executing an act which would be
lawful without the easement. (1%)
SUGGESTED ANSWER:
True. In negative easements, acquisitive
prescription runs from the moment the
owner of the dominant estate forbade, by
an

instrument

acknowledged

before

notary public, the owner of the servient


estate from executing an act which
would be lawful without the easement
(Art. 621, NCC).

Easement; Right of Way (2013)


No.VII.In 2005, Andres built a residential
house on a lot whose only access to the
national highway was a pathway crossing
Brando's property. Andres and others have

1980.

determine
In 2006, Brand0 fenced off his property,
thereby blocking Andres' access to the
national highway. Andres demanded that
part of the fence be removed to maintain
his

old

access

route

to

the

highway

(pathway A), but Brando refused, claiming


that there was another available pathway
(pathway B) for ingress and egress to the
highway. Andres countered that pathway B
has defects, is circuitous, and is extremely
inconvenient to use.

hired Damian, a geodetic and civil engineer,


to survey and examine the two pathways
the

surrounding

way

shortest
through

and
the

the

least

servient

estates. After the survey, the engineer


concluded that pathway B is the longer
route and will need improvements and
repairs, but will not significantly affect the
use of Brando's property. On the other
hand, pathway A that had long been in
place, is the shorter route but would
significantly affect the use of Brando's
property.
In light of the engineer's findings and the

To settle their dispute, Andres and Brando

and

prejudicial

the

areas,

and

to

circumstances of the case, resolve the


parties' right of way dispute. (6%)
SUGGESTED ANSWER:

Never Let The Odds Keep You From Pursuing What You Know In Your Heart You Were Meant To Do.Leroy Satchel Paige

Page 70 of 180

Civil Law Q&As (2007-2013)

hectorchristopher@yahoo.com

Andres is not entitled to the easement of


right of way for Pathway A. Pathway B
must be used.
The owner of a dominant estate may
validly obtain a compulsory right of way
only

after

he

has

established

the

existence of four requisites, to wit:


(1) The (dominant) estate is surrounded
by other immovables and is without
adequate outlet to a public highway;
(2)

After

payment

of

the

proper

indemnity;
(3) The isolation was not due to the
proprietors own acts; and
(4) The right of way claimed is at a point
least prejudicial to the servient estate,
and insofar as consistent with this rule,
where the distance from the dominant
estate to the public highway maybe the
shortest (Art 650, civil Code).
However,

the

Supreme

Court

has

consistently ruled that in case both


criteria cannot be complied with, the
right of way shall be established at the
point least prejudicial to the servient
estate.

dbaratbateladot@gmail.com

The first and fourth requisites are not


complied with. First, there is another
available outlet to the national highway

to the servient estate is controlling


(Pathway B). Second, the right of way

(Quimen v. Quimen and CA, G.R. No.

obtained (Pathway A) is not the least

112331, May 29, 1996).

prejudicial

to

Brandos

property,

as

evidence by the reports of the geodetic


and civil engineer.
When

there

adequate

is

outlet

exists an easement in favor of the lot


belonging to Andres and if Brandos lot is

already
from

an

the

existing
dominant

estate to the public highway, even if the


said outlet, for one reason or another, be
inconvenient,

(Note: It is not clear from the problem if there

easement. If the used pathway was only a

must ask for the constitution of a legal

(Costabella Corporation v. CA, G.R. No.

easement through Brandos lot by proving

80511, Jan 25, 1991). The rule that the

the four requisites required by Art 649 and

easement

65, Civil Code).

way

open

created as legal easement or as a voluntary

another servitude is entirely unjustified

of

to

easement burdening Brandos lot, was it

tolerance, then Brando may close it. Andres

right

need

way as a servient estate. If there is such an

up

of

the

burdened as a servient estate by a right of

shall

be

established at the point least prejudicial

Never Let The Odds Keep You From Pursuing What You Know In Your Heart You Were Meant To Do.Leroy Satchel Paige

Page 71 of 180

Civil Law Q&As (2007-2013)

hectorchristopher@yahoo.com

Easement; Right of Way (2010)


No.XIII. Franz was the owner of Lot E which
was surrounded by four (4) lots one of
which Lot C he also owned. He promised
Ava that if she bought Lot E, he would give
her a right of way in Lot C.
Convinced, Ava bought Lot E and, as
promised, Franz gave her a right of way in
Lot C.
Ava cultivated Lot E and used the right of
way granted by Franz.
Ava later found gainful employment abroad.
On her return after more than 10 years, the
right of way was no longer available to her
because Franz had in the meantime sold
Lot C to Julia who had it fenced.
(A). Does Ava have a right to demand from
Julia the activation of her right of way?
Explain. (2.5%)
SUGGESTED ANSWER:
Yes. Ava has the right to demand from
Julia the activation of the right of way,
for the following reasons:
(1) The easement of the right of way is a
real right which attaches to, and is
inseperable from, the estate to which it
belongs.

dbaratbateladot@gmail.com

(2) The sale of the property includes the

ten years of non-user, shall be computed


easement or servitude, even if the deed

from the day it ceased to be used under

of sale is silent on the matter.

Act 6341 (2) CC.

(3) The vendee of the property in which a


servitude

or

easement

exists

cannot

close or put obstructions thereon to


prevent the dominant estate from using
it.

(5)

Renunciation

or

waiver

of

an

easement must be specific, clear, express


and made in a public instrument in
accordance of Art 1358 of the New Civil
Code.
ALTERNATIVE ANSWER:

(4) Avas working abroad for more than

Yes. Ava has the right to demand from

ten (10) years should not be construed as

Julia the activation of the right of way. A

non-user, because it cannot be implied

voluntary easement of right of way, like

from the fact that she or those she left

any

behind to cultivate the lot no longer use

extinguished only by mutual agreement

the right of way.

or by renunciation of the owner of the

other

contract,

could

be

dominant estate. Also, like any other


Note:

Since

right

of

way

is

discontinuous easement, the period of


Never Let The Odds Keep You From Pursuing What You Know In Your Heart You Were Meant To Do.Leroy Satchel Paige

Page 72 of 180

Civil Law Q&As (2007-2013)

hectorchristopher@yahoo.com

dbaratbateladot@gmail.com

ALTERNATIVE ANSWER:
contract,

an

easement

is

generally

effective between parties, their heirs and


assignees, except in case where the
rights and obligations arising from the
contract are not transmissible by their
nature, or by stipulations or by provision
of law (Unisource Commercial v. Chung,
593 SCRA 530 [2009]).
(B). Assuming Ava opts to demand a right of
way from any of the owners of Lots A, B,
and D, can she do that? Explain. (2.5%)
SUGGESTED ANSWER:
Yes. Ava has the option to demand a
right of way on any of the remaining lots
of Franz more so after Franz sold lot C
to Julia. The essential elements of a
legal right of way under Art 649 and 650
of the New Civil Code are complied with.
ALTERNATIVE ANSWER:
Yes. Ava has the option to demand a
right of way from the other lots. The law
provides that whenever a piece of land
acquired by sale, exchange or partition is
surrounded

by

other

estates

of

the

vendor, exchanger, or co-owner, he shall


be obliged to grant a right of way
without indemnity (Art 652, NCC).

No. There was merely a promise to Ava


that a right of way shall be granted to

lot which he (Blas) owns. While digging on


her in lot C if Ava purchase lot E. The
promise was not reduced to writing (Obra
v. Baldria, 529 SCRA 621 [2007]). Hence,
it was not or could not have been
registered as to warn buyers of lot C

the lot in order to lay down the foudation of


the house, Adam hit a very hard object. It
turned out to be the vault of the old Banco
de las Islas Filipinas. Using a detonation
device, Adam was able to open the vault
containing old notes and coins which were

about the existence of the easement on

in circulation during the Spanish era. While

the property. Not having been annotated

the notes and coins are no longer legal

on the TCT to lot C, the buyer acquired

tender, they were valued at P100 million

lot C free from such right of way granted

because of their historical value and the

to Ava.

coins silver nickel content. The following


filed legal claims over the notes and coins:
(i). Adam, as finder;

Hidden Treasure (2008)


(ii). Blas, as owner of the property where
No. VIII. Adam, a building contractor, was

they were found;

engaged by Blas to construct a house on a


Never Let The Odds Keep You From Pursuing What You Know In Your Heart You Were Meant To Do.Leroy Satchel Paige

Page 73 of 180

Civil Law Q&As (2007-2013)

hectorchristopher@yahoo.com

dbaratbateladot@gmail.com

made on the property of another, or of


(iii). Bank of the Philippine Islands, as

the State and by chance, one-half of it

successor-in-interest of the owner of the

shall belong to the finder who is not a

vault; and

trespasser (Art. 438, Civil Code). In the

(iv). The Philippine Government because of


their historical value.
(A). Who owns the notes and coins? (4%)
SUGGESTED ANSWER:
The notes and coins are no longer owned
by the Bank of the Philippine Islands,
which has either lost or abandoned the
vault and its contents, and it has not
taken any effort to search, locate or
recover the vault. In any case, since the
vault is in actual possession of Adam,
BPI may attempt, in a judicial action to
recover, to rebut the presumption of
ownership in favor of Adam and Blas
(Art. 433, Civil Code). Hidden treasure is
any hidden and unknown deposit of
money,

jewelry,

or

other

precious

objects, the lawful ownership of which


does not appear. Given the age and
importance of the items found, it would
be safe to consider the vault, notes and
coins

abandoned

by

BPI

and

its

predecessor (Art. 439, Civil Code). It


belongs to the owner of the land on
which it is found. When the discovery is

present case, Adam, as finder, and Blas,


as owner of the land, are entitled to
share

50-50

in

the

treasure.

The

government can only claim if it can


establish that the notes and coins are of
interest to science or the arts, then it

If either or both Adam and Blas are


adjudged as owners, the notes and coins
shall be deemed part of their absolute
community or conjugal partnership of
gains with their respective spouses (Art.
117, par 4, FC).

must pay just price of the things found,


to be divided equally between Adam and
Blas (Art. 438, Civil Code).

Mortgage; Public or Private Instrument


(2013)

(B). Assuming that either or both Adam and


Blas are adjudged as owners, will the notes

No.VI. Lito obtained a loan of P1,000,000

and coins be deemed part of their absolute

from Ferdie, payable within one year. To

community or conjugal partnership of gains

secure payment, Lito executed a chattel

with their respective spouses? (2%)

mortgage on a Toyota Avanza and a real


estate mortgage on a 200-square meter

SUGGESTED ANSWER:

piece of property.

Never Let The Odds Keep You From Pursuing What You Know In Your Heart You Were Meant To Do.Leroy Satchel Paige

Page 74 of 180

Civil Law Q&As (2007-2013)

hectorchristopher@yahoo.com

(A) Would it be legally significant - from the


point of view of validity and enforceability if the loan and the mortgages were in public
or private instruments? (6%)
SUGGESTED ANSWER:
From the point of view of validity and
enforceability,

there

would

be

legal

significance if the mortgage was in a


public or private instrument. As for the
loan,

there

is

no

legal

significance

except of interest were charged on the


loan, in which case, the charging of
interest must be in writing.
A contract of loan is a real contract and
is perfected upon delivery of the object
of the obligation (Art 1934, Civil Code).
Thus, a contract of loan is valid and
enforceable even if it is neither in a
private nor in a public document.
As a rule, contracts shall be obligatory in
whatever

form

they

may

have

been

entered into provided all the essential


requisites for their validity are present.
With regards to its enforceability, a
contact of loan is not among those
enumerated under Art. 1403 (2) of the
Civil Code, which are covered by the
Statute of Frauds.

dbaratbateladot@gmail.com

It is important to note that under Art.


1358 of the Civil Code, all the other
contracts where the amount involved

order that a mortgage may be validly


exceeds Five Hundred pesos (P500.00)

constituted the document in which it

must appear in writing, even in private

appears be recorded. If the instrument is

one. However, the requirement is not for

not

validity of the contract, but only for its

nevertheless valid and binding between

greater efficacy.

the parties. Hence, for validity of both

recorded,

the

mortgage

is

chattel and real estate mortgages, they


With regard to the chattel mortgage, Art.

must appear in a public instrument. But

1508, the Chattel Mortgage Law, requires

the

an affidavit of good faith stating that the

submitted that the form of the contract,

chattel mortgage is supposed to stand as

whether in a public or private document,

security

would

of

the

loan;

thus,

for

the

validity of the chattel mortgage, it must

purpose

be

of

enforceability,

immaterial

(Mobil

it

Oil

is

v.

Diocaresa, 29 SCRA 656, 1969).

be in a public document and recorded in


the Chattel Mortgage Register in the

Also, under Art 1358, acts and contracts

Register

which have for their object the creation

of

Deeds.

real

estate

mortgage, under the provisions of Art.

or

transmission

of

real

rights

over

2125 of the Civil Code, requires that in

immovable property must be in a public

Never Let The Odds Keep You From Pursuing What You Know In Your Heart You Were Meant To Do.Leroy Satchel Paige

Page 75 of 180

Civil Law Q&As (2007-2013)

hectorchristopher@yahoo.com

dbaratbateladot@gmail.com

intention to become the owner thereof.


document for greater efficacy and a real

Possession, on the other hand, is the

estate mortgage is a real right over

holding of the thing or an enjoyment of a

immovable property.

Occupation vs. Possession (2007)


No.I. Distinguish the following concepts:
(A). Occupation v. possession. (5%)
SUGGESTED ANSWER:
Occupation

is

an

original

mode

of

acquiring ownership (Art. 712, NCC).


Things appropriable by nature which are
without an owner, such as animals that
are the object of hunting and fishing,
hidden

treasure

and

abandoned

movables, are acquired by occupation


(Art. 713, NCC). However, ownership of a
piece of land cannot be acquired by
occupation (Art. 714, NCC).
ALTERNATIVE ANSWER:
Occupation

is

mode

of

acquiring

dominion by the seizure of corporeal


things which have no owner, with the
intention of acquiring the ownership
thereof.

It

is

an

original

mode

of

acquiring ownership upon seizure of a


res nullius by the occupant who has the

right. Possession may be the real right of


possession or jus possessiones or it can
be merely the right to possess or jus
possedendi, which are among the basic

No. XI. TRUE or FALSE. Answer TRUE if


the statement is true, or FALSE if the
statement is false. Explain your answer in
not more than two (2) sentences.

rights of ownership. If the real right of

(D). The renunciation by a co-owner of his

possession is possession in the concept

undivided share in the co-owned property

of

in lieu of the performance of his obligation

owner,

limitations,

but
it

subject
may

to

ripen

certain
into

full

ownership of the thing or property right


through

acquisitive

prescription

depending on whether it is a case of

to contribute to taxes and expenses for the


preservation of the property constitutes
dacion en pago. (1%)
SUGGESTED ANSWER:

ordinary or extraordinary prescription

True, Under the Civil Code, a co-owner

and whether the property is movable or

may renounce his share in the co-owned

immovable.

property in lieu of paying for his share in


the

Ownership; Co-Ownership (2009)

taxes

and

expenses

for

the

preservation of the co-owned property.

Never Let The Odds Keep You From Pursuing What You Know In Your Heart You Were Meant To Do.Leroy Satchel Paige

Page 76 of 180

Civil Law Q&As (2007-2013)

In

effect,

there

is

hectorchristopher@yahoo.com

dacion

en

pago

because the co-owner is discharging his


monetary obligation by paying it with his
non-monetary interest in the co-owned
property. The fact that he is giving up
his entire interest simply means that he
is accepting the value of his interest as
equivalent to his share in the taxes and
expenses of preservation.

Ownership; Co-Ownership (2008)


No. VI. Alex died without a will, leaving only
an undeveloped and untitled lot in Tagiug
City. He is survived by his wife and 4
children. His wife told the children that she
is waiving her share in the property, and
allowed Bobby, the eldest son who was
about to get married, to construct his
house on of the lot, without however
obtaining the consent of his siblings. After
settlement of Alex's estate and partition
among the heirs, it was discovered that
Bobby's house was constructed on the
portion allocated to his sister, Cathy asked
Bobby to demolish his house and vacate the
portion alloted to her. In leiu of demolition,
Bobby offered to purchase from Cathy the
lot

portion

on

which

his

constructed.

At

that

time,

house
the

constructed was valued at P350.000.

was
house

dbaratbateladot@gmail.com

(A). Can Cathy lawfully ask for demolition of


Bobby's house? (3%)

dominion

or

ownership

such

as

construction of a house." In the present

SUGGESTED ANSWER:

case, of Alex is the real owner of the


Yes, Cathy can lawfully ask for the

undeveloped and untitled lot in Taguig,

demolition

Where

co-ownership is created among his wife

there are two or more heirs, the whole

and four children over said property

estate

before

upon his death. Since the construction

partition, owned in common by such

of the house by Bobby was done without

heirs, subject to the payment of debts of

obtaining the consent of his siblings, the

the deceased (Art. 1078, Civil Code),

alteration effected is illegal. Bobby is

Under the rules on co-ownership, "none

considered to be in bad faith and as a

of

sanction for his conduct, he can be

the

of

of

the

Bobby's

house.

decedent,

co-owners

shall,

is,

without

the

consent of the others make alterations

compelled

in the thing owned in common, even

remove

though benefits for all would results

expense.

by

the

Cathy

to

structure

demolish
at

his

or
own

therefrom." In Cruz v. Catapang, G.R. No.


164110, 12 Feb., 2008, the Court held

(B). Can Bobby legally insist on purchasing

that "alterations include any act of strict

the land? (2%)

Never Let The Odds Keep You From Pursuing What You Know In Your Heart You Were Meant To Do.Leroy Satchel Paige

Page 77 of 180

Civil Law Q&As (2007-2013)

hectorchristopher@yahoo.com

that

Bobby

cannot

legally

insist

on

purchasing the land. Being in bad faith,


he has no option to pay for the price of
the lot (Art. 450, Civil Code).

Property; Movable or Immovable (2007)


No.II. Manila Petroleum Co. owned and
operated a petroleum operation facility off
the coast of Manila. The facility was located
on a floating platform made of wood and
metal,

upon

which

was

permanently

attached the heavy equipment for the


petroleum operations and living quarters of
the crew. The floating platform likewise
contained a garden area, where trees,
plants

and

flowers

and

structures

which,

though floating, are intended by their

SUGGESTED ANSWER:
No.

"docks

dbaratbateladot@gmail.com

were

planted.

The

platform was tethered to a ship, the MV


101, which was anchored to the seabed.
Please briefly give the reason for your
answers. (10%)
(A).Is the platform movable or immovable
property?
SUGGESTED ANSWER:
The platform is an immovable property
under Art. 415 (9) NCC, which provides

hence, it remains a movable property. If


nature and object to remain at a fixed

the intention was to make the platform

place on a river, lake or coast." Since the

stay permanent where it was moored, it

floating

would not have been simply tethered to a

operation

platform
facility,

is
it

a
is

petroleum
intended

to

remain permanently where it is situated,


even if it is tethered to a ship which is

vessel but itself anchored to the seabed.


(B). Are the equipment and living quarters
movable or immovable property?

anchored to the seabed.


SUGGESTED ANSWER:
ALTERNATIVE ANSWER:
The thing and living quarters of the crew
The platform is a movable property
because it is attached to a movable
property,

i.e.

the

vessel

which

was

merely anchored to the seabed. The fact


that the vessel is merely anchored to the
sea

bed

only

shows

that

it

is

not

are immovable property under Art. 415


(3) NCC, classifies as an immovable
"everything attached to an immovable in
a fixed manner, in such a way that it
cannot be separated therefrom without
breaking the material or deterioration of

intended to remain at a fixed place;


Never Let The Odds Keep You From Pursuing What You Know In Your Heart You Were Meant To Do.Leroy Satchel Paige

Page 78 of 180

Civil Law Q&As (2007-2013)

hectorchristopher@yahoo.com

dbaratbateladot@gmail.com

(C). Are the trees, plants and flowers


the object." Both the equipment and the
living quarters are permanently attached
to

the

platform

which

is

also

an

immovable. The equipment can also be


classified

as

an

immovable

property

under Art. 415 (5) NCC because such


equipment are "machinery, receptacles,
instruments or implements intended by
the

owner

of

the

tenement

for

an

industry or works which may be carried


on in a building or on a piece of land and
which tend directly to meet the needs of
the industry or works." It is logically
assumed that the petroleum industry
may be carried on in a building or on a
piece

of

land

and

the

platform

is

analogous to a building.
ALTERNATIVE ANSWER:
The equipment and living quarters of the
crew are movable properties since they
are attached to a platform which is also
a movable property, because it is simply
attached

to

vessel

is

likewise

movable property since it was merely


anchored on the seabed only shows that
it is not intended to remain at a fixed
place;

hence,

property.

it

remains

movable

immovable or movable property?


SUGGESTED ANSWER:

immovable.

The

platform

is

not

an

The trees, plants and flowers planted in

immovable property for the same reason

the garden area of the platform are

already given in the Alternative Answer

immovable property under Art. 415 (2)

to Item (a) above.

NCC which classifies as an immovable


property

"trees,

plants

and

growing

fruits, while they are attached to the


land or form an integral part of an
immovable,

the

petroleum

operation

facility.

Land Titles and Deeds


Acquisition

of

Lands;

Sale

of

Real

Property to an Alien (2009)


No.XIX. In 1972, Luciano de la Cruz sold to
Chua Chung Chun, a Chinese citizen, a

ALTERNATIVE ANSWER:

parcel of land in Binondo. Chua died in

The trees, plants and flowers planted in


the garden area of the platform are
movable property because they are not

1990, leaving behind his wife and three


children,

one

of

whom,

Julian,

is

naturalized Filipino citizen. Six years after


Chuas

death,

the

heirs

executed

an

permanently attached t the land and do

extrajudicial settlement of estate, and the

not

parcel of land was allocated to Julian. In

form

an

integral

part

of

an

Never Let The Odds Keep You From Pursuing What You Know In Your Heart You Were Meant To Do.Leroy Satchel Paige

Page 79 of 180

Civil Law Q&As (2007-2013)

hectorchristopher@yahoo.com

2007, Luciano filed suit to recover the land

dbaratbateladot@gmail.com

Non-Registrable Properties (2007)

he sold to Chua, alleging that the sale was


void

because

it

contravened

the

Constitution which prohibits the sale of


private lands to aliens. Julian moved to
dismiss the suit on grounds of pari delicto,
laches and acquisitive prescription. Decide

No.IV.

(B).

What

properties

are

not

registrable? (5%)
Supply this information.
SUGGESTED ANSWER:

the case with reasons. (4%)


The

following

properties

are

not

SUGGESTED ANSWER:

registrable:

The case must be dismissed. Julian, who

(1.) Properties of the Public dominion;

is a naturialized Filipino citizen and to


whom the property was allocated in a n

(2.) Properties for public use or public

extra-judicial partition of the estate, is

service;

now the owner of the property. The


defect in ownership of the property of
Julians alien father has already been

(3.)

Inalienable

lands

of

the

public

domain;

cured by its transfer to Julian. It has

(4.) Military installations, civil and quasi-

been validated by the transfer of the

public lands; and

property to a Filipino citizen. Hence,


there

is

no

more

violation

of

the

Constitution because the subject real


property is now owned by a Filipino
citizen (Halili v. CA, 287 SCRA 465,

(5.) All lands not classified as alienable


and disposable.
ALTERNATIVE ANSWER:

[1998]). Further, after the lapse of 35

(1).

Properties

year, laches has set in and the motion to

intended

dismiss may be granted, for the failure of

canals,

Luciano to question the ownership of

bridges constructed by the State, banks,

Chua before its transfer of ownership to

shores, roadsteads, and the like, are

Julian.

incapable of private appropriation, much

for

of

public

rivers,

public
use,

torrents,

dominium
like
ports

roads,
and

less registration (Art. 420 NCC). This

includes public markets, public plazas,


municipal streets and public buildings
Never Let The Odds Keep You From Pursuing What You Know In Your Heart You Were Meant To Do.Leroy Satchel Paige

Page 80 of 180

Civil Law Q&As (2007-2013)

hectorchristopher@yahoo.com

dbaratbateladot@gmail.com

to the public domain, and can only be


(Municipality of Antipolo v. Zapanta, 133

acquired by lease if not needed by the

SCRA 820, 1986; Martinez v. CA, 56

government for public or quasi-public

SCRA 647, 1974; Navera v. Quicho, 5

purposes.

SCRA 454, 1962).


(2.) Lands proclaimed or classified as
forest, timberlands, mineral lands and
national parks. Under Sec 2, Art XII,
Constitution of the Philippines, these
lands are inalienable.
(3.) Lands that are reserved by law or
Presidential proclamation for military,
civic or quasi-public purpose, Under Sec
88, Chapter XII of the Public Land Act,
such lands shall be inalienable and shall
not be subject to occupation, entry, sale,
lease or other disposition.
(4.) In general, all lands of the public
domain that has not been classified as
alienable and disposable under the Public
Land Act.
(5.) Lands that form part of the seabed,
riverbed or lakebed. These lands are not
susceptible to private appropriation.
(6.) Foreshore lands is that strip of land
that lies between the high and low water
marks

and

alternately

wet

and

dry

according to the flow of the tide belong

on the property. Anthony is not aware of the


(7.) Lands reclaimed by the government
from the sea, lakes, or other bodies of
water are disposed or acquisible only by
lease

and

not

otherwise,

under

the

defect in Bert's title, but has been in actual


physical possession of the property from
the time he bought it from Bert, who had
never been in possession of the property for
one year.

Public Land Act.


(A). Can Anthony acquire ownership of the
property by acquisitive prescription? How
Prescription;

Acquisitive

Prescription

(2008)
No. VII. Anthony bought a piece of untitled
agricultural land from Bert. Bert, in turn,
acquired the property by forging carlo's
signature in a deed of sale over the
property. Carlo had been in possession of
the property for 8 years, declared it for tax

many more years does he have possess it to


acquire ownership? (2%)
SUGGESTED ANSWER:
Yes, Anthony can acquire ownership of
the

property

prescription.

through
In

the

acquisitive

present

case,

Anthony is a buyer/possessor in good

purposes, and religiously paid all taxes due


Never Let The Odds Keep You From Pursuing What You Know In Your Heart You Were Meant To Do.Leroy Satchel Paige

Page 81 of 180

Civil Law Q&As (2007-2013)

hectorchristopher@yahoo.com

Yes,

Carlos

dbaratbateladot@gmail.com

can

appropriate

only

faith because he was not aware of the

portion of the standing crops on the

defect in Bert's title (Art. 526, Civil

property once he recovers possession.

Code). As such, Anthony can acquire

Anthony being a possessor in good faith,

ownership and other real rights over


immovable

property

through

open,

continuous possession of 10 years (Art.


1134, Civil Code). Anthony needs nine
(9) more years of possession, in addition
to his one (1) year of possession in good
faith.
(B).If Carlo is able to legally recover his
property,

can

he

require

Anthony

to

account for all the fruits he has harvested


from the property while in possession? (2%)
SUGGESTED ANSWER:
If Carlo is able to legally recover his
property, he cannot require Anthony to
account for all the fruits harvested from
the property. Anthony is entitled to the
fruits harvested in good faith before his
possession was legally interrupted (Art.
544, Civil Code).
(C).If there are standing crops on the
property when Carlo recovers possession,
can Carlo appropriate them? (2%)
SUGGESTED ANSWER:

SUGGESTED ANSWER:
shall have a right to a part of the

Yes, it prescribes in five (5) years. If the

expenses of cultivation, and to a part of

real property mortgaged

is judicially

the net harvest of the standing crops,

foreclosed,

for

both in proportion to the time of the

foreclosure should be filed within a

possession (Art 545, Civil Code).

period of ten (10) years. The request for

the

action

judicial

issuance of a writ of possession should


be filed upon motion of the winning
Prescription; Judicially Foreclosed Real

bidder within five (5) years after the

Property Mortgage (2012)

judgment of foreclosure. The writ of

No.IX.a) Does the right to request for the


issuance of a writ of possession over a

possession is an order commanding the


sheriff to place a person named therein

foreclosed real property prescribe in five (5)

in possession of real property (BPI v.

years? (5%)

Icot. G.R. No. 168081, Oct 12, 2009).

Never Let The Odds Keep You From Pursuing What You Know In Your Heart You Were Meant To Do.Leroy Satchel Paige

Page 82 of 180

Civil Law Q&As (2007-2013)

hectorchristopher@yahoo.com

Bank
Purchaser in

Good

Faith;

Mortgaged

Property (2008)
No. XIX. Juliet offered to sell her house and
lot, together with all the furniture and
appliances

therein

to

Dehlma.

Before

agreeing to purchase the property, Dehlma


went to the Register of Deeds to verify
Juliet's title. She discovered that while the
property was registered in Juliet's name
under

the

amended

Land

by

the

Registration
Property

Act,

as

Registration

Decree, it property, Dehlma told Juliet to


redeem the property from Elaine, and gave
her an advance payment to be used for
purposes of realesing the mortgage on the
property. When the mortgage was released,
Juliet executed a Deed of Absolute Sale over
the property which was duly registered with
the Registry of Deeds, and a new TCT was
issued

in

Dehlma's

name.

Dehlma

immediately took possession over the house


and

lot

and

the

movables

therein.

Thereafter, Dehlma went to theAssessor's


Office to get a new tax declaration under
her name. She was surprised to find out
that the property was already declared for
tax purposes in the name of XYZ Bank
which had foreclosed the mortgage on the
property before it was sold to her. XYZ Bank
was also the purchaser in the foreclosure
sale of the property. At that time, the
property was still unregistered but XYZ

registered

dbaratbateladot@gmail.com

the

Sheriff's

Deed

of

Conveyance in the day book of the Register

bought the property. Thus, Dehlma is a


of Deeds under Act. 3344 and obtained a

purchaser in good faith (Mathay v. CA,

tax declaration in its name.

G.R. No. 115788, 17 Sept, 1998).

(A). Was Dehlma a purchaser in good faith?

(B). Who as between Dehlma and XYZ Bank

(2%)

has a better right to the house and lot? (2%)

SUGGESTED ANSWER:

SUGGESTED ANSWER:

Yes, Dehlma is a purchaser in good faith.

Between Dehlma and XYZ Bank, Dehlma

In

Dehlma

has a better right to the house and lot.

bought the property, she went to the

After the release of the mortgage, the

Register of Deeds to verify Juliet's title.

Deed of Absolute Sale was registered and

When she discovered that the property

a new title was issued in Dehlma's name.

was mortgaged to Elaine, she gave an

Act 3344 is applicable exclusively to

advance payment so that Juliet could

instruments resulting from agreement of

release the mortgage. It was only after

parties thereto and does not apply to

the mortgage was released and free from

deeds of a sheriff conveying to a

the

present

case,

before

the claims of other persons that Dehlma


Never Let The Odds Keep You From Pursuing What You Know In Your Heart You Were Meant To Do.Leroy Satchel Paige

Page 83 of 180

Civil Law Q&As (2007-2013)

hectorchristopher@yahoo.com

dbaratbateladot@gmail.com

manual containing a summary of the


purchaser unregistered lands sold to him
under execution (Williams v. Suer, 49
Phil. ,534).
(C). Who owns the movables inside the
house? (2%)
SUGGESTED ANSWER:
Dehlma

owns

the

movables

because

when she acquired the house and lot


from

Juliet,

all

the

furniture

and

appliances therein were included in the


sale. As owner of the real property,
Dehlma also owns the movables found
therein (Art. 542, Civil Code).

Registration; Governing Law (2007)


No.IV.

Bedrock

Development

Land

Corp.

is

&
a

Property

development

company engaged in developing and selling


subdivisions,

condominium

units

and

industrial estates. In order to replenish its


inventories, it embarked on an aggressive
land banking program. It employed "scouts"
who roam all over the Philippines to look for
and conduct investigations on prospective
sites

for

acquisition

and

development,

whether developed, semi-developed or raw


land. The management of Bedrock asks you
as the company counsel to prepare a

pertinent laws and regulations relating to

land registration and acquisition of title to


land.

The

manual

should

include

the

following items:

ALTERNATIVE ANSWER:
In general, the governing law relating to
registration and acquisition of title to
land is Act 496 of 1902 as amended by

(A). What is the governing law? (5%)

PD 1529, otherwise known as Property


SUGGESTED ANSWER:
The

governing

law

Registration Decree of June 11, 1978.


is

the

Land

(1.)

Chapter

III-I

governs

original

Registration Act as amended by Property

registration

Registration Decree (Act 496 as amended

Torrens System by voluntary ordinary

by PD 1529).

judicial proceedings.

[Note: It is respectfully recommended

(2.) Chapter II-II governs compulsory

that full credit be given to examinees

registration of lands through cadastral

who did not give the exact title or

proceedings.

of

land

title

under

the

number of the law but merely stated a


description of the law.]
Never Let The Odds Keep You From Pursuing What You Know In Your Heart You Were Meant To Do.Leroy Satchel Paige

Page 84 of 180

Civil Law Q&As (2007-2013)

hectorchristopher@yahoo.com

(3.) Section 103 governs registration of


homestead, sales, free patent under CA
No. 141, as amended, otherwise known
as the Public Land Act.
(4.) Section 104 governs registration of
certificates

of

land

transfers,

emancipation patents and Certificates of


Land Ownership Award (CLOA) under
Comprehensive Land Reform Law.
(5.) Chapter V governs the registration of
land dealings on registered land like
conveyances,

transfers,

mortgages,

leases, powers of attorney, trusts and


similar contracts inter vivos.
(6.) Chapter V-II governs the registration
of involuntary dealings on registered
land like attachments, adverse claims,
enforcement of liens on registered land,
notices of lis pendens. (7.) Chapter VI
governs the registration of judgments,
orders and partitions, condemnation in
eminent domain proceedings, judicial
and extra-judicial settlement of estates.
(8.) Sections 107, 108 and 109 govern
petitions

and

registration

actions

after

original

like:

(a).Compulsory

surrender of withheld owner's duplicate


certificate of title;

(b)

Amendment

certificate of title;

dbaratbateladot@gmail.com

and

alteration

of

register a deed or when he is in doubt as


(c) Replacement of lost or destroyed

to what action to take on an instrument

owner's duplicate certificate of title.

presented for registration.

(9.)

R.A.

No.

reconstitution

26
of

governs
lost

or

judicial
destroyed

originals of the certificate of title.


(10.)

R.A.

No.

6732

Registration;

Party

Who

First

took

Possession (2013)

governs

No.IX.Rica petitioned for the annulment of

administrative reconstitution of lost or

her

destroyed original certificates of title.

Richard hired Atty. Cruz to represent him in

ten-year

old

marriage

to

Richard.

the proceedings. In payment for Atty. Cruz's


(11.)

Section

registration

of

113

governs

instruments

the

acceptance and legal fees, Richard conveyed

affecting

to Atty. Cruz a parcel of land in Taguig that

unregistered private lands.

he

recently

purchased

with

his

lotto

winnings. The transfer documents were


(12.) Section 117 governs "consultas,"

duly signed and Atty. Cruz

where the Register of Deeds refuses to

Never Let The Odds Keep You From Pursuing What You Know In Your Heart You Were Meant To Do.Leroy Satchel Paige

Page 85 of 180

Civil Law Q&As (2007-2013)

hectorchristopher@yahoo.com

dbaratbateladot@gmail.com

urged that the land itself is not the


immediately took possession by fencing off

object of the litigation, the annulment of

the property's entire perimeter.

marriage, if granted, will carry with it

Desperately needing money to pay for his


mounting legal fees and his other needs
and despite the transfer to Atty. Cruz,
Richard offered the same parcel of land for
sale to the spouses Garcia. After inspection
of the land, the spouses considered it a
good investment and purchased it from
Richard. Immediately after the sale, the
spouses

Garcia

commenced

the

construction of a three-story building over


the land, but they were prevented from
doing this by Atty. Cruz who claimed he has
a

better

right

in

light

of

the

prior

conveyance in his favor.


Is Atty. Cruz's claim correct? (8%)
SUGGESTED ANSWER:
No. Atty. Cruz is not correct. At first
glance, it may appear that Atty. Cruz is
the one who has the better right because
he first took possession of the property.
However, a lawyer is prohibited under
Art

1491

of

the

Civil

Code

from

acquiring the property and rights which


may be the object of any litigation in
which they may take part by virtue of
their profession. While the suit is for
annulment of marriage and it may be

property

of

the

community

or

the

absolute

conjugal property without the consent of

community or conjugal partnership of

the other spouse is void (Art 96 and Art

the spouses as the case may be (Art. 50

124, Family Code).

the

liquidation

of

the

in relation to Art 43 of the Family Code).


Richard purchased the land with his
lotto winnings during the pendency of
the suit for annulment and on the
assumption
governed

that

by the

community

parties

are

regime of absolute

in a 1 000-square meter property where he


grew

up

helping

his

father,

Michael,

cultivate the land. Michael has lived on the

winnings from gambling or betting will

property since the land was opened for

form part thereof. Also, since the land is

settlement

part

or

Commonwealth government in 1935, but

and

for some reason never secured any title to

Rica, it may not be sold or alienated

the property other than a tax declaration in

without the consent of the latter and

his name. He has held the property through

conjugal

the

conjugal

No.X. Manuel was born on 12 March 1940

partnership,

of

or

the

Registration; Requisites; Proof (2013)

absolute

partnership

community
of

Richard

at

about

the

time

of

the

any disposition or encumbrance of the


Never Let The Odds Keep You From Pursuing What You Know In Your Heart You Were Meant To Do.Leroy Satchel Paige

Page 86 of 180

Civil Law Q&As (2007-2013)

hectorchristopher@yahoo.com

has

been

dbaratbateladot@gmail.com

in

open,

the years in the concept of an owner and

notorious

his stay was uncontested by others. He has

thereof under bona fide claim of

also conscientiously and continuously paid


the realty taxes on the land.
Michael died in 2000 and Manuel - as
Michaels only son and heir -now wants to
secure and register title to the land in his
own name. He consults you for legal advice
as he wants to perfect his title to the land
and secure its registration in his name.
(A) What are the laws that you need to
consider in advising Manuel on how he can
perfect his title and register the land in his
name? Explain the relevance of these laws
to your projected course of action. (4%)
SUGGESTED ANSWER:
(Note:

With

all

due

respect,

it

is

recommended that the examiner accept and


give full credit to any of the answers given in
each of the following paragraphs.)
I

would

advice

Manuel

to

file

an

application for registration under Sec 14


of Pres. Decree No. 1529, or the Property
Registration Decree (PRD), specifically
Sec14 (1) which requires (a) that the land
applied for forms part of the alienable
and disposable (A & D) portion of the
public domain, and (b) that the applicant

possession

continuous
and

and

occupation

ownership

since

June

12,

1945,

or

earlier. However, it is only necessary


that the land is already declared A & D
land at the time for application for
registration

is

filed

(Malabanan

v.

Republic, G.R. No. 180067, June 30,


2009).

confirmation of imperfect or incomplete


title

through

judicial

legalization

under Sec. 48 (b) of CA no. 141, or the


Public Land Act (PLA). But, as held in
Malabanan,

there

is

no

substantial

difference between this provision and


Sec 14 (1) of the PRD. Both refer to

Manuel could also invoke Sec 14 (2) of


the

Manuel could also file an application for

same

Decree,

which

allows

registration through ordinary acquisitive


prescription for thirty years, provided,
however, that the land is patrimonial
in character, i.e. already declared by the
government (a) as A & D, and (b) no
longer needed for public use or public
service (Malabanan, supra).

agricultural lands already classified as


alienable and disposable at the time the
application

is

filed,

and

require

possession and occupation since June


12, 1945. The only difference is that
under the PRD, there already exists a
title which is to be confirmed, whereas
under the PLA, the presumption is that
land is still public land (Republic v.

Never Let The Odds Keep You From Pursuing What You Know In Your Heart You Were Meant To Do.Leroy Satchel Paige

Page 87 of 180

Civil Law Q&As (2007-2013)

hectorchristopher@yahoo.com

dbaratbateladot@gmail.com

(B) What do you have to prove to secure


Aquino, G.R. No. L-33983, January 27,
1983).
Manuel may also invoke vested rights
acquired under Rep. Act. No. 1942, dated
June 2, 1957, which amended Sec. 48 (b)
of the PLA by providing for a prescriptive
period

of

thirty

years

for

judicial

confirmation of imperfect title. It must


only be demonstrated that possession
and occupation commenced on January
24, 1947 and the 30-year period was
completed prior to the effectivity of PD
No. 1073 on January 25, 1977. PD No.
1073

now

occupation

requires
since

possession
June

12,

and
1945

(Republic v. Espinosa, G.R. No. 171514,


July 18, 2012).
Another alternative is for Manuel to
secure

title

through

administrative

proceedings under the homestead or free


patent provisions of the PLA. The title
issued has the same efficacy and validity
as

title

issued

through

judicial

proceedings, but with the limitations


that the land cannot be sold or disposed
of within five years from the issuance of
patent

(Sec.

amended).

118,

CA

No.

141,

as

Manuel's

objectives

and

documentation are necessary? (4%)


SUGGESTED ANSWER:

what

valuable improvements like fencing the


Manuel has a the burden to overcome

land, constructing a residential house

the presumption of State ownership by

thereon,

well-nigh

planting fruit bearing trees, declaring

(Ong

v.

incontrovertible
Republic,

G.R.

evidence

No.

175746,

the

land

cultivating

for

the

taxation

land

purposes

and

and

March 12, 2008). Accordingly, he must

paying realty taxes, all of which are

show that ht eland is already classified

corroborative proof of possession.

as A & D at the time the application for


registration is filed and that he has
been

in

possession

and

occupation

thereof in the manner required by law


since June 12, 1945, or earlier.

To identify the land, he must submit the


tracing cloth plan or a duly-certified
blueprint

or whiteprint

copy

thereof

(Director of Lands v. Reyes, G.R. No. L27594, November 28, 1975; Director of

Manuel may tack his possession to that

Lands v. CA and Iglesia ni Cristo, G.R.

of his predecessor-in-interest (Michael)

No. L-56613, March 14, 1988).

by the testimony of disinterested and


knowledgeable eyewitnesses. Overt acts
of possession may consist in introducing

To show the classification of the land as


A

&

D,

the

application

must

be

accompanied by (1) a CENRO or PENRO

Never Let The Odds Keep You From Pursuing What You Know In Your Heart You Were Meant To Do.Leroy Satchel Paige

Page 88 of 180

Civil Law Q&As (2007-2013)

hectorchristopher@yahoo.com

certification; and (2) a certified true


copy

of

approved

the
by

original
the

classification

DENR

Secretary

(Republic v. Bantigue, G.R.No. 162322,


March

14,

2012).

presidential

or

legislative act may also be considered.

Remedies; Fraud; Rights

of Innocent

Purchaser (2009)
No.IX. Before migrating to Canada in 1992,
the spouses Teodoro and Anita entrusted
all their legal papers and documents to
their nephew, Atty. Tan. Taking advantage
of the situation, Atty. Tan forged a deed of
sale, making it appear that he had bought
the couples property in Quezon City. In
2000, he succeeded in obtaining a TCT over
the property in his name. Subsequently,
Atty. Tan sold the same property to Luis,
who built an auto repair shop on the
property. In 2004, Luis registered the deed
of conveyance, and title over the property
was transferred in his name.
In 2006, the spouses Teodoro and Anita
came to the Philippines for a visit and
discovered what had happened to their
property. They immediately hire you as
lawyer. What action or actions will you
institute in order to vindicate their rights?
Explain fully. (4%)

dbaratbateladot@gmail.com

SUGGESTED ANSWER:

(c). A criminal action for forgery or


falsification of public document;
I will institute the following actions
(d).

against Atty. Tan:

complaint

with

the

Supreme

Court/Integrated Bar of the Philippines


(a). A civil action for damage for the

to

disbar

or

suspend

him

or

other

fraudulent transfer of the title in his

disciplinary action for violation or the

name and to recover the value of the

Code of Professional Ethics.

property;
Any action against Luis will not prosper
National

because he is an innocent purchaser for

Treasurer for compensation from the

value. The Title to the land he bought

State Assurance Fund which is set aside

was already in the name of the person

by law to pay those who lose their land

who sold the property to him, and there

suffer damages as a consequence of the

is nothing on the title which will make

operation of the Torrens system;

him suspect about the fraud committed

(b).

An

action

against

the

by Atty. Tan.

Never Let The Odds Keep You From Pursuing What You Know In Your Heart You Were Meant To Do.Leroy Satchel Paige

Page 89 of 180

Civil Law Q&As (2007-2013)

hectorchristopher@yahoo.com

Contracts

dbaratbateladot@gmail.com

Rescission of Contract; Fortuitous Event


(2008)

Contract to Sell vs. Conditional Contract


No.XVIII. AB Corp. entered into a contract

of Sale (2012)

with XY Corp. whereby the former agreed to

No.X.a) A contract to sell is the same as a

construct

the

research

and

laboratory

conditional contract of sale. Do you agree?

facilities of the latter. Under the terms of

Explain your answer. (5%)

the contract, AB Corp. agreed to complete


the facility in 18 months, at the total

SUGGESTED ANSWER:

contract price of P10 million. XY Corp. paid

No. A contract to sell is a species of


conditional sale. The contract to sell
does not sell a thing or property; it sells
the right to buy property. A conditional

50% of the total contract price, the balance


to be paid upon completion of the work. The
work stated immediately, but AB Corp. later
experienced work slippage because of labor
unrest

in

his

company.

AB

Corp.'s

sale is a sale subject to the happening or

employees claimed that they are not being

performance of a condition, such as

paid on time; hence, the work slowdown. As

payment of the full purchase price, or

of the 17th month, work was only 45%

the performance of other prestation to

completed. AB Corp. asked for extension of

give, to do or not to do. Compliance with

time, claiming that its labor problems is a

the condition automatically gives the

case of fortuitous event, but this was

right to the vendee to demand the


delivery of the object of the sale. In a
contract to sell, however, the compliance
with

the

condition

does

not

denied by XY Corp. When it became certain


that the contruction could not be finished
on time, XY Corp. sent written notice
cancelling the contract, and requiring AB
Corp. to immediately vacate the premises.

automatically sell the property to the


vendee. It merely gives the vendee the

(A). Can the labor unrest be considered a

right to compel the vendor to execute

fortuitous event? (1%)

the deed of absolute sale.

SUGGESTED ANSWER:
No.

The

labor

unrest

cannot

be

considered a fortuitous event under Art.

1174 of the Civil Code. A fortuitous


event should occur independent of the
Never Let The Odds Keep You From Pursuing What You Know In Your Heart You Were Meant To Do.Leroy Satchel Paige

Page 90 of 180

Civil Law Q&As (2007-2013)

hectorchristopher@yahoo.com

dbaratbateladot@gmail.com

No. XI. TRUE or FALSE. Answer TRUE if


will

of

the

debtor

or

without

his

participation or aggravation (Paras, Civil


Code Annotated, vol. IV, 2000 ed., p
159). As mentioned in the facts, labor
unrest of the employees was caused by
AB Corp.'s failure to pay its employees
on time.
(B).

Can

XY

Corp.

unilaterrally

and

immediately cancel the contract? (2%)


SUGGESTED ANSWER:
No, XY Corp. cannot unilaterally and
immediately cancel the contract. In the
absence of any stipulation for automatic
rescission, rescission must be judicial
(Art. 1191, Civil Code).
(C).

Must

AB

Corp.

return

the

50%

downpayment? (2%)
SUGGESTED ANSWER:
AB Corp. need not return the 50% down
payment because 45% of the work was
already completed, otherwise, XY Corp.
would be unjustly enriching itself at the
expense of AB Corp.

Stipulation; Arbitration Clause (2009)

the statement is true, or FALSE if the

No.XV. Sarah had a deposit in a savings


statement is false. Explain your answer in
not more than two (2) sentences.

account with Filipino Universal Bank in the


amount

of

five

million

pesos

(P5,000,000.00). To buy a new car, she


(A). A clause in an arbitration contract

obtained a loan from the same bank in the

granting one of the parties the power to

amount of P1,200,000.00, payable in twelve

choose more arbitrators than the other

monthly installments. Sarah issued in favor

renders the arbitration contract void. (1%)

of the bank post-dated checks, each in the


amount of P100,000.00, to cover the twelve

SUGGESTED ANSWER:

monthly installment payments. On the

True. The Civil Code provides that Any

third,

clause giving one of the parties power to

corresponding checks bounced.

fourth

and

fifth

months,

the

choose more arbitrators than the other


is void and of no effect (Art 2045, NCC).

Obligations
Extinguishment; Compensation (2009)

The

bank

then

declared

the

whole

obligation due, and proceeded to deduct the


amount of one million pesos
(P1,000,000.00) from Sarahs deposit after
notice to her that this is a form of

Never Let The Odds Keep You From Pursuing What You Know In Your Heart You Were Meant To Do.Leroy Satchel Paige

Page 91 of 180

Civil Law Q&As (2007-2013)

hectorchristopher@yahoo.com

compensation allowed by law. Is the bank

dbaratbateladot@gmail.com

Extinguishment; Compensation (2008)

correct? Explain. (4%)


No. XV. Eduardo was granted a loan by XYZ
SUGGESTED ANSWER:

Bank for the purpose of improving a

No, the bank is not correct. While the

building

Bank is correct about the applicability of

Eduardo, executed the promissory note

compensation, it was not correct as to

("PN") in favor of the bank, with his friend

which

XYZ

leased

from

him.

Recardo as co-signatory. In the PN, they

the amount compensated.

both

acknowledged

that

they

are

A bank deposit is a contract of loan,

"individually and collectively" liable and

where the depositor is the creditor and

waived the need for prior demand. To

the bank the debtor. Since Sarah is also


the debtor of the bank with respect to
the loan, both are mutually principal
debtors and creditors of each other. Both
obligation

are

due,

demandable

and

secure the PN, Recardo executed a real


estate mortgage on his own property. When
Eduardo defaulted on the PN, XYZ stopped
payment of rentals on the building on the
ground that legal compensation had set in.
Since there was still a balance due on the

liquidated but only up to the extent of

PN

P300,000.00 (covering the unpaid third,

foreclosed the real estate mortgage over

fourth and fifth monthly installments).

Recardo's property. Recardo opposed the

The entire one million was not yet due

foreclosure on the ground that he is only a

because the loan has no acceleration

co-signatory; that no demand was made

clause in case of default. And since there

upon him for payment, and assuming he is

is

no

retention

commenced

by

or

third

controversy
person

and

communicated in due time to the debtor,


then

all

the

requisites

of

legal

compensation are present but only up to

after

applying

the

rentals,

XYZ

liable, his liability should not go beyond


half the balance of the loan. Further,
Recardo said that when the bank invoked
compensation between the reantals and the
amount of the loan, it amounted to a new
contract or novation, and had the effect of

the amount of P300,000.00. The bank,

extinguishing the security since he did not

therefore, may deduct P300,000.00 from

give his consent (as owner of the property

Sarahs

under the real estate mortgage) thereto.

bank

compensation.

deposit

by

way

of

(A). Can XYZ Bank validly assert legal


compensation? (2%)

Never Let The Odds Keep You From Pursuing What You Know In Your Heart You Were Meant To Do.Leroy Satchel Paige

Page 92 of 180

Civil Law Q&As (2007-2013)

hectorchristopher@yahoo.com

dbaratbateladot@gmail.com

estate mortgage on his own property. When


Eduardo defaulted on the PN, XYZ stopped

SUGGESTED ANSWER:

payment of rentals on the building on the

Yes, XYZ Bank can validly assert legal


compensation. In the present case, all of
the elements of legal compensation are
present: (1) XYZ Bank is the creditor of
Eduardo while Eduardo is the lessor of
XYZ Bank; (2) both debts consist in a
sum of money, or if the things due are
consumable, they be of the same kind,
and also of the same quality if the latter
has been stated; (3) the two debts be
due;

(4)

they

be

liquidated

and

demandable, and (5) over neither of them


there be any retention or controversy,
commenced

by

third

persons

and

communicated in due time to the debtor


(Art. 1279, Civil Code).

Extinguishment; Novation (2008)


No. XV. Eduardo was granted a loan by XYZ
Bank for the purpose of improving a
building

which

XYZ

leased

from

him.

Eduardo, executed the promissory note


("PN") in favor of the bank, with his friend
Recardo as co-signatory. In the PN, they
both

acknowledged

that

they

are

"individually and collectively" liable and


waived the need for prior demand. To
secure the PN, Recardo executed a real

ground that legal compensation had set in.


Since there was still a balance due on the
PN

after

applying

the

rentals,

XYZ

foreclosed the real estate mortgage over

(C). Does Recardo have basis under the


Civil Code for claiming that the original
contract was novated? (2%)
SUGGESTED ANSWER:

Recardo's property. Recardo opposed the


foreclosure on the ground that he is only a

No. Recardo has no basis for claiming

co-signatory; that no demand was made

novation of the original contract when

upon him for payment, and assuming he is

the bank invoked compensation because

liable, his liability should not go beyond

there was simply partial compensation

half the balance of the loan. Further,

(Art. 1290, Civil Code) and this would

Recardo said that when the bank invoked


compensation between the reantals and the
amount of the loan, it amounted to a new
contract or novation, and had the effect of

not bar the bank from recovering the


remaining balance of the obligation.
ALTERNATIVE ANSWER:

extinguishing the security since he did not


give his consent (as owner of the property

No. In order that an obligation may be

under the real estate mortgage) thereto.

extinguished by another, it is imperative

Never Let The Odds Keep You From Pursuing What You Know In Your Heart You Were Meant To Do.Leroy Satchel Paige

Page 93 of 180

Civil Law Q&As (2007-2013)

hectorchristopher@yahoo.com

dbaratbateladot@gmail.com

debt is not a valid tender of payment and


that it be so declared in unequivocal
terms,

or

that

the

old

and

new

obligations be on every point compatible


with

each

other.

Novation

is

never

presumed (Art. 1292, Civil Code).

Extinguishment;

Payment

of

Check

(2013)
No.VI. Lito obtained a loan of P1,000,000
from Ferdie, payable within one year. To
secure payment, Lito executed a chattel
mortgage on a Toyota Avanza and a real
estate mortgage on a 200-square meter
piece of property.
(B) Lito's failure to pay led to the extrajudicial foreclosure of the mortgaged real
property. Within a year from foreclosure,
Lito tendered a manager's check to Ferdie to
redeem the property. Ferdie refused to
accept payment on the ground that he
wanted payment in cash: the check does
not qualify as legal tender and does not
include the interest payment. Is Ferdie's
refusal justified? (4%)
SUGGESTED ANSWER:
A check, whether a managers check or
an ordinary check is not legal tender,
and an offer of a check in payment of a

may be refused receipt by the oblige

is sufficient to compel redemption but is


or creditors (Philippine Airlines v. CA

not in itself a payment that relieves the

and

L-49188,

redemptioner from his liability to pay

1990). Mere delivery of checks does not

the redemption price (Biana v. Gimenez,

discharge

G.R. No. 132768, Sept 9, 2005, citing

Amelia

Tan,

the

G.R.

No.

obligation

under

judgment. A check shall produce the


effect of payment only when they have
been cashed or where through the fault
of the creditor they have been impaired
(Art 1249, Civil Code).

Fortunado v. CA).
Redemption within the period allowed by
law is not a matter of intent but a
question of payment or valid tender of
full redemption prices within the said

However, it is not necessary that the

period. Whether redemption is being

right of redemption be exercised by

made under Art. 3135 or under the

delivery of legal tender. A check may be

General Banking Law, the mortgagor or

used

his

for

the

exercise

of

right

of

assignee

is

required

to

tender

redemption, the same being a right and

payment to make said redemption valid

not an obligation. The tender of a check

(Heirs of Quisumbing v. PNB and SLDC,


G.R. No. 178242, Jan 20, 2009).

Never Let The Odds Keep You From Pursuing What You Know In Your Heart You Were Meant To Do.Leroy Satchel Paige

Page 94 of 180

Civil Law Q&As (2007-2013)

hectorchristopher@yahoo.com

dbaratbateladot@gmail.com

had become stale. Gustavo now wants


Moreover, Ferdies refusal was justified
on the ground that the amount tendered
does not include interest. In order to
effect the redemption of the foreclosed
property, the payment to the purchaser
must include the following sums: (a) the
bid price; (b) the interest on the bid
price, computed at one per centum (1%)
per month; and (c) the assessments and
taxes, if any, paid by the purchaser with
the same rate of interest (Sec 28, 1997
Rules of Civil Procedure). Unless there is
an express stipulation to that effect, the
creditor cannot be compelled to receive
partial payment of the prestation (Art.
1248, Civil Code).

Extinguishment;

Payment

of

Check;

Legal Tender (2008)


No.

XVII.

Felipe

borrowed

$100

from

Gustavo in 1998, when the Phil P - US$


exchange rate was P56 - US$1. On March
1, 2008, Felipe tendered to Gustavo a
cashier's check in the amount of P4,135 in
payment of his US$ 100 debt, based on the
Phil P - US$ exchange rat at that time.
Gustavo accepted the check, but forgot to
deposit it until Sept. 12, 2008. His bank
refused to accepted the check because it

Felipe to pay him in cash the amount of


P5,600.

Claiming

that

the

previous

payment was not in legal tender, and that

check became stale. He is now estopped


there

has

been

extraordinary

deflation

since 1998, and therefore, Felipe should


pay him the value of the debt at the time it
was incurred. Felipe refused to pay him
again, claiming that Gustavo is estopped
from raising the issue of legal tender,
having accepted the check in March, and

from raising the issue that a cashier's


check is not legal tender.
(B). Can Felipe validly refuse to pay Gustavo
again? (2%)
SUGGESTED ANSWER:

that it was Gustavo's negligence in not

Yes, Felipe can refuse to pay Gustavo,

depositing

who allowed the check to become stale.

the

check

immediately

that

caused the check to become stale.


(A). Can Gustavo now raised the issue that
the cashier's check is not legal tender? (2%)

Although a check is not legal tender


(Belisario v. Natividad. 60 Phil 156),
there

are

instances

when

check

produces the effects of payment, for


SUGGESTED ANSWER:
No. Gustavo previously accepted a check
as payment. It was his fault why the

example: (a) when the creditor is in


estoppel or he had previously promised
he would accept a check (Paras, Civil

Never Let The Odds Keep You From Pursuing What You Know In Your Heart You Were Meant To Do.Leroy Satchel Paige

Page 95 of 180

Civil Law Q&As (2007-2013)

hectorchristopher@yahoo.com

both
Code Annotated, Vol IV, 2000 ed., p.
394); (b) when the check has lost its
value because of the fault of the creditor
(Art. 1249, 2nd par.),as when he was
unreasonably delayed in presenting the
check for payment (PNB v. Seeto, G.R.
No, L-4388, 13 August 1952).
(C). Can Felipe compel Gustavo to receive
US$100 instead? (1%)
SUGGESTED ANSWER:
Felipe cannot compel Gustavo to receive
US$100 because under RA 529, payment
of loans should be at Philippine currency
at the rate of exchange prevailing at the
time of the stipulated date of payment.
Felipe could only compel Gustavo to
receive US$ 100 if they stipulated that
obligation be paid in foreign currency
(R.A. 4100).

Liability; Solidary Liability (2008)


No. XV. Eduardo was granted a loan by XYZ
Bank for the purpose of improving a
building

which

XYZ

leased

from

him.

Eduardo, executed the promissory note


("PN") in favor of the bank, with his friend
Recardo as co-signatory. In the PN, they

dbaratbateladot@gmail.com

acknowledged

that

they

are

"individually and collectively" liable and


waived the need for prior demand. To

contract or novation, and had the effect of


secure the PN, Recardo executed a real
estate mortgage on his own property. When
Eduardo defaulted on the PN, XYZ stopped
payment of rentals on the building on the
ground that legal compensation had set in.
Since there was still a balance due on the
PN

after

applying

the

rentals,

XYZ

extinguishing the security since he did not


give his consent (as owner of the property
under the real estate mortgage) thereto.
(B). Can Recardo's property be foreclosed to
pay the full balance of the loan? (2%)
SUGGESTED ANSWER:

foreclosed the real estate mortgage over


Recardo's property. Recardo opposed the

Yes, Recardo's property can be foreclosed

foreclosure on the ground that he is only a

to pay the full balance of the loan

co-signatory; that no demand was made

because when he signed as co-signatory

upon him for payment, and assuming he is

in the promissory note, he acknowledged

liable, his liability should not go beyond

he is solidarily liable with Eduardo. In

half the balance of the loan. Further,


Recardo said that when the bank invoked
compensation between the reantals and the
amount of the loan, it amounted to a new

solidary obligations, a creditor has the


right to demand full payment of the
obligation from any of

Never Let The Odds Keep You From Pursuing What You Know In Your Heart You Were Meant To Do.Leroy Satchel Paige

Page 96 of 180

Civil Law Q&As (2007-2013)

hectorchristopher@yahoo.com

takes

charge

dbaratbateladot@gmail.com

of

the

agency

or

the solidary debtors (Art. 1207, Civil

management of the business or property

Code).

of another without any power from the

Obligations; Without Agreement (2007)


No.V. What are obligations without an
agreement"?
situations

Give

giving

five
rise

to

examples

of

this

of

type

obligations? (10%)
SUGGESTED ANSWER:
"Obligations without an agreement" are
obligations

that

do

not

arise

from

contract such as those arising from: 1.


delicts;

2.

quasi-delicts;

3.

solutio

indebiti; 4. negotiorum gestio; and 5. all


other obligations arising from law.
ALTERNATIVE ANSWER:
"Obligations without an agreement" refer
to the juridical relation of quasi-contract
which

arise

from

certain

lawful,

voluntary and unilateral acts to the end


that no one shall be unjustly enriched or
benefited at the expense of another. (Art.
2142, NCC)
First Example of an obligation without
an agreement is a case of negotiorum
gestio,

whereby

one

who

voluntarily

support, it is given by a stranger, the


latter, is obliged to continue the same

latter shall have a right to claim the

until the termination of the affair and its

same from the former, unless it appears

incidents,

or

person

that he gave it out of piety and without

concerned

to substitute him, if the

intention of being repaid (Art. 2164,

to

require

the

owner is in a position to do so (Art.


2144, NCC).

NCC).
Fourth

example,

is

when

through

solutio

accident or other causes a person is

to

an

injured or becomes seriously ill, and he

obligation without an agreement. This

is treated or helped while he is not in a

refers to the obligation to return which

condition to give consent to a contract,

arises when something is received when

he shall be liable to pay for the services

there is no right to demand it, and it was

of the physician or other person aiding

unduly delivered through mistake (Art.

him,

2154, NCC).

rendered out of pure generosity (Art.

Second

example,

indebiti

may

also

case
give

of
rise

unless

the

service

has

been

2167, NCC).
Third example, is when without the
knowledge of the person obliged to give
Never Let The Odds Keep You From Pursuing What You Know In Your Heart You Were Meant To Do.Leroy Satchel Paige

Page 97 of 180

Civil Law Q&As (2007-2013)

hectorchristopher@yahoo.com

dbaratbateladot@gmail.com

a trust against one who, by fraud, duress


Fifth instance of an obligation without

or abuse of confidence, obtains or holds

an agreement is when the person obliged

the legal right to property which he

to support an orphan or an insane or


other indigent person unjustly refuses to
give support to the latter, any third
person may furnish support to the needy
individual, with right of reimbursement
from the person obliged to give support.
The provisions of this article apply when
the father or mother of a child under
eighteen years of age unjustly refuses to
support him (Art. 2166, NCC).

Trust
Trust De Son Tort (2007)
No.III. Explain the following concepts and
doctrines and give an example of each:
trust de
(A).
concept
of
tort (constructive trust) (5%)

son

SUGGESTED ANSWER:
A constructive trust is a trust NOT
created by any word or phrase, either
expressly or impliedly, evincing a direct
intention to create a trust, but is one
that

arises

in

order

to

satisfy

the

demands of justice. It does not come


about by agreement or intention but
mainly operation of law and construed as

provides: "If an absolute conveyance of


good

property is made in order to secure the

conscience, to hold (Heirs of Lorenzo Yap

performance of an obligation of the

v. CA, 371 Phil 523, 1991). The following

grantor toward the grantee, a trust by

are examples of constructive trust: 1.

virtue

Art.

"If

fulfillment of the obligation is offered by

property is acquired through mistake or

the grantor when it becomes due, he

fraud, the person obtaining it is, by force

may demand the reconveyance of the

of law considered a trustee of an implied

property to him." 4. Art 1455 NCC which

trust for the benefit of the person for

provides: "When any trustee, guardian or

whom the property comes." 2. Art 1451

any

NCC which provides: "When land passes

relationship uses trust funds for the

by succession through any person and he

purchase

causes the legal title to be put in the

conveyance to be made to him or to

name of another, a trust is established

third person, a trust is established by

by implication of law for the benefit of

operation of law in favor of the person to

the true owner." 3. Art 1454 NCC which

whom the funds belong."

ought

not,

1456

in

NCC

equity

which

and

provides:

of

law

person

of

is

established.

holding

property

and

If

the

fiduciary

causes

Never Let The Odds Keep You From Pursuing What You Know In Your Heart You Were Meant To Do.Leroy Satchel Paige

Page 98 of 180

Civil Law Q&As (2007-2013)

hectorchristopher@yahoo.com

business of buying and selling large cattle.

Sales
Condominium

Act;

In order to secure the financial capital, she

Partition

of

is

Condominium (2009)
No.XVIII.

The

Ifugao

Arms

condominium project in Baguio City. A


strong earthquake occurred which left huge
cracks in the outer walls of the building. As
a result, a number of condominium units
were rendered unfit for use. May Edwin,
owner of one of the condominium units
affected, legally sue for partition by sale of
the whole project? Explain. (4%)
SUGGESTED ANSWER:
Yes, Edwin may legally sue for partition
by

sale

of

the

whole

condominium

project under the following conditions:


(a) the damage or destruction caused by
the earthquake has rendered one-half
(1/2)

or

more

of

the

untenantable,

and

condominium

owners

(b)

units

therein

that
holding

the
an

aggregate of more than thirty percent


(30%) interests of the common areas are
opposed

to

the

dbaratbateladot@gmail.com

No.VI. (b) Eulalia was engaged in the

restoration

of

the

condominium project (Sec 8 [b], Republic


Act No. 472 Condominium Act).

Mortgage; Equitable Mortgage (2012)

mortgage for the purpose of securing the


advanced for her employees (biyaheros). She
required them to surrender TCT of their
properties and to execute the corresponding
Deeds

of

Sale

in

her

favor.

Domeng

Bandong was not required to post any


security but when Eulalia discovered that
he incurred shortage in cattle procurement
operation, he was required to execute a

shortage

incurred

by

Domeng

in

the

amount of P 70, 000.00 while employed as


"biyahero" by Eulalia. Was the Deed of Sale
between Domeng and Eulalia a contract of
sale or an equitable mortgage? Explain.
(5%)
SUGGESTED ANSWER:

Deed of Sale over a parcel of land in favor of

The contract between Domeng Bandong

Eulalia. She sold the property to her

and Eulalia was an equitable mortgage

grandneice

thereafter

rather than a contract of sale. The

instituted an action for ejectment against

purported deed of sale was actually

Jocelyn

who

the Spouses Bandong.

intended to merely secure the payment

To assert their right, Spouses Bandong filed

of the shortage incurred by Domeng in

an action for annulment of sale against

the

Eulalia and Jocelyn alleging that there was

operations. Under Art 1602, Civil Code,

no

sale

intended

but

only

conduct

of

the

cattle-buying

equitable

Never Let The Odds Keep You From Pursuing What You Know In Your Heart You Were Meant To Do.Leroy Satchel Paige

Page 99 of 180

Civil Law Q&As (2007-2013)

hectorchristopher@yahoo.com

the contract shall be presumed to be an


equitable mortgage when it may be fairly
inferred that the real intention of the
parties is simply to secure the payment
of a debt or the performance of any
other obligation. The present transaction
was clearly intended to just secure the
shortage incurred by Eulalia because
Bandung remained in possession of the
property inspite of the execution of the
sale.

Option Contract; Liquor & Pulutan as


Consideration (2013)
No.III.Sergio is the registered owner of a
500-square meter land. His friend, Marcelo,
who

has

long

been

interested

in

the

property, succeeded in persuading Sergio to


sell it to him. On June 2, 2012, they agreed
on the purchase price of P600,000 and that
Sergio would give Marcelo up to June30,
2012 within which to raise the amount.
Marcelo, in a light tone usual between
them, said that they should seal their
agreement through a case of Jack Daniels
Black and P5,000 "pulutan" money which
he immediately handed to Sergio and which
the latter accepted. The friends then sat
down and drank the first bottle from the
case of bourbon.

dbaratbateladot@gmail.com

On June 15, 2013, Sergio learned of


another buyer, Roberto, who was

Yes. Marcelo has a cause of action


offering P800,000 in ready cash for the

against Sergio.

land. When Roberto confirmed that he


could pay in cash as soon as Sergio could

Under Art. 1324, when the offerer has

get

Sergio

allowed the offeree a certain period to

decided to withdraw his offer to Marcelo,

accept, the offer may be withdrawn at

the

documentation

ready,

hoping to just explain matters to his friend.


Marcelo,
withdrawal

however,

objected

when

the

was communicated to him,

taking the position that they have a firm


and binding agreement that Sergio cannot
simply walk away from because he has an
option to buy that is duly supported by a
duly accepted valuable consideration.

any

time

before

acceptance

by

communicating such withdrawal, except


when

the

option

consideration,

as

is

founded

something

upon

paid

or

promised.
An accepted unilateral promise to buy or
sell a determinate thing for a price

(A) Does Marcelo have a cause of action

certain

against Sergio? (5%)

promise is supported by a consideration

SUGGESTED ANSWER:

is binding upon him if the

distinct from the price (Art. 1479).

Never Let The Odds Keep You From Pursuing What You Know In Your Heart You Were Meant To Do.Leroy Satchel Paige

Page 100 of 180

Civil Law Q&As (2007-2013)

hectorchristopher@yahoo.com

dbaratbateladot@gmail.com

of sale for the real property. Therefore,


Consideration in an option contract may
be anything of value,, unlike in sale
where it must be the price certain in
money or its equivalent (San Miguel
Properties Inc. v. Spouses Huang, G.R.
No. 137290, July 31, 2000).
Here, the case of Jack Daniels Black and
the P5,000.00 pulutan money was a
consideration to seal their agreement,
an agreement that Marcelo is given until
June 30, 2012 to buy the parcel of land.
There is also no showing that such
consideration will be considered part of
the

purchase

unilateral

price.

withdrawal

Thus,
of

Sergios

the

offer

violated the Option Contract between


him and Marcelo.
(B) Can Sergio claim that whatever they
might have agreed upon cannot be enforced
because any agreement relating to the sale
of real property must be supported by
evidence in writing and they never reduced
their agreement to writing? (3%)
SUGGESTED ANSWER:
No. Sergios claim has no legal basis.
The contract at issue in the present case
is the option contract, not the contract

Art. 1403 does not apply.

ALTERNATIVE ANSWER:
The

Statute

of

Frauds

covers

an

agreement for the sale of real property

No. Sergios claim has no legal basis.

Such

The contract of sale has already been

agreement is unenforceable by action,

partially executed which takes it outside

unless

or

the ambit of the Statute of Frauds is

memorandum, thereof, be in writing,

applicable only to executory contracts,

(Art. 1403 (e), Civil Code). Here, Marcelo

not to contracts that are totally or

and

partially performed (Carbonnel v. Poncio,

or

of

an

the

Sergio

Option

interest

same,

or

merely

Contract,

therein.

some

entered

which

note

into

refers

an

to

G.R. No. L-11231, May 12, 1958).

unilateral promise to buy or sell, which


need not be in writing to be enforceable
(Sanchez v. Rigos, G.R. No. L-25494,

Right of First Refusal; Lessee; Effect

June 14, 1972, citing Atkins, Kroll and

(2008)

Co.

Inc.

v.

Cua

Hian

Tek

and

Southwestern Sugar & Molasses Co. v.


Atlantic Gulf & Pacific Co.).

No.XVI. Dux leased his house to Iris for a


period of 2 years, at the rate of P25,000.00

Never Let The Odds Keep You From Pursuing What You Know In Your Heart You Were Meant To Do.Leroy Satchel Paige

Page 101 of 180

Civil Law Q&As (2007-2013)

hectorchristopher@yahoo.com

dbaratbateladot@gmail.com

that the mother is not a third party, this


monthly, payable annually in advance. The

would make her privy to the agreement

contract stipulated that it may be renewed

of Dux and Iris, aware of the right of first

for another 2-year period upon mutual


agreement of the parties. The contract also
granted Iris the right of first refusal to
purchase the property at any time during
the lease, if Dux decides to sell the property
at the same price that the property is
offered for sale to a third party. Twentythree months after execution of the lease
contract, Dux sold breach of her right of
first refusal. Dux said there was no breach
because the property was sold to his
mother who is not a third party. Iris filed an
action to rescind the sale and to compel
Dux to sell the property to her at the same
price. Alternatively, she asked the court to
extend the lease for another 2 years on the
same terms.
(A). Can Iris seek rescission of the sale of
the property to Dux's mother? (3%)
SUGGESTED ANSWER:
Yes, because the right of first refusal is
included in the contract signed by the
parties. Only if the lessee failed to
exercise the right of first refusal could
the

lessor

lawfully

sell

the

subject

property to others, under no less than


the

same

terms

and

conditions

previously offered to the lessee. Granting

refusal. This makes the mother a buyer


in bad faith, hence giving more ground
for

rescission

(Equatorial

of

Realty,

the
et

sale
al.

v.

to

her

Mayfair

Theater, G.R. No. 106063, 21 Nov. 1996).

No. The contract stipulated that it may


be renewed for another 2-year period
upon mutual agreement of the parties.
Contracts

are

binding

between

the

parties; validity or compliance cannot be


left to the will of one of the parties (Art.

ALTERNATIVE ANSWER:

1308, Civil Code).

No, Iris cannot seek rescission of the

ALTERNATIVE ANSWER:

sale of the property to Duxs mother


because the sale is not one of those
rescissible contracts under Art. 1381 of
the Civil Code.

It depends. The alternative prayer for


the extension of the lease may prosper if
(a) there is a stipulation in the contract
of sale; (b) Dux's mother is aware of the

(B). Will the alternative prayer for extension

existing contract of lease; or (c) the lease

of the lease prosper? (2%)

is recorded in the Registry of Property

SUGGESTED ANSWER:

(Art. 1676, Civil Code).

Never Let The Odds Keep You From Pursuing What You Know In Your Heart You Were Meant To Do.Leroy Satchel Paige

Page 102 of 180

Civil Law Q&As (2007-2013)

hectorchristopher@yahoo.com

dbaratbateladot@gmail.com

(IVa) Resolve Boboy's claim that as a builder


in good faith, he should be

Lease
Builder;

Good

Faith;

Useful

Improvements (2013)
No.IV.Anselmo is the registered owner of a
land and a house that his friend Boboy
occupied for a nominal rental and on the
condition that Boboy would vacate the
property

on

demand.

With

Anselmo's

knowledge, Boboy introduced renovations


consisting of an additional bedroom, a
covered veranda, and a concrete block
fence, at his own expense.
Subsequently, Anselmo needed the property
as his residence and thus asked Boboy to
vacate and turn it over to him. Boboy,
despite an extension, failed to vacate the
property, forcing Anselmo to send him a
written demand to vacate.
In his own written reply, Boboy signified
that he was ready to leave but Anselmo
must first reimburse him the value of the
improvements

he

introduced

on

the

property as he is a builder in good faith.


Anselmo

refused,

insisting

that

Boboy

cannot ask for reimbursement as he is a


mere lessee. Boboy responded by removing
the improvements and leaving the building
in its original state.

As Boboy is a lessee of the property,


reimbursed the value of the improvements

even if he was paying nominal rental,

he introduced. (4%)

Art. 1678, Civil Code, is applicable.


Under this provision, if the lessee makes,

SUGGESTED ANSWER:

in

Boboys claim that he is a builder in


good faith has no basis. A builder in good
faith

is

someone

who

occupies

the

property in concept of an owner. The


provisions

on

builder-planter-sower

under the Civil Code cover cases in


which the builder, planter and sower
believe themselves to be owners of the
land, or at least, to have a claim of title
thereto.

good

faith,

useful

improvements

which are suitable to the use for which


the lease is intended, without altering
the form or substance of the property
leased, the lessor upon the termination
of the lease, shall pay the lessee one-half
of the value of improvements at that
time.

Should

the

lessor

refuse

to

reimburse said amount, the lessee may


remove the improvements, even though
the principal thing may suffer damage
thereby.

Never Let The Odds Keep You From Pursuing What You Know In Your Heart You Were Meant To Do.Leroy Satchel Paige

Page 103 of 180

Civil Law Q&As (2007-2013)

hectorchristopher@yahoo.com

dbaratbateladot@gmail.com

when the term of their lease shall have


(IVb) Can Boboy be held liable for damages
for

removing

the

improvements

over

Anselmo's objection? (4%)


SUGGESTED ANSWER:
No. Boboy cannot be held liable for
damages.
The

lessor,

Anselmo,

refused

to

reimburse one-half of the value of the


improvements, so the lessee, Boboy, may
remove
principal
thereby.

the

same,

thing
If

in

improvements

even

may

suffer

removing
Boboy

though

the

damage

the

useful

caused

more

impairment in the property leased than


is necessary he will be liable for damages
(Art. 1678, Civil Code).

Lease; Caveat Emptor (2009)


No.VIII. Jude owned a building which he
had leased to several tenants. Without
informing

his

tenants,

Jude

sold

the

building to Ildefonso. Thereafter, the latter


notified all the tenants that he is the new
owner of the building. Ildefonso ordered the
tenants to vacate the premises within thirty
(30) days from notice because he had other
plans for the building. The tenants refused
to vacate, insisting that they will only do so

expired. Is Ildefonso bound to respect the

status of the occupants of their right to


lease contracts between Jude and his

occupy the building before buying it.

tenants? Explain your answer. (3%)

Agency

SUGGESTED ANSWER:
Yes, Ildefonso must respect the lease
contracts between Jude and his tenants.
While it is true that the said lease
contracts

were

No.XVI. X was the owner of an unregistered

annotated on the title to the property,

parcel of land in Cabanatuan City. As she

Ildefonso

innocent

was abroad, she advised her sister Y via

purchaser for value. He ought to know

overseas call to sell the land and sign a

the existence of the lease because the

contract of sale on her behalf.

still

registered

an Agent (2010)

and

is

not

Agency; Sale of a Real Property through

not

an

building was already occupied by the


tenants

at

the

time

he

bought

it.

Y thus sold the land to B1 on March 31,


2001 and executed a deed of absolute sale

Applying the principle of caveat emptor,

on behalf of X. B1 fully paid the purchase

he should have checked and known the

price.

Never Let The Odds Keep You From Pursuing What You Know In Your Heart You Were Meant To Do.Leroy Satchel Paige

Page 104 of 180

Civil Law Q&As (2007-2013)

hectorchristopher@yahoo.com

dbaratbateladot@gmail.com

inscribed in the Registry of Deeds, the


B2, unaware of the sale of the land to B1,

case is governed by Art 1544 of the New

signified to Y his interest to buy it but

Civil Code which provides that in case of

asked Y for her authority from X. Without


informing X that she had sold the land to
B1, Y sought X for a written authority to
sell.
X e-mailed Y an authority to sell the land. Y
thereafter sold the land on May 1, 2001 to
B2 on monthly installment basis for two
years, the first installment to be paid at the
end of May 2001.
Who between B1 and B2 has a better right
over the land? Explain. (5%)
SUGGESTED ANSWER:
B-2 has a better title. This is not a case
of double sale. Since the first sale was
void. The law provides that when a sale
of a piece of land or any interest therein
is through an agent, the authority of the
latter shall be in writing; otherwise, the
sale shall be void (Art 1874, NCC). The
property was sold by Y to B1 wihtout any
written authority from the owner X.
Hence, the sale to B1 was void.
ALTERNATIVE ANSWER:
Under the facts, B-1 has a better right to
the land. Given the fact that the Deed of
Sale in favor of B-1 and B-2 are not

extinguishing right to real property not


double sales of an immovable property,

registered under Act 496 shall be valid

the

the

except as between the parties. Thus, the

person who is in good faith was first in

Deed of Sale of B-2 has no binding effect

possession and in the absence thereof to

on B-1.

ownership

shall

pertain

to

the person who presents the oldest title,

Partnership

provide there is good faith.


In a case, the Supreme Court has held
that in a sale of real estate the execution
of

notarial

document

of

sale

is

Liability; Liability of a Partner (2010)


No.XV.

A,

partnership

B,

and

to

operate

entered
a

into

restaurant

tantamount to delivery of the possession

business. When the restaurant had gone

of the property sold. The ownership of

past break-even stage and started to garner

the land therefore pertains to the first

considerable profits, C died. A and B

buyer. It may also be mentioned that

continued the business without dissolving

under Art 3344 no instruments or deed

the partnership. They in fact opened a

establishing,
acknowledging,

transmitting,
modifying,

or

branch

of

the

restaurant,

incurring

obligations in the process. Creditors started

Never Let The Odds Keep You From Pursuing What You Know In Your Heart You Were Meant To Do.Leroy Satchel Paige

Page 105 of 180

Civil Law Q&As (2007-2013)

hectorchristopher@yahoo.com

dbaratbateladot@gmail.com

1829, 1835, par 2, NCC; Testate Estate


demanding

for

the

payment

of

their

464 [1925]). However, the liability of Cs

obligations.
(A). Who are liable for the settlement of the
partnerships obligations? Explain? (3%)
SUGGESTED ANSWER:
The two remaining partners, A and B, are
liable. When any partner dies and the
business

is

continued

without

any

settlement of accounts as between him


or his estate, the surviving partners are
held liable for continuing the business
despite the death of C (Art 1841, 1785,
par 2, and Art 1833 of NCC).
(B).What are the creditors

recourse/s?

Explain. (3%)
SUGGESTED ANSWER:
Creditors
actions,

of Mota v. Serra, 47 Phil

can
for

file

instance,

the
an

appropriate
action

for

collection of sum of money against the


partnership at will and if there are no
sufficient funds, the creditors may go
after the private properties of A and B
(Art 816, NCC). Creditors may also sue
the estate of C. The estate is not excused
from the liabilities of the partnership
even if C is dead already but only up to
the time that he remained a partner (Art

individual properties shall be subject


first to the payment of his separate
debts (Art 1835. NCC).

ALTERNATIVE ANSWER:
TRUE. An oral is a consensual of the
partnership is valid even though not in
writing.

However,

If

it

involves

contribution of an immovable property


or a real right, an oral contract of

Oral Partnership (2009)

partnership is void. In such a case, the

No.I. TRUE or FALSE. Answer TRUE if the

contract of partnership to be valid, must

statement

be in a public instrument ( Art. 1771

is

true,

or

FALSE

if

the

statement is false. Explain your answer in

,NCC

not more than two (2) sentences.

property signed by the parties must be

(C). An oral partnership is valid. (1%)

),

and

the

inventory

of

said

attached to said public instrument (Art.


1773, NCC).

SUGGESTED ANSWER:
TRUE.

Partnership

is

consensual

ALTERNATIVE ANSWER:

contract, hence, it is valid even though


not in writing.

Never Let The Odds Keep You From Pursuing What You Know In Your Heart You Were Meant To Do.Leroy Satchel Paige

Page 106 of 180

Civil Law Q&As (2007-2013)

hectorchristopher@yahoo.com

dbaratbateladot@gmail.com

the existence of a partnership. Do you


TRUE.

Partnership

is

consensual

contract, hence, it is valid even though


not in writing. The oral contract of
partnership is also valid even if an
immovable property or real right is
contributed thereto. While the law, in
such a case, requires the partnership to
be in a public document, the law does
not expressly declare the contract void if
not

executed

in

the

required

form

(Article 1409 (7 ,NCC ). And there being


nothing in the law from which it can be
inferred that the said requirement is
prohibitory
NCC),

the

or

mandatory

said

oral

(Article

contract

5,
of

partnership must also be valid. The


interested party may simply require the
contract

to

be

made

into

public

document in order to comply with the


required form (Article 1357, NCC). The
purpose of the law in requiring a public
document is simply to notify the public
about the contribution.

Share; Demand during the Existence of


Partnership (2012)
No.X.b) A partner cannot demand the
return of his share (contribution) during

agree? Explain your answer. (5%)


SUGGESTED ANSWER:

Such agreement was not reduced to writing.


Yes, he is not entitled to the return of
his contribution to the capital of the
partnership, but only to the net profits
from the partnership business during the
life of the partnership period. If he is a
limited partner, however, he may ask for
the

return

of

his

contributions

by Villanueva in cash and checks. The total


amount Villanueva paid accumulated to P
1, 200, 000.00. Upon advice of her lawyer,
Villanueva demanded for the return of the
excess amount of P 660, 000.00 which was
ignored by Siga-an.

as

provided in Art 1856 and 1857, Civil

(1) Is the payment of interest valid? Explain.


(3%)

Code.

SUGGESTED ANSWER:

Commodatum & Mutuum


Mutuum;

Siga-an demanded interest which was paid

Interest;

Solutio

Indebiti

No, Art. 1956, Civil Code, provides that


no interest shall be due unless it has

(2012)

been expressly stipulated in writing.


No.VI.a)

Siga-an

granted

loan

to

Villanueva in the amount of P 540, 000.00.

(2) Is solution indebiti applicable? Explain.


(2%)

Never Let The Odds Keep You From Pursuing What You Know In Your Heart You Were Meant To Do.Leroy Satchel Paige

Page 107 of 180

Civil Law Q&As (2007-2013)

hectorchristopher@yahoo.com

promise

answer

dbaratbateladot@gmail.com

for

the

debt,

or

miscarriage of another, the Statute of

SUGGESTED ANSWER:

Frauds requires it to be in writing to be


Yes,

Solutio

because

Indebiti

Villanueva

P600,000.00

is

applicable

Overpaid

representing

by

interest

payment which is not due. He can,


therefore, demand its return.

Guaranty
Guaranty (2009)
No.I. TRUE or FALSE. Answer TRUE if the
statement

is

true,

or

FALSE

if

the

statement is false. Explain your answer in


not more than two (2) sentences.
(D). An oral promise of guaranty is valid
and binding. (1%)
SUGGESTED ANSWER :
FALSE. An oral contract of guaranty,
being a special promise to answer for the
debt of
another,

is

unenforceable

unless

in

writing (Article 1403 [2] b, NCC ).


ALTERNATIVE ANSWER:
TRUE. An oral promise of guaranty is
valid and binding. While the contract is
valid,

however

,it

is

unenforceable

because it is not writing . Being a special

(1)The

obligation

in

guaranty

is

enforceable ( Article 1403 [2] b, NCC).The

secondary; whereas, in suretyship, it is

validity

primary.

of

the

contract

should

be

distinguished from its enforceability .

Surety

(2) In guranty, the undertaking is to pay


if

the

principal

debtor

cannot

pay;

whereas, in suretyship, the undertaking


Surety (2010)

is to pay if the principal debtor does not

No.III. Define, Enumerate or Explain. (2%


each)
(A).

pay .
(3) In guranty, the guarantor is entitled

What

is

the

difference

between

"guaranty" and "suretyship"?


SUGGESTED ANSWER:
Guaranty and Suretyship distinguished

to the benefit of excussion; whereas, in


suretyship the surety is not entitled.
(4) Liability in guaranty depends upon
an independent agreement to pay the
obligations of the principal if he fails to
do so; whereas, in suretyship, the surety
assumes liability as a regular party.

Never Let The Odds Keep You From Pursuing What You Know In Your Heart You Were Meant To Do.Leroy Satchel Paige

Page 108 of 180

Civil Law Q&As (2007-2013)

hectorchristopher@yahoo.com

dbaratbateladot@gmail.com

by the pledgee of the thing pledged in


(5)The Guarantor insures the solvency
of the principal debtor; whereas, the
surety insures the debt.
(6)In

guaranty,

subsidiarlty
suretyship,

the

liable;
the

guarantor

whereas,

surety

binds

in

is
a

himself

solidarity with the principal debtor (Art


2047, Civil Code).

Pledge
Pledge; Pactum Commissorium (2009)
No.XVII.

Rosario

obtained

loan

of

P100,000.00 from Jennifer, and pledged her


diamond ring. The contract signed by the
parties stipulated that if Rosario is unable
to redeem the ring on due date, she will
execute a document in favor of Jennifer
providing that the ring shall automatically
be considered full payment of the loan.
(A). Is the contract valid? Explain. (3%)
SUGGESTED ANSWER:
The contract is valid because Rosario
has to execute a document in favor of
Jennifer to transfer the ownership of the
pledged ring to the latter. The contract
does

not

commissorium

amount
because

to
it

pactum
does

not

provide for the automatic appropriation

case of default by the pledgor.

them xxx. Jennifer cannot immediately


(B). Will your answer to [a] be the same if

sell by herself the thing pledged. It must

the contract stipulates that upon failure of

be foreclosed by selling it at a public

Rosario to redeem the ring on due date,

auction

Jennifer may immediately sell the ring and

procedure under Art 2112 of the New

appropriate the entire proceeds thereof for

Civil Code.

herself

as

full

payment

of

the

loan?

Reasons. (3%)
SUGGESTED ANSWER:
No, my answer will be different. While
the contract of pledge is valid, the

in

accordance

with

the

Torts and Damages


Damages (2012)
No.I. a) Roberto was in Nikko Hotel when he
bumped into a friend who was then on her

stipulation authorizing the pledgee to

way to a wedding reception being held in

immediately sell the thing pledged is

said hotel. Roberto alleged that he was then

void under Art 2088 of the New Civil

invited by his friend to join her at the

Code, which provides that the creditor

wedding reception and carried the basket

cannot appropriate the things given by

full of fruits which she was bringing to the

way of pledge or mortgage, or dispose of

affair. At the reception, the wedding

Never Let The Odds Keep You From Pursuing What You Know In Your Heart You Were Meant To Do.Leroy Satchel Paige

Page 109 of 180

Civil Law Q&As (2007-2013)

hectorchristopher@yahoo.com

dbaratbateladot@gmail.com

that exposed Roberto to unnecessary


coordinator of the hotel noticed him and
asked him, allegedly in a loud voice, to leave
as he was not in the guest list. He retorted
that he had been invited to the affair by his
friend,
Deeply

who

however

embarrassed

denied
by

doing

the

so.

incident,

Roberto then sued the hotel for damages


under Articles 19 and 21 of the
Civil Code. Will Robertos action prosper?
Explain. (5%)
SUGGESTED ANSWER:
No. Robertos action will not prosper.
From the facts given in the problem, the
wedding coordinator did not abuse her
right when she asked him to leave the
wedding reception because he was not in
the guest list. Hotel Nikko could not be
held liable for damages as its liable
spring from the liability of its employee
(Nikko Hotel Manila Garden v. Reyes,
G.R. No. 154259, Feb 28, 2005).
ALTERNATIVE ANSWER:
It depends. While the hotel has the right
to exclude an uninvited guest from the
wedding reception, that does not give
the

hotel

the

license

to

humiliate

Roberto. If the wedding coordinator of


the hotel acted wrongfully e.g. with the
abuse of right, unfairly, or in a matter

ridicule or shame, his action will

finding. In the contrary, the problem


prosper. Otherwise, Robertos action will

states that it is a mere allegation.

not prosper.
The hotel is liable for the wrongful acts
of its employees.

Damages; Moral & Exemplary (2009)


No.XIV. Rodolfo, married to Sharon, had an

COMMENT:

illicit affair with his secretary, Nanette, a

The facts of the problem are almost

19-year old girl, and begot a baby girl,

similar to the facts of Nikko Hotel

Rona. Nanette sued Rodolfo for damages:

Manila

actual, for hospital and other medical

Garden

v.

Reyes,

G.R.

No.

154259, Feb 28, 2005. In the said case,


however, there is a categorical finding
that the hotel employee did not, exposed
the complainant to the ridicule, shame
or

embarrassment;

hence,

did

not

expenses

in

delivering

the

child

by

caesarean section; moral, claiming that


Rodolfo

promised

to

marry

her,

representing that he was single when, in


fact, he was not; and exemplary, to teach a
lesson to like-minded Lotharios.

commit any abuse of right. The present


problem makes no statement of that

Never Let The Odds Keep You From Pursuing What You Know In Your Heart You Were Meant To Do.Leroy Satchel Paige

Page 110 of 180

Civil Law Q&As (2007-2013)

hectorchristopher@yahoo.com

dbaratbateladot@gmail.com

thereby imposed higher taxes), which the


(A). If you were the judge, would you award
all the claims of Nanette? Explain. (3%)
SUGGESTED ANSWER:
If Rodolfo's marriage could not have
been possibly known to Nanette or there
is no gross negligence on the part of
Nanette, Rodolfo could be held liable for
moral damages.

If there is gross negligence in a suit for


quasi-delict,

exemplary

could

be

awarded.

Damages; Public Officers acting in the


Performance of their Duties (2012)
No.II.a) Liwayway Vinzons-Chato was then
the

Commissioner

of

Internal

Revenue

while Fortune Tobacco Corporation is an


entity

engaged

in

the

manufacture

of

different brands of cigarettes, among which


are

"Champion,"

"Hope,"

and

"More"

cigarettes.
Fortune filed a complaint against VinzonsChato to recover damages for the alleged
violation of its constitutional rights arising
from Vinzons-Chatos issuance of Revenue
Memorandum Circular No. 37-934 (which
re-classified Fortune cigarettes as locally
manufactured with foreign brands and

Supreme Court later declared invalid.

The public officer is not automatically


Vinzons-Chato filed a Motion to Dismiss

considered to have violated the rights or

arguing that she cannot be held liable for

liberties of a person simply because the

damages for acts she performed while in

rule

the

declared

discharge

of

her

duties

as

BIR

Commissioner. Is she correct? Explain. (5%)


SUGGESTED ANSWER:

the

public
invalid

officer
by

the

issued

was

court.

The

complainant must still allege and prove


the particular injury or prejudice he has
suffered

from

the

violation

of

his

Yes. As a general rule, a public officer is

constitutional right by the issuance of

not liable for acts performed in the

the invalidated rule.

discharge of his duties. The exceptions


are when he acted with malice, bad faith,
or gross negligence in the performance
of his duty, or when his act is in
violation of a Constitutional guaranteed
right and liberties of a person under
Art32 of the NCC.

The problem does not state any fact from


which any malice, bad faith or gross
negligence on the part of Vinzons-Chato
may be inferred, or the particular injury
or prejudice the complainant may have
suffered as a result of the violation of his
constitutional right. Hence, she

Never Let The Odds Keep You From Pursuing What You Know In Your Heart You Were Meant To Do.Leroy Satchel Paige

Page 111 of 180

Civil Law Q&As (2007-2013)

hectorchristopher@yahoo.com

dbaratbateladot@gmail.com

civil liability, Rommel is subsidiarily


cannot

be

held

liable.

The

facts

presented are similar to facts of the case


of Vinzons-Chato v. Fortune, G.R. No.
141309, Dec 23, 2008.

Death Indemnity (2009)


No. X. Rommels private car, while being
driven by the regular family driver, Amado,
hits a pedestrian causing the latters death.
Rommel is not in the car when the incident
happened.
(A). Is Rommel liable for damages to the
heirs of the deceased? Explain. (2%)
SUGGESTED ANSWER:
Yes, Rommel may be held liable for
damages if he fails to prove that he
exercised the diligence of a good father
of a family (Art. 2180, par 5, NCC) in
selecting

and

supervising

his

family

driver. The owner is presumed liable


unless

he

proves

the

defense

of

diligence. If the driver was performing


his assigned task when the accident
happened, Rommel shall be solidarily
liable with the driver.

In

case

the

driver

is

convicted

of

reckless imprudence and cannot pay the

liable for the damage awarded against

Yes, my answer will be the same except


the driver and the defense of diligence is

that in such case the liability of the

not available.

owner is not presumed. When the owner


is inside the vehicle, he becomes liable

(B).Would your answer be the same if

only when it is shown that he could have

Rommel was in the car at the time of the

prevented the misfortune by the use of

accident? Explain. (2%)


SUGGESTED ANSWER:

due diligence (Art. 2184, NCC). For the


owner to be held liable, the burden of

Yes, my answer would be the same.

proving that he could have prevented

Rommel, who was in the car, shall be

the misfortune rests on the shoulder of

liable for damages if he could have

the victim.

prevented the misfortune by the use of


due diligence in supervising his driver
but failed to exercise it (Art. 2184, NCC).

Doctrine of Discovered Peril (Last Clear

In such case, his liability is solidary with

Chance) (2007)

his driver.
ALTERNATIVE ANSWER:

No.III. Explain the following concepts and


doctrines and give an example of each:

Never Let The Odds Keep You From Pursuing What You Know In Your Heart You Were Meant To Do.Leroy Satchel Paige

Page 112 of 180

Civil Law Q&As (2007-2013)

hectorchristopher@yahoo.com

dbaratbateladot@gmail.com

No.XIV. Primo owns a pet iguana which he


(B). doctrine of discovered peril (last clear

keeps in a man-made pond enclosed by a

chance) (5%)

fence situated in his residential lot. A

SUGGESTED ANSWER:
The doctrine of last clear chance states
that where the plaintiff was guilty of
prior or antecedent negligence, but the
defendant,

who

opportunity

to

had
avoid

the
the

ultimate
impending

harm failed to do so, it is the defendant


who is liable for all the consequences of
the accident notwithstanding the prior
negligence of the plaintiff. An example is
where a person was riding a pony on a
bridge and improperly pulled the pony to
the wrong side when he saw a car
coming. The driver of the car did not
stop or change direction, and nearly hit
the horse, and, the frightened animal
jumped to its death. The driver of the
car is guilty of negligence because he
had a fair opportunity to avoid the
accident and failed to avail himself of
that opportunity. He is liable under the
doctrine of last clear chance (Picart v.
Smith, 37 Phil. 809, 1918).

Liability; Owner of a Pet; Fortuitous


Event (2010)

in case the damage should come from


typhoon knocked down the fence of the

force majeure or from the fault of the

pond and the iguana crawled out of the gate

person who has suffered damage (Art

of Primos residence. N, a neighbor who was

2183, NCC).

passing by, started throwing stones at the


iguana, drawing the iguana to move toward
him. N panicked and ran but tripped on
something and suffered a broken leg.

Liability;

Special

Parental

Authority

(2010)

Is anyone liable for Ns injuries? Explain.


(4%)

No.XII. On May 5, 1989, 16-year old


Rozanno, who was issued a student permit,

SUGGESTED ANSWER:
No one is liable. The possessor of an
animal or whoever may make use of the
same is responsible for the damage it

drove to school a car, a gift from his


parents. On even date, as his class was
scheduled to go on a field trip, his teacher
requested him to accommodate in his car,
as he did, four (4) of his classmates

may cause, although it may escape or be


lost. This responsibility shall cease only
Never Let The Odds Keep You From Pursuing What You Know In Your Heart You Were Meant To Do.Leroy Satchel Paige

Page 113 of 180

Civil Law Q&As (2007-2013)

hectorchristopher@yahoo.com

dbaratbateladot@gmail.com

principally liable under Art 221 (FC), if


because the van rented by the school was
too crowded. On the way to a museum
which the students were scheduled to visit,
Rozanno made a wrong maneuver, causing
a collision with a jeepney. One of his
classmates died. He and the three (3) others
were badly injured.
(A). Who is liable for the death of Rozannos
classmate and the injuries suffered by
Rozanno and his 3 other classmates?
Explain. (2%)
SUGGESTED ANSWER:
At the time the incident occurred in May
1989, Rozanno was still a minor. Being a
minor, Art 218 of the Family Code
applies. Pursuant to Art 218, the school,
its administrators and teachers shall be
liable for the acts of minor Rozanno
because of the special parental authority
and responsibility that they exercise
over him. The authority applies to all
authorized activities, whether inside or
outside the premises of the school,
entity or institution. The field trip on
which occasion Rozanno drove the car,
was an authorized activity, and , thus,
covered by the provision. Furthermore,
the parents of Rozanno are subsidiarily
liable pursuant to Art 219 (FC), and

they are negligent.

the middle of 1994, Rozanno have been


(B). How about the damage to the jeepney?

21 years old at the time. Hence, he was

Explain. (2%)

already of legal age. The law reducing the


age of majority to 18 years took effect in

SUGGESTED ANSWER:
With respect to the damages caused to
the jeepney, only Rozanno should be
held liable because his negligence or
tortuous act was the sole, proximate and
immediate cause thereof.

December 1989.
Being of legal age, articles 218, 219, and
221 of the Family Code are no longer
applicable. In such case, only Rozanno
will be personally responsible for all the
consequences

of

his

act

unless

his

(C). Under the same facts, except the date of

school or his parents were themselves

occurrence of the incident, this time in

also

mid-1994, what would be your answer?

contributed to the happening of the

Explain. (2%)

incident. In that event, the school or his

SUGGESTED ANSWER:

negligent

and

such

negligence

parents are not liable under Art 218, 218


or 221 of the Family Code, but will be

Since Rozanno was 16 years old in 1989,


if the incident happened sometime in
Never Let The Odds Keep You From Pursuing What You Know In Your Heart You Were Meant To Do.Leroy Satchel Paige

Page 114 of 180

Civil Law Q&As (2007-2013)

hectorchristopher@yahoo.com

evidence
liable under general provision on the
Civil Code on quasi-delict.

Quasi-Delict; Claims; Requisites (2013)


No.II. A collision occurred at an intersection
involving a bicycle and a taxicab. Both the
bicycle rider (a businessman then doing his
morning

exercise)

and

the

taxi

driver

claimed that the other was at fault. Based


on the police report, the bicycle crossed the
intersection first but the taxicab, crossing
at a fast clip from the bicycle's left, could
not brake in time and hit the bicycle's rear
wheel, toppling it and throwing the bicycle
rider into the sidewalk 5 meters away.
The bicycle rider suffered a fractured right
knee, sustained when he fell on his right
side on the concrete side walk. He was
hospitalized

and

was

subsequently

operated on, rendering him immobile for 3


weeks and requiring physical rehabilitation
for another 3 months. In his complaint for
damages, the rider prayed for the award
ofP1,000,000 actual damages,P200,000
moral
damages, P200,000
damages, P1 00,000 nominal

exemplary
damages

and P50,000 attorney's fees.

Assuming the police report to be correct


and as the lawyer for the bicycle rider, what

dbaratbateladot@gmail.com

(documentary

and

testimonial)

and legal arguments will you present in

The case clearly involves quasi-delict


court to justify the damages that your client

where

my

claims? (8%)

suffered

injury

negligence
SUGGESTED ANSWER:

client,

of

as
the

the
a

bicycle
result

rider,
of

the

over-speeding

taxi

driver, without fault on my clients part.

I will the base the claim of my client on


quasi-delict under Art 2176 of the Civil
Code of the Philippines. The requisites
for a claim under quasi-delict to prosper
are as follows:

To prove actual damages aside from the


testimony of client, I will present his
hospital and medical bills. Receipts paid
on

the

rehabilitation

will

also

be

presented. [The sentence in red should

(1) Act or omission, there being fault or

be replaced with the following sentence


because he is a businessman and not an

negligence;

employee. Furthermore, I will present


(2) Damage or injury; and
(3)

Causal

connection

income tax returns, contracts and other


between

damage and the act or omission.

the

documents to prove unrealized profits as


a result of this temporary injury.] I will

Never Let The Odds Keep You From Pursuing What You Know In Your Heart You Were Meant To Do.Leroy Satchel Paige

Page 115 of 180

Civil Law Q&As (2007-2013)

hectorchristopher@yahoo.com

dbaratbateladot@gmail.com

eyewitness accounts, in order to support


also call the attending physician to
testify as to the extent of the injuries
suffered by my client, and to corroborate
the contents of the medical documents.
Based on Art. 2202, in quasi-delicts, the
defendant shall be liable for all damages
which

are

the

natural

and

probable

consequences of the act or omission


complained of. It is not necessary that
the damages have been foreseen or could
have been foreseen by the defendant.
Unlike

actual

damages,

no

proof

of

pecuniary loss is necessary in order that


moral, nominal, temperate liquidated or
exemplary damages may be adjudicated.
The assessment is left to the discretion
of the Court (Art. 2216, Civil Code).
There

must

be

proof

pecuniary

estimation, however.
Moral damages can be recovered by my
client under Articles 2219 and 2200.
Moral damages may be recovered in case
of

quasi-delict

causing

physical

injuries. Additionally, it must be proved


that such damages were the proximate
result of the act complained of. Medical
certificates will be presented, along with
the testimony from my client and other

the award for moral damages.


Exemplary damages may be granted if
the defendant acted in wanton,

Finally, attorneys fees may be recovered


fraudulent,

reckless,

oppressive,

or

malevolent manner. While the amount of

when exemplary damages are awarded


(Art 2208, Civil Code).

exemplary damages may not be proved,


the

plaintiff

entitled
damages.

to
In

must
moral

show
or

support

of

that

he

is

compensatory
this,

will

present the police report showing the


circumstance under which the accident

Quasi Tort (2010)


No.III. Define, Enumerate or Explain. (2%
each)

took place, taking into account the

(B). Define quasi tort. Who are the persons

actions of the parties. I will ask the

liable under quasi torts and what are the

officials who responded to the accident

defenses available to them?

to testify as to the conduct of the


parties at the time of the accident in
order to determine whether defendant
was guilty of gross negligence.

Note: It is recommended that the examiner


exercise leniency and liberality in grading the
answers given to this question. The term quasitort is not part of legal developments in civil
law. In Philippine legal tradition, quasi-

Never Let The Odds Keep You From Pursuing What You Know In Your Heart You Were Meant To Do.Leroy Satchel Paige

Page 116 of 180

Civil Law Q&As (2007-2013)

hectorchristopher@yahoo.com

Quasi

-tort

is

dbaratbateladot@gmail.com

considered

as

the

delict has been treated as the closest civil

equivalent of quasi-delict. Hence the

law equivalent of the common law tort. In

rules of the latter pertaining to persons

fact,

in

decisions,

number
the

two

of

Supreme

terms

have

Court
been

considered synonymous. In reality, however,


the common law tort is much broader in
scope than the civil law quasi-delict. In
recent developments in common law, the
concept of quasi-torts can be considered as
the closest common law equivalent of the
civil law concept of quasi-delict. This is
because it is argued that the growing
recognition of quasi-torts as a source of
obligation is hinged on the acceptance at
common law of the civil law principles of
quasi-delict.
SUGGESTED ANSWER:
Quasi -tort is a legal concept upholding
the doctrine that some legal duty exists
that cannot be classified strictly as a
personal duty (thus resulting in a tort),
nor as a contractual duty but rather
some other kind of duty recognizable by
the law. Tort or Quasi-tort is an
Anglo

American

or

Common

Law

concept, while Delict or Quasi-Delict


is

Civil

Law

concept

encyclopedia).
ALTERNATIVE ANSWER:

(Wikipedia

proximate cause of the injury ( Article


who can be held liable and their defenses
would also apply.

(c) That the plaintiff's own negligence


was the immediate and proximate cause

Those liable for quasi-delict include:


(1)

Those

2179 NCC );

tortfeasor

or

the

person

causing damage to another through fault

of his injury ( Article 2179 NCC );


(d ) That the person vicariously liable

or negligence ( Article 2176 NCC ); and

has observed all the diligence of a good

(2)

father of a family to prevent damage

Persons

vicariously

liable

under

( Article 2180 NCC ); and

Article 2180 (NCC ).

(e)
The defenses available include:

prescribed after the lapse s (Article 2179

(a) That the defendant was not negligent


or

that

he

exercised

due

diligence

( Article 2176 NCC );


(b)

That

negligent

although
his

That the cause of action has

NCC ).
The

fact

that

the

plaintiff

had

committed contributory negligence is a


the

negligence

defendant
is

not

is

partial defense (Art 2179, NCC).

the

Never Let The Odds Keep You From Pursuing What You Know In Your Heart You Were Meant To Do.Leroy Satchel Paige

Page 117 of 180

Civil Law Q&As (2007-2013)

hectorchristopher@yahoo.com

dbaratbateladot@gmail.com

If all the brothers/sisters are disqualified


to inherit, the nephews/nieces inherit

MULTIPLE CHOICE
QUESTIONS
2013

Taxation

Law

Exam

MCQ (October 13, 2013)


I. Armand died intestate. His full-blood
brothers, Bobby and Conrad, and halfblood brothers, Danny, Edward and Floro,
all predeceased him. The following are the
surviving relatives:
1. Benny and Bonnie, legitimate children of
Bobby;
2. Cesar, legitimate child of Conrad;
3. Dante, illegitimate child of Danny;
4. Ernie, adopted child of Edward; and
5. Felix, grandson of Floro.
The

net

value

of

Armand's

estate

is

Pl,200,000.
I. (1) How much do Benny and Bonnie
stand to inherit by right of representation?
(1%)
(A) P200,000
(B) P300,000
(C) P400,000
(D) P150,000
(E) None of the above.
SUGGESTED ANSWER:
(E) None of the above.

relationship is legitimate. Thus, Dante,


per capita, and not by right of

an illegitimate child of Danny, is barred

representation (Art 975, Civil Code)

from inheriting from Armand pursuant


to

the

iron

curtain

rule

which

I. (2) How much is Dante's share in the net

disqualifies an illegitimate child from

estate? (1%)

inheriting

(A) P150,000.

legitimate children and relatives of his

(B) P200,000.

intestao

from

the

father or mother, and vice versa (Art

(C) P300,000.

992, Civil Code).

(D) P400,000.
(E) None of the above.

I. (3) How much is Ernie's share in the net


estate . (1%)

SUGGESTED ANSWER:

(A) P 0.

E. None of the above.


There is no showing that Danny is an
illegitimate

ab

half-blood

brother

of

(B) P400,000.
(C) P150,000.
(D) P200,000.

Armand. In the absence of proof to the

(E) None of the above.

contrary, the law presumes that the

SUGGESTED ANSWER:

Never Let The Odds Keep You From Pursuing What You Know In Your Heart You Were Meant To Do.Leroy Satchel Paige

Page 118 of 180

Civil Law Q&As (2007-2013)

hectorchristopher@yahoo.com

dbaratbateladot@gmail.com

obligation became due and demandable, C


turned out to be insolvent.

(A) 0 or (E) None of the above.


The

legal

relationship

created

by

adoption is strictly between the adopter


and the adopted. It does not extend to
the relatives of either party (Sayson v.
CA, G.R. Nos. 89224-25, Jan 23, 1992).
(Note: E. None of the above is another
answer because Ernie has no share at
all in the net estate).

I. (4) How much is Felix's share in the net


estate? (1%)
(A) P400,000.
(B) P150,000.
(C) P300,000.
(D) P0.
(E) None of the above.
SUGGESTED ANSWER:
(D). 0. Or (E) None of the above.
In the collateral line, representation is
granted only to children of brother or
sisters,

Felix

is

grandson

of

predeceased brother.
(Note: E. None of the above: is another
answer because Felix has no share at
all in the net estate)

II. A, B, C and D are the solidary debtors of


X for P40,000. X released D from the
payment of his share of PI 0,000. When the

(D). No, as the release of the share of one


Should the share of insolvent debtor C be

debtor would then increase the burden of

divided

the other debtors without their consent.

only

between

the

two

other

remaining debtors, A and B? (1%)

When one of the solidary debtors cannot,

(A) Yes. Remission of D's share carries with

because of his insolvency, reimburse his

it total extinguishment of his obligation to

share

the benefit of the solidary debtors.

obligation, such share shall be borne by

(B) Yes. The Civil Code recognizes remission


as a mode of extinguishing an obligation.
This clearly applies to D.
(C) No. The rule is that gratuitous acts
should be restrictively construed, allowing
only the least transmission of rights.
(D) No, as the release of the share of one
debtor would then increase the burden of
the other debtors without their consent.
SUGGESTED ANSWER:

to

the

debtor

paying

the

all his co-debtors, in proportion to the


debt of each (Art 1217, Civil Code).
Additionally, D was released only from
his share of P10,000.00 not from the
solidary tie that binds him to A, B and C.

III. Amador obtained a loan of P300,000


from Basilio payable on March25, 2012. As
security for the payment of his loan,
Amador constituted a mortgage on his
residential house and lot in Basilio's favor.

Never Let The Odds Keep You From Pursuing What You Know In Your Heart You Were Meant To Do.Leroy Satchel Paige

Page 119 of 180

Civil Law Q&As (2007-2013)

hectorchristopher@yahoo.com

dbaratbateladot@gmail.com

and has resorted to all the legal remedies


Cacho, a good friend of Amador, guaranteed

against the debtor (Art. 2058, Civil Code)

and obligated himself to pay Basilio, in case

(Note: A is not the correct answer

Amador fails to pay his loan at maturity.

because it states that all the assets of

III. (1) If Amador fails to pay Basilio his loan


on March 25, 2012, can Basilio compel
Cacho to pay? (1%)
(A) No, Basilio cannot compel Cacho to pay
because as guarantor, Cacho can invoke the
principle of excussion, i.e., all the assets of
Basilio must first be exhausted.
(B) No, Basilio cannot compel Cacho to
pay because Basilio has not exhausted
the available remedies against Amador.
(C) Yes, Basilio can compel Cacho to pay
because the nature of Cacho's undertaking
indicates

that

he

has

bound

himself

solidarily with Amador.


(D) Yes, Basilio can compel Cacho who
bound himself to unconditionally pay in
case Amador fails to pay; thus the benefit of
excussion will not apply.
SUGGESTED ANSWER:
(B) No, Basilio cannot compel Cacho to
pay because Basilio has not exhausted
the available remedies against Amador.
The guarantor cannot be compelled to
pay the creditor unless the latter has
exhausted all the property of the debtor

ownership on the buyer, Diego, who must


Basilio

(the

creditor)

must

first

be

therefore consent.

exhausted)

(D) No, Basilio cannot foreclose the real

III. (2) If Amador sells his residential house

estate mortgage. To deprive the new owner

and lot to Diego, can Basilio foreclose the

of ownership and possession is unjust and

real estate mortgage? (1%)

inequitable.

(A) Yes, Basilio can foreclose the real estate

SUGGESTED ANSWER:

mortgage because real estate mortgage

(B) Yes, Basilio can foreclose the real

creates a real right that attaches to the

estate mortgage. It is binding upon Diego

property.

as the mortgage is embodied in a public

(B) Yes, Basilio can foreclose the real

instrument.

estate mortgage. It is binding upon Diego

Since

as the mortgage is embodied in a public

instrument, there is constructive notice

instrument.

to

(C) No, Basilio cannot foreclose the real

mortgaged property.

estate

mortgage.

The

sale

the

Diego,

mortgage

who

is

is

the

in

buyer

public

if

the

confers
ALTERNATIVE ANSWER:

Never Let The Odds Keep You From Pursuing What You Know In Your Heart You Were Meant To Do.Leroy Satchel Paige

Page 120 of 180

Civil Law Q&As (2007-2013)

hectorchristopher@yahoo.com

dbaratbateladot@gmail.com

(C) Yes, Jose's refusal is justified. The


(C) No, Basilio cannot foreclose the real
estate

mortgage.

The

sale

confers

ownership on the buyer, Diego, who must


therefore consent.
The mortgage is not registered, thus,
cannot be binding against third persons
(Art. 2125, Civil Code)

IV. Cruz lent Jose his car until Jose


finished his Bar exams. Soon after Cruz
delivered

the

car,

Jose

brought

it

to

Mitsubishi Cubao for maintenance check


up and incurred costs of P8,000. Seeing the
car's peeling and faded paint, Jose also had
the car repainted for P10,000. Answer the
two

questions

below

based

on

these

common facts.
IV. (1) After the bar exams, Cruz asked for
the return of his car. Jose said he would
return it as soon as Cruz has reimbursed
him for the car maintenance and repainting
costs of P 18,000.
Is Jose's refusal justified? (1%)
(A) No, Jose's refusal is not justified. In this
kind of contract, Jose is obliged to pay for
all

the

expenses

incurred

for

the

preservation of the thing loaned.


(B) Yes, Jose's refusal is justified. He is
obliged to pay for all the ordinary and
extraordinary

expenses,

reimbursement from Cruz.

but

subject

to

principle of unjust enrichment warrants the


reimbursement of Jose's expenses.

The

bailee,

Jose,

has

no

right

of

(D) No, Jose's refusal is not justified. The

retention on the ground that the bailor

expenses he incurred are useful for the

owes him something, even if it may be

preservation of the thing loaned. It is

by reason of expenses. He can only

Jose's obligation to shoulder these useful

retain it if he suffers damages by reason

expenses.

of a flaw or defect in the thing loaned of


which the bailor knows (Art 1951, Civil

SUGGESTED ANSWER:

Code).

(D) No, Jose's refusal is not justified. The


expenses he incurred are useful for the

IV. (2) During the bar exam month, Jose

preservation of the thing loaned. It is

lent the car to his girlfriend, Jolie, who

Jose's obligation to shoulder these useful


expenses.
In commodatum, the bailee is obliged to

parked the car at the Mall of Asia's open


parking lot, with the ignition key inside the
car. Car thieves broke into and took the car.

pay for the ordinary expenses for the use

Is Jose liable to Cruz for the loss of the car

and preservation of the thing loaned (Art

due to Jolie's negligence? (1%)

1941, Civil Code).


Never Let The Odds Keep You From Pursuing What You Know In Your Heart You Were Meant To Do.Leroy Satchel Paige

Page 121 of 180

Civil Law Q&As (2007-2013)

hectorchristopher@yahoo.com

dbaratbateladot@gmail.com

000,000. P joined as an industrial partner,


(A) No, Jose is not liable to Cruz as the loss
was not due to his fault or negligence.
(B) No, Jose is not liable to Cruz. In the
absence of any prohibition, Jose could lend
the car to Jolie. Since the loss was due to
force majeure, neither Jose nor Jolie is
liable.
(C) Yes, Jose is liable to Cruz. Since Jose
lent the car to Jolie without Cruz's
consent, Jose must bear the consequent
loss of the car.
(D) Yes, Jose is liable to Cruz. The contract
between them is personal in nature. Jose
can neither lend nor lease the car to a third
person.
SUGGESTED ANSWER:
(C) Yes, Jose is liable to Cruz. Since Jose
lent the car to Jolie without Cruz's
consent, Jose must bear the consequent
loss of the car.
The bailee is liable for the loss of the
thing, even if it should be through a
fortuitous event if he lends or leases the
thing to a third person, who is not a
member of his household (Art 1942, Civil
Code).

V. In 2005, L, M, N, 0 and P formed a


partnership. L, M and N were capitalist
partners who contributed P500,000 each,
while 0, a limited partner, contributed P1 ,

contributing only his services. The Articles


of Partnership, registered with the

V. (l) Assuming that the just and equitable


Securities

and

Exchange

Commission,

designated L and 0 as managing partners; L


was liable only to the extent of his capital
contribution; and P was not liable for

share of the industrial partner, P, in the


profit in 2006 amounted to P1 00,000, how
much is the share of 0, a limited partner, in
the P800,000 net profit? (1%)

losses.

(A) P160,000.

In 2006, the partnership earned a net profit

(B) P175,000.

of P800,000. In the same year, P engaged in

(C) P280,000.

a different business with the consent of all

(D) P200,000.

the

(E) None of the above.

partners.

partnership

However,

incurred

in

2007,

net

loss

the
of

P500,000. In 2008,the partners dissolved

SUGGESTED ANSWER:

the partnership. The proceeds of the sale of

(C) P280,000.

partnership

First, deduct the share of P from the

assets

were

insufficient

to

settle its obligation. After liquidation, the

profits.

partnership

P700,000. Next, get the share of O by

ofP300,000.

had

an

unpaid

liability

P800,000

less

P100,000

is

following the proportion that the shares


of L, M, N, O is 1:1:1:2, respectively.

Never Let The Odds Keep You From Pursuing What You Know In Your Heart You Were Meant To Do.Leroy Satchel Paige

Page 122 of 180

Civil Law Q&As (2007-2013)

hectorchristopher@yahoo.com

dbaratbateladot@gmail.com

the partnership, 0 is liable as capitalist


partner.
V. (2) In 2007, how much is the share of 0,
a limited partner, in the net loss of
P500,000? (1%)
(A) P 0.
(B) P1 00,000.
(C) P125,000.
(D) P200,000.
(E) None of the above.
SUGGESTED ANSWER:
(D) P200,000
A limited partner shall not become liable
a s a general partner unless, in addition
to the exercise of his rights and powers
as a limited partner, he takes part in the
control of the business (Art 1948, Civil
Code). In the absence of stipulation as to
profits and losses, the share of each
partner

in

proportionate

the

losses

shall

be

to what he may have

contributed (Art 1797).

V. (3) Can the partnership creditors hold L,


0 and Pliable after all the assets of the
partnership are exhausted? (1%)
(A) Yes. The stipulation exempting P from
losses is valid only among the partners. L is
liable because the agreement limiting his
liability to his capital contribution is not
valid insofar as the creditors are concerned.
Having taken part in the management of

Articles of Partnership embodying such


(B) No. P is not liable because there is a
valid stipulation exempting him from losses.
Since the other partners allowed him to
engage in an outside business activity, the
stipulation absolving P from liability is

stipulations serves as constructive notice to


the partnership creditors.(E) None of the
above is completely accurate.
(E) None of the above is completely
accurate.

valid. For 0, it is basic that a limited


partner is liable only up to the extent of his

SUGGESTED ANSWER:

capital contribution.

(E) None of the above is completely

(C) Yes. The stipulations exempting P and L

accurate.

from losses are not binding upon the

VI. Gary is a tobacco trader and also a

creditors. 0 is likewise liable because the

lending investor. He sold tobacco leaves to

partnership was not formed in accordance

Homer

with

although the period for delivery was not

the

requirements

of

limited

for

delivery

within

month,

partnership.

guaranteed. Despite Gary's efforts to deliver

(D) No. The Civil Code allows the partners

on

to stipulate that a partner shall not be

government red tape hindered his efforts

liable for losses. The registration of the

and he could only deliver after 30 days.

time,

transportation

problems

and

Never Let The Odds Keep You From Pursuing What You Know In Your Heart You Were Meant To Do.Leroy Satchel Paige

Page 123 of 180

Civil Law Q&As (2007-2013)

hectorchristopher@yahoo.com

dbaratbateladot@gmail.com

contrary to the terms agreed upon, Gary


Homer refused to accept the late delivery
and to pay on the ground that the agreed
term had not been complied with.
As

lending

investor,

Gary

granted

Pl,000,000 loan to Isaac to be paid within


two years from execution of the contract. As
security for the loan, Isaac promised to
deliver to Gary his Toyota Innova within
seven (7) days, but Isaac failed to do so.
Gary

was

thus

compelled

to

demand

payment for the loan before the end of the


agreed two-year term.
VI. (l) Was Homer justified in refusing to
accept the tobacco leaves? (1%)
(A) Yes. Homer was justified in refusing to
accept the tobacco leaves. The delivery was
to be made within a month. Gary's promise
of delivery on a "best effort" basis made the
delivery uncertain. The term, therefore, was
ambiguous.
(B)

No.

Homer

was

not

justified

in

refusing to accept the tobacco leaves. He


consented to the terms and conditions
of

the

sale

and

must

abide

by

it.

Obligations arising from contract have


the force of law between the contracting
parties.
(C) Yes. Homer was justified in his refusal
to

accept

the

delivery.

The

contract

contemplates an obligation with a term.


Since the delivery was made after 30 days,

could not insist that Homer accept the


tobacco leaves.

It is clear under the facts that the period


(D) No. Homer was not justified in refusing

of delivery of the tobacco leaves was not

to accept the tobacco leaves. There was no

guaranteed.

term in the contract but a mixed condition.

factors which may prevent him from

The fulfillment of the condition did not

making the delivery within a month.

depend purely on Gary's will but on other

True enough, transportation problems

factors, e.g., the shipping company and the

and government

government. Homer should comply with his


obligation.

No.

Homer

was

not

justified

in

consented to the terms and conditions


the

sale

other

red tape did. Such

slight delay was, thus, excusable.

the force of law between the contracting

refusing to accept the tobacco leaves. He

of

anticipated

Obligations arising from contract have

SUGGESTED ANSWER:
(B)

Gary

and

must

abide

by

it.

Obligations arising from contract have

parties and should be complied with in


good faith (Art. 1160, Civil Code)

VI. (2) Can Gary compel Isaac to pay his


loan even before the end of the two-year
period? (1%)

the force of law between the contracting


parties.
Never Let The Odds Keep You From Pursuing What You Know In Your Heart You Were Meant To Do.Leroy Satchel Paige

Page 124 of 180

Civil Law Q&As (2007-2013)

hectorchristopher@yahoo.com

or
(A)

Yes,

Gary

can

immediately

pay

compel
the

Isaac

loan.

to

Non-

compliance with the promised guaranty


or

security

renders

the

obligation

immediately demandable. Isaac lost his


right to make use of the period.
(B)

Yes,

Gary

can

compel

Isaac

to

immediately pay the loan. The delivery of


the Toyota Innova is a condition for the
loan. Isaac's failure to deliver the car
violated the condition upon which the loan
was granted. It is but fair for Gary to
demand immediate payment.
(C) No, Gary

cannot compel

Isaac to

immediately pay the loan. The delivery of


the car as security for the loan is an
accessory contract; the principal contract is
still the P 1,000,000 loan. Thus, Isaac can
still make use of the period.
(D) No, Gary cannot compel Isaac to
immediately pay the loan. Equity dictates
that Gary should have granted a reasonable
extension of time for Isaac to deliver his
Toyota Innova. It would be unfair and
burdensome
for
Isaac
to
the P1,000,000
simplybecause

pay
the

promised security was not delivered.

SUGGESTED ANSWER:
(A)

Yes,

Gary

immediately

can
pay

compel
the

Isaac

loan.

to

Non-

compliance with the promised guaranty

security

dbaratbateladot@gmail.com

renders

the

obligation

immediately demandable. Isaac lost his


right to make use of the period.

Under Art 1198 (2) of the Civil Code, the


debtor shall lose every right to make use
of the period when he does not furnish
to

the

creditor

the

guaranties

or

securities which he has promised.

In 2008, Lita met and married Jaime. They


now have a child of their own.
While on a tour with her former high school
classmates in a remote province of China in
2010, Lita was surprised to see Lito or
somebody who looked exactly like him, but
she was sure it was Lito because of the

VII. Lito was a commercial pilot who flew for


Pacific-Micronesian Air. In 1998, he was the
co-pilot of the airline's Flight MA916 that
mysteriously disappeared two hours after
take-off from Agana, Guam, presumably
over the Pacific Ocean. No trace of the plane

extreme surprise that registered in his face


when he also saw her. Shocked,

she

immediately fled to her hotel and post haste


returned to the country the next day. Lita
now comes to you for legal advice. She asks
you the following questions:

and its 105 passengers and crew was ever


found despite diligent search; Lito himself

VII. (l) If Lito is alive, what is the status of

was never heard of again. Lito left behind

his marriage to Lita? (1%)

his wife, Lita, and their two children.


Never Let The Odds Keep You From Pursuing What You Know In Your Heart You Were Meant To Do.Leroy Satchel Paige

Page 125 of 180

Civil Law Q&As (2007-2013)

hectorchristopher@yahoo.com

dbaratbateladot@gmail.com

(A) The marriage is valid because Lita's


(A) The marriage subsists because the
marital bond has not been terminated by
death.
(B) The marriage was terminated when Lita
married Jaime.
(C) The marriage subsists because Lita's
marriage to Jaime is void.
(D) The marriage is terminated because Lito
is presumed dead after his plane has been
missing for more than 4 years.
(E) The marriage can be formally declared
terminated if Lito would not resurface.
SUGGESTED ANSWER:
(C) The marriage subsists because Lita's
marriage to Jaime is void.
For

the

purpose

of

contracting

the

subsequent marriage under Art 41 of the


Family Code, the spouse present must
institute

summary

proceeding

as

provided in the Family Code for the


declaration of presumptive death of the
absentee, without prejudice to the effect
of

the

reappearance

of

the

absent

spouse.

VII. (2) If Lito is alive, what is the status of


Lita's marriage to Jaime? (1%)

marriage to Lito was terminated upon Lito's


disappearance for more than seven years.

he

shall

be

presumed

dead

doe

all

(B) The marriage is valid. After an absence

purposes, except for those of succession.

of more than 10 years, Lito is already

This provision was not repealed by the

presumed dead for all purposes.

present Family Code. Applying this to

(C) The marriage

the problem, (A) may be correct. (B) may

is void. Lito's mere

absence, however lengthy, is insufficient to

also be correct. (C) and

authorize Lita to contract a subsequent

(D) may also be correct under Art 41 of

marriage.
(D) The marriage is void. If Lito is indeed
alive, his marriage to Lita was never
dissolved

and

they

can

resume

their

the Family Code.

VIII.Which

of

the

following

actions

or

defenses are meritorious: (1%)

marital relations at any time.


(A) An action for recovery of down
SUGGESTED ANSWER:
Any answer is correct.
Under Art 390 of the Civil Code, after an

payment paid under a rescinded oral sale


of real property.
(B) A defense in an action for ejectment that

absence of seven years, it being unknown

the lessor verbally promised to extend or

whether or not the absentee still lives,

renew the lease.

Never Let The Odds Keep You From Pursuing What You Know In Your Heart You Were Meant To Do.Leroy Satchel Paige

Page 126 of 180

Civil Law Q&As (2007-2013)

hectorchristopher@yahoo.com

dbaratbateladot@gmail.com

cancel the contract, the vendee likewise


(C) An action for payment of sum of money

can file an action for the recovery of the

filed against one who orally promised to

down payment on the basis of solution

answer another's debt in case the latter

indebiti.

defaults.
(D) A defense in an action for damages that
the debtor has sufficient, but unliquidated
assets to satisfy the credit acquired when it
becomes due.
(E) None of the above.
SUGGESTED ANSWER:
(A) An action for recovery of down
payment paid under a rescinded oral sale
of real property.
An oral sale of real property is an
unenforceable

contract

under

the

Statute of Frauds. Since, in the problem,


the vendee paid down payment, it takes
it out of the ambit of Statute of Frauds.
The rescission here must be in the sense
of resolution of the reciprocal obligation
arising from the contract of sale. If
rescinded (resolved) by the vendee on
account of the vendors failure to deliver
the thing sold, the parties will go back to
their status prior to the contract. If the
vendor

refuses

to

return

the

down

payment, then the vendee can file an


action to recover the down payment.
If, on the other hand, the vendor and the
vendee mutually agree to rescind i.e.

can be enforced in a court action. (Art


1403, Civil Code)
ALTERNATIVE ANSWER:

(d)

The

fact

that

debtor

has

(E) None of the above.

unliquidated assets does not excuse him

(a) The recovery of the down payments

from paying his debt.

should be made in the same action for

(e) In the technical meaning of rescission

rescission. Otherwise, it would be a

under Art 1191 of the Civil Code will be

ground for dismissal under Rule 2, Sec 4

adhered to, then there is no absolutely

of Rules of Court.

correct answer. Hence, letter E is also a

(b) Lease of a real property is covered by

possible answer.

the Statute of Frauds. Furthermore, it


also consists of interest in real property.

IX. Betty entrusted to her agent, Aida,

Hence, it must be in writing. (Art 1403,

several pieces of jewelry to be sold on

Civil Code)
(c) A contract of guaranty is a promise to
answer for the debt of another and
hence, it is also covered by the Statute
of Frauds. It must be in writing before it

commission with the express obligation to


turn over to Betty the proceeds of the sale,
or to return the jewelries if not sold in a
month's

time.

Instead

of

selling

the

jewelries, Aida pawned them with the

Never Let The Odds Keep You From Pursuing What You Know In Your Heart You Were Meant To Do.Leroy Satchel Paige

Page 127 of 180

Civil Law Q&As (2007-2013)

hectorchristopher@yahoo.com

dbaratbateladot@gmail.com

reimbursed for the pawn value of the


Tambunting

Pawnshop,

and

used

the

money for herself. Aida failed to redeem the


pawned jewelries and after a month, Betty
discovered what Aida had done. Betty
brought criminal charges which resulted in
Aida's conviction for estafa.
Betty thereafter filed an action against
Tambunting Pawnshop for the recovery of
the

jewelries.

Tambunting

raised

the

defense of ownership, additionally arguing


that it is duly licensed to engage in the
pawnshop and lending business, and that it
accepted the mortgage of the jewelry in
good faith and in the regular course of its
business.
If you were the judge, how will you decide
the case? (1%)
(A) I will rule in favor of Betty. My ruling
is based on the Civil Code provision that
one who has lost any movable or has
been unlawfully deprived thereof may
recover it from the person in possession
of the same. Tam bunting's claim of good
faith is inconsequential.
(B)

will

rule

in

favor

of

Betty.

Tambunting's claim of good faith pales into


insignificance

in light

of the

unlawful

deprivation of the jewelries. However, equity


dictates

that

Tambunting

must

be

jewelries.

Although possession of movable property


(C) I will rule in favor of Tambunting. Its

acquired in good faith is equivalent to a

good faith takes precedence over the right

title, nevertheless, one who has lost any

of Betty to recover the jewelries.

movable or has been unlawfully deprived

(D) I will rule in favor of Tambunting. Good

thereof may recover it from the person

faith is always presumed. Tambunting's

in possession of the same. Betty has

lawful acquisition in the ordinary course of

been deprived unlawfully of her jewelries

business coupled with good faith gives it


legal right over the jewelries.
SUGGESTED ANSWER:
(A) I will rule in favor of Betty. My ruling
is based on the Civil Code provision that
one who has lost any movable or has
been unlawfully deprived thereof may
recover it from the person in possession
of the same. Tam bunting's claim of good
faith is inconsequential.

by the estafa committed by Aida. The


pledge of the said jewelries by Aida to
Tambunting pawnshop is void because
the pledgor is not the owner (Art 2085
(2), Civil Code). Tambuntings claim of
good faith is inconsequential, because,
aside from good faith, Tambunting must
prove also that it acquired the jewelries
at a public sale in order to be able to
retain the jewelries until reimbursed by

Never Let The Odds Keep You From Pursuing What You Know In Your Heart You Were Meant To Do.Leroy Satchel Paige

Page 128 of 180

Civil Law Q&As (2007-2013)

hectorchristopher@yahoo.com

dbaratbateladot@gmail.com

dismiss, claiming that the lease on which


Betty the amount of loan including
interest (Art 559, Civil Code).
The only exception the law allows is
when there is acquisition in good faith of
the possessor at a public sale, in which
case, the owner cannot obtain its return
without reimbursing the price (Dizon v.
Suntay, 47 SCRA 160, Sept 29, 1972).

X. Arlene owns a row of apartment houses


in Kamuning, Quezon City. She agreed to
lease Apartment No. 1 to Janet for a period
of 18 months at the rate of P10,000 per
month. The lease was not covered by any
contract. Janet promptly gave Arlene two
(2) months deposit and 18 checks covering
the rental payment for 18 months. This
show of good faith prompted Arlene to
promise Janet that should Arlene decide to
sell the property, she would give Janet the
right of first refusal.
X. (1) Not long after Janet moved in, she
received news that her application for a
Master

of

Laws

scholarship

at

King's

College in London had been approved.


Since her acceptance of the scholarship
entailed a transfer of residence, Janet asked
Arlene

to

payments

return
she

the

made.

advance
Arlene

rental
refused,

prompting Janet to file an action to recover


the payments. Arlene filed a motion to

the action is based, is unenforceable.

seek to enforce any right under the


If you were the judge, would you grant

contract of lease.

Arlene's motion? (1%)


SUGGESTED ANSWER:
(A) Yes, I will grant the motion because the

(D) No. I will not grant the motion

lease contract between Arlene and Janet

because the cause of action does not

was not in writing, hence, Janet may not

seek to enforce any right under the

enforce any right arising from the same

contract of lease.

contract.

Janet is not asking for the continued use

(B) No, I will not grant the motion because


to allow Arlene to retain the advance
payments

would

amount

to

unjust

enrichment.
(C) Yes, I will grant the motion because the
action for recovery is premature; Janet
should first secure a judicial rescission of
the contract of lease.
(D) No. I will not grant the motion

of the leased premises. Moreover, the


contract

is

aside

the

ambit

of

the

Statute of Frauds as the same has


already been partially performed.

X. (2)Assume that Janet decided not to


accept

the

scholarship

and

continued

leasing Apartment No. 1. Midway through

because the cause of action does not


Never Let The Odds Keep You From Pursuing What You Know In Your Heart You Were Meant To Do.Leroy Satchel Paige

Page 129 of 180

Civil Law Q&As (2007-2013)

hectorchristopher@yahoo.com

dbaratbateladot@gmail.com

The right of first refusal involves a


the lease period, Arlene decided to sell

transfer of interest in the real property.

Apartment No. 1 to Jun in breach of her

As such, it is covered by the Statute of

promise to Janet to grant her the right of


first refusal. Thus, Janet filed an action
seeking the recognition of her right of first
refusal, the payment of damages for the
violation of this right, and the rescission of
the sale between Arlene and Jun.
Is Janet's action meritorious? (1%)
(A) Yes, under the Civil Code, a promise to
buy

and

sell

determinate

thing

is

reciprocally demandable.
(B) No, the promise to buy and sell a
determinate thing was not supported by a
consideration.
(C) Yes, Janet's right of first refusal was
clearly violated when the property was not
offered for sale to her before it was sold to
Jun.
(D) No, a right of first refusal involves an
interest over real property that must be
embodied in a written contract to be
enforceable.
(E) None of the above.
SUGGESTED ANSWER:
(D) No, a right of first refusal involves an
interest over real property that must be
embodied in a written contract to be
enforceable.

d) Juridical capacity cannot exist


without capacity to act.

Frauds under Art 1403 (2)(e) of the Civil


Code. It must be in writing in order to be
enforceable.

2.

Which

of

the

following

is

NOT

restriction on ones capacity to act?


a) Minority

2012

Taxation

Law

Exam

MCQ (October 14, 2012)

b) Marriage
c) Deaf-mute
d) Civil Interdiction

1. Which of the following is NOT included in

SUGGESTED ANSWER:

the attributes of juridical capacity?

This question should be disregarded.

a) Juridical capacity is inherent in

(NOTE: There is no correct answer among

every natural person, and therefore

the choices given. All choices are restrictions

it is not acquired.

on ones capacity to act. While Marriage is

b) Juridical capacity is lost only

the only one not mentioned in Articles 38

through death.

and 39 of the NCC as a restriction on

c) Juridical capacity is the fitness to


be the subject of legal relations.
Never Let The Odds Keep You From Pursuing What You Know In Your Heart You Were Meant To Do.Leroy Satchel Paige

Page 130 of 180

Civil Law Q&As (2007-2013)

hectorchristopher@yahoo.com

dbaratbateladot@gmail.com

capacity to act, it restricts the capacity of a

b) Yes, the will is not valid under

married person in cases of adoption.)

Philippine law.
c) No, attestation clause is not an

3. This attribute or incident of a case


determine whether it is a conflict-of-laws
case or one covered by domestic law.

d)

b) Foreign element

governing

law

is

observed by the testator in executing his

d) Forum non conveniens

will. He could have observed Spanish Law

4. The capacity of an heir to succeed shall


be governed by the:
a) national law of the decedents
heirs
b) law of the country where the
decedent was a resident at the time

or Philippine Law (see comment of Tolentino


to Art. 815 NCC in 3Tolentino117, 1992). If
he observed Spanish Law, the opposition is
not correct because the will is valid under
Spanish Law, hence choice (d) is the correct
answer. If he observed Philippine Law, the
opposition

of his death
c) national law of the person who
died
d) law of the country where the
the

the

Note: The facts do not state the Law

c) Jurisdiction

of

No,

Spanish law.

a) Cause of action

properties

act of the testator.

decedent

are

located.
5. Atty. BUKO, a Filipino, executed a will
while he was in Spain. The attestation
clause of the said will does not contain
Bukos signature. It is valid under Spanish
law. At its probate in Manila, it is being
opposed on the ground that the attestation
clause does not contain BUKOs signature.
Is the opposition correct? Choose the best
answer..
a) Yes, because it is a fatal defect.

is

still

not

correct

because

Philippine Law does not require the testator


to sign the Attestation Clause of his will,
said clause not being his act. In such case,
choice (c) is the correct answer).

6. Ramon, a Filipino, executed a will in


Manila, where he left his house and located
in BP Homes Paraaque in favor of his
Filipino

son,

Ramgen.

Ramons

other

children RJ and Ramona, both Turkish


nationals, are disputing the bequest to
Ramgen. They plotted to kill Ramgen.
Ramon learned of the plot, so he tore his
will in two pieces out of anger. Which
statement is most accurate?

a) The mere act of Ramon Sr. is

readable.

immaterial because the will is still


Never Let The Odds Keep You From Pursuing What You Know In Your Heart You Were Meant To Do.Leroy Satchel Paige

Page 131 of 180

Civil Law Q&As (2007-2013)

hectorchristopher@yahoo.com

dbaratbateladot@gmail.com

b) Philippine law
b) The mere act of tearing the will

c) Indonesia law

amounts to revocation.

d) Japanese law

c) The tearing of the will may


amount to revocation if coupled
with intent of revoking it.
d) The act of tearing the will is
material.
7. Even if the applicable law is a foreign
law, a count in the Philippines may be
constrained to apply Philippine law under
any of the following instances, except:
a) when the foreign law, judgment or
contract is contrary to a sound and
important

public

policy

of

the

forum;
b) when the property subject of
the case is located outside of the
Philippines;
c)

when

the

foreign

law

or

judgment is penal in nature;


d)

when

the

foreign

law

is

procedural in nature.
8. If a will is executed by a testator who was
born

Filipino

citizen

but

became

naturalized Japanese citizen at the time of


his

death,

testamentary

what

law

provisions

will
if

govern
the

will

its
is

executed in China and the property being


disposed is located in Indonesia?
a) Chinese law

a) American law
b) Philippine law
9. A Japanese national and a Filipino
national entered into a contract for services
in Thailand. The services will be rendered
in Singapore. In case of breach, what law
will govern?
a) Thailand law
b) Philippine law
c) Singapore law
d) Japanese law
10. Pedro (Filipino) and his wife Jane

c) Canadian law
d) Japanese law
11. A French national revokes his will in
Japan where he is domiciled. He then
changed his domicile to the Philippines
where he died. The revocation of his will in
Japan is valid under Japanese law but
invalid under Philippine law. The affected
heir is a Malaysian national residing in the
Philippines. What law will apply?

(American) executed a joint will in Canada,

a) Japanese law

where such joint will is valid. In case the

b) Philippine law

joint will is probated in Japan, what law

c) French law

will govern the formalities of the joint will?

d) Malaysian law

Never Let The Odds Keep You From Pursuing What You Know In Your Heart You Were Meant To Do.Leroy Satchel Paige

Page 132 of 180

Civil Law Q&As (2007-2013)

hectorchristopher@yahoo.com

dbaratbateladot@gmail.com

construction, Bill will transfer and convey


12. In the absence of contrary stipulation in
a marriage settlement, property relations of
Filipino spouses shall be governed by --a) Philippines laws
b) Law of the place where the
spouses reside
c) Law of the place where the
properties are situated
d) Law of the place where they were
married.
13. The will of a Filipino executed in a
foreign country --a)

cannot

be

probated

in

the

in

the

Philippines;
b)

may

be

probated

Philippines provided that properties


in the estate are located in the
Philippines;
c) cannot be probated before the
death of the testator;
d)

may

be

Philippines

probated
provided

in
it

the
was

executed in accordance with the


laws of the place where the will
was executed.

14. Pedro (Filipino and Bill (American)


entered

into

contract

in

Australia,

whereby it was agreed that Pedro will build


a commercial building for Bill in the
Philippines,

and

in

payment

for

the

his cattle ranch located in Japan in favor of


Pedro.

In

case

Pedro

performs

his

Filipino), by whom she had a daughter,


Regine. In 2009, Regine married James

obligation, but Bill fails or refuses to pay,

(son of Justine with Lea) in California,

what law will govern?

where such marriage is valid.

a) American law
b) Philippine law

15. What is the current status of the

c) Australian law

marriage of Charice and Justine under

d) Japanese law

Philippine laws?

(Facts for item numbers 15-18)


In 1989, Charice (Filipina) and Justine
(American),

were

married

in

the

Philippines. In 1990, they separated and


Justine

went

to

Las

Vegas

where

he

obtained a divorce in the same year. He


then married another Filipina, Lea, in
Canada on January 1, 1992. They had two
(2) sons, James and John (who were both
born in 1992). In 1993, after failing to hear
from Justine, Charice married Bugoy (a

a) Valid
b) Void
c) Voidable
d) Dissolved
(Note: While Art 26 of the FC does not
categorically provide that the first marriage
is dissolved by the divorce obtained by the
foreign spouse abroad, but provides that
such divorce merely gives the Filipino spouse
the capacity to contract a second marriage, it

Never Let The Odds Keep You From Pursuing What You Know In Your Heart You Were Meant To Do.Leroy Satchel Paige

Page 133 of 180

Civil Law Q&As (2007-2013)

hectorchristopher@yahoo.com

is believed that the dissolution of the first


marriage us the necessary consequence of
the foreign divorce.)
16. What id the status of the marriage
between

Charice

and

Bugoy

under

Philippine laws?
a) Valid
b) Void
c) Voidable
d) Unenforceable
17. What is the status of the marriage
between

Charice

and

Bugoy

under

Philippine laws?
a) Valid
b) Void
c) Voidable
d) Unenforceable
18. What is the status of the marriage
between

Regine

and

James

under

Philippine laws?
a) Valid
b) Void
c) Voidable
d) Unenforceable
19. Ricky and Princess were sweethearts.
Princess became pregnant. Knowing that
Ricky is preparing for the examinations,
Marforth, a lawyer and cousin of Princess,
threatened

Ricky

with

dbaratbateladot@gmail.com

complaint for immorality in the Supreme

the

filing

of a

Court, thus preventing him from taking

examinations unless he marries Princess.


As a consequence of the threat, Ricky
married Princess. Can the marriage be
annulled on the ground of intimidation
under Article 45 of the Family Code?
Choose the best answer.
a) Yes, because without the threat,
Ricky

would

not

have

married

Princess.
b) Yes, because the threat to enforce
the claim of Princess vitiates the
consent of Ricky in contracting the
marriage.
c) No, because the threat made by
Marforth is just and legal.

20. Audrey, single, bought a parcel of land


in Malolos City from Franco for P 1Million.
A contract was executed between them
which already vested upon Audrey full
ownership of the property, although payable
in monthly installments for a period of four
(4) years. One (1) year after the execution of
the contract, Audrey got married to Arnel.
They

executed

marriage

settlement

whereby they agreed that their properties


shall be governed by the regime of conjugal
partnership

of

gains.

Thereafter,

subsequent installments were paid from the


conjugal partnership funds. Is the land
conjugal or paraphernal?

d) No, because Marforth is not a


party to the contract of marriage
between Princess and Ricky.
Never Let The Odds Keep You From Pursuing What You Know In Your Heart You Were Meant To Do.Leroy Satchel Paige

Page 134 of 180

Civil Law Q&As (2007-2013)

hectorchristopher@yahoo.com

dbaratbateladot@gmail.com

Australia, a marriage solemnized by a


a) The land is conjugal because the
installments were paid from the
conjugal partnership funds.
b)

The

land

is

paraphernal

because ownership thereof was


acquired before the marriage.
c) The land is both conjugal and
paraphernal funds of installments
were paid from both the personal
funds of Audrey and the conjugal
partnership funds.
d) The land is paraphernal because
it was Audrey who purchased the
same.
21.

Ernesto

donated

mobile

phone

worth P 32,000 to Hubert orally and


delivered the unit to Hubert who accepted.
Which statement is most accurate?
a)

The

donation

is

void

and

Ernesto may get mobile phone


back.
b) The donation is void but Ernesto
cannot get the mobile phone back.
c) The donation is voidable and may
be anulled.
d) The donation is valid.
22. Agay, a Filipino citizen and Topacio, an
Australian
consular
Australia.

citizen,
office

got

of

According

married

the
to

in

Philippines
the

laws

the
in
of

consular official is valid, provided that such


marriage is celebrated in accordance with

d) Valid, because such marriage is


the laws of such consular official. Under
Philippine law, what is the status of the
marriage of Agay and Topacio? Choose the
best answer.
a)

Void,

official

because
only

solemnize

has

the

consular

authority

marriages

to

between

Filipinos.

recognized as valid in the place


where it was celebrated.
(Note: The issues in the problem is whether
or not the fact that one of the parties to the
marriage was an alien constituted absence
of authority or mere irregularity of authority.
The problem only give the choice, letter (a),
in case it is interpreted as absence of

b) Valid, because according to the

authority. The problem does not give a

laws of Australia, such consular

choice in case it is interpreted as an

official has authority to celebrate

irregularity thereby making all the other

the marriage.

answers wrong).

c) Voidable, because there is an


irregularity in the authority of the
consular
marriages.

official

to

solemnize

23. Separation of property between spouses


during the marriage may take place only:
a) by agreement of the spouses.

Never Let The Odds Keep You From Pursuing What You Know In Your Heart You Were Meant To Do.Leroy Satchel Paige

Page 135 of 180

Civil Law Q&As (2007-2013)

hectorchristopher@yahoo.com

dbaratbateladot@gmail.com

b) If one of the spouses has given


ground for legal separation.

b) No trial shall be held without the


6-month cooling off period being

c) Upon order of the court.

observed.

d) If one spouse has abandoned the

c) The spouses will be entitled to live

other.

separately upon the start of the


trial.

24. The husband may impugn the

d) The prosecuting attorney has

legitimacy of his child but not on the

to conduct his own investigation.

ground that:
a)

the

wife

is

suspected

of27. A husband by chance discovered

infidelity.
b) the husband had a serious illness

hidden treasure on the paraphernal


property of his wife. Who owns the

that prevented him from engaging in

discovered treasure?

sexual intercourse.

a) The half pertaining to the

c) they were living apart.

husband (finder) belongs to the

d) he is physically incapable of

conjugal partnership.

sexual intercourse.

b) The half pertaining to the wife (as


owner) belongs to the conjugal

25. A marriage is void if:

partnership.

a) solemnized with a marriage

c) One half shall belong to the

license issued without complying

husband as finder and the other

with the required 10-day posting.

half shall belong to the wife as

b) solemnized by a minister whom

owner of the property.

the parties believe to have the

d) a and b

authority.
c) between parties both 23 years of

28. Which of the following marriages is void

age but without parental advice.

for reasons of public policy?

d) none of the above

a) Between brothers and sisters,


whether of the full or half blood.

26. In legal separation, which is not

b) Between step-parents and step

correct?

children.

a) The aggrieved spouse may file the

c) Between parents-in-law and

action within five (5) years from the

children-in-law.

time of the occurrence of the cause.

d) b and c

Never Let The Odds Keep You From Pursuing What You Know In Your Heart You Were Meant To Do.Leroy Satchel Paige

Page 136 of 180

Civil Law Q&As (2007-2013)

hectorchristopher@yahoo.com

dbaratbateladot@gmail.com

d) The donation must be made in


29. The following constitute the different
circumstances or case of fraud which will
serves as ground for the annulment of a
marriage, except?
a) Non-disclosure of the previous
conviction by final judgment of the
other party of a crime involving
moral turpitude.
b)

Concealment

of

sexually-

transmissible disease, regardless of


its nature, existing at the time of the
marriage.
c) Concealment of drug addiction,
habitual alcoholism, homosexuality
or lesbianism existing at the time of
marriage.
d) Concealment by the wife or the
husband of the fact of sexual
relations prior to the marriage.

30. Which of the following is not a requisite


for a valid donation propter nuptias?
a) The donation must be made
before

the

celebration

of

the

shall

be

marriage.
b)

The

donation

automatically revoked in case of


non-celebration of the marriage.
c) The donation must be made in
consideration of the marriage.

favor of one or both of the future


spouses.
31. Who are illegitimate children?

a) Filiation has been recognized by


a)

Children

conceived

or

born

birth appearing in the civil register

outside a valid marriage.


b) Children born under a valid
marriage, which was later declared
void because of the psychological
incapacity of either or both of the

c) Children conceived and born

d) Children born under a valid


but

the

parents

later

obtained a legal separation.

surname of his father when his filiation is

instances, except:

c) Private handwritten instrument is


made by the father acknowledging

any

of

the

d) Affidavit by the mother stating

33. Under RA 8043, an adopter is required


to be at least ____ years old and ____ years
older than the child to be adopted at the

32. An illegitimate child may use the

in

father in a public document.

the name of his true father.

outside a valid marriage.

established

b) Admission of filiation by the

his filiation.

spouses.

marriage,

the father through the record of

following

time of the application unless the adopter is


the parent by nature of the child.
a) 30 and 15
b) 27 and 16

Never Let The Odds Keep You From Pursuing What You Know In Your Heart You Were Meant To Do.Leroy Satchel Paige

Page 137 of 180

Civil Law Q&As (2007-2013)

hectorchristopher@yahoo.com

dbaratbateladot@gmail.com

c) 50 and 10

d) Subjects the child or allows him

d) 18 and 15

to

be

subjected

to

acts

of

lasciviousness.
34. Under RA 8043, a child qualified to be
adopted is any person below _____ years

37. Which

old.

is wrong?

of the following

statements

a) 18

a) The possessor in bad faith shall

b) 21

reimburse the fruits received and

c) 15

those which the legitimate possessor

d) 16

could have received.


b) The possessor in bad faith has

35. Which of the following DOES NOT

right

result in permanent termination of parental

the

a) Death of the parents.

d) The possessor in bad faith is

penalty of civil interdiction.

entitled to a refund of useful


expenses.

36. The court, in an action filed for the

harshness or cruelty.
b) Gives the child corrupting orders,
counsel or example.
c) Compels the child to take up a
course in college against his/her
will.

and

expenses.

crime which carries with it the

a) Treats the child with excessive

gathering

entitled to a refund of ornamental

d) Conviction of the parents of a

except:

production,

c) The possessor in bad faith is not

c) Emancipation of the child.

authority commits any of the following acts,

for

preservation of the fruits.

b) Death of the child.

the parent or the person exercising parental

reimbursement

necessary expenses and those for

authority?

purpose, may suspend parental authority if

of

38.

Which

phrase

most

accurately

completes the statement The expenses


incurred in improvements for the luxury or
mere pleasure shall not be refunded to thew
possessor in bad faith:
a) but he may remove the objects
for which such expenses have
been incurred, provided that the
thing suffers no injury thereby,

and that the lawful possessor does


not prefer to retain them.

Never Let The Odds Keep You From Pursuing What You Know In Your Heart You Were Meant To Do.Leroy Satchel Paige

Page 138 of 180

Civil Law Q&As (2007-2013)

hectorchristopher@yahoo.com

dbaratbateladot@gmail.com

41. Action to recover real property based on


b) and he may not remove the
objects for which such expenses
have been incurred.
c) and he may not remove the
objects for which such expenses
have been incurred, unless he pays
the value they may have at the time
he entered into possession.
d) but he may remove the objects for
which such expenses have been
incurred.
39. The following are the limitations on the
right of ownership imposed by the owner
himself, except:
a) Will/Succession
b) Mortgage
c) Pledge
d) Lease
40. A plenary action for the recovery of the
possession

of

real

estate,

upon

mere

allegation and proof of a better right


thereto, and without allegation of proof of
title. This action can only be brought after
the expiration of one (1) year. What action is
being referred to?
a) Accion publiciana
b) Accion reinvindicatoria
c) Accion interdictal
d) Quieting of Title

ownership. Here, the object is the recovery

43. The following things are property of


of the dominion over the property as owner.

public dominion, except:


a) ports and bridges constructed by

What action is being referred to?

the State.

a) Accion publiciana

b) vehicles and weapons of the

b) Accion reinvindicatoria

Armed Forces of the Philippines.

c) Accion interdictal

c) rivers.

d) Quieting of Title

d) lands reclaimed by the state from


the sea.

42. A summary action to recover physical


or material possession only and must be
brought within one (1) year from the time
the cause of action arises. What action is
being referred to?
a) Accion publiciana

SUGGESTED ANSWER:
This

question

should

be

disregarded

because there is no correct answer.


(Note: At first glance, one gets the impression

b) Accion reinvindicatoria

that vehicles and weapons of the AFP are

c) Accion interdictal

not property of the public domain. But they

d) Quieting of Title

are actually property of the public dominion


under the second paragraph of Art 420 of

Never Let The Odds Keep You From Pursuing What You Know In Your Heart You Were Meant To Do.Leroy Satchel Paige

Page 139 of 180

Civil Law Q&As (2007-2013)

hectorchristopher@yahoo.com

the NCC. Property of the state which are not


for public use but are intended for some
public service are properties of the public
dominion. While the vehicles and weapons
of the AFP are not for public use, they are
used for the defense of the State which is a
public service.)
44. Which

of the following

statements

is wrong?
a) patrimonial property of the
state, when no longer intended for
public use or for public service,
shall become property of public
dominion.
b) all property of the State, which is
not

of

public

dominion,

is

patrimonial property.
c) The property of provinces, cities
and municipalities is divided into
property

for

public

use

and

patrimonial property.
d)

Property

is

either

of

public

dominion or of private ownership.


45. The

following

cannot

ask

for

the

reduction of inofficious donation, except:


a) Creditors of the deceased
b) Devisees or legatees
c) Compulsory heirs of the donor
d) The surviving spouse of the
donee.

dbaratbateladot@gmail.com

46. Donation is perfected from the moment


---

a) the donee accepts the donation.


b) the donor executes the deed of

48. It is a conduct that may consist of


giving, doing, or not doing something.
a) Obligation

donation.

b) Juridical necessity

c) the donor knows of the donees

c) Prestation

acceptance even if the latter has

d) Contract

not received the copy of the deed


of donation.
d) the

donee

donor

has

confirms

learned

the

that

the

formers

acceptance.
47. The following are the elements of an
obligation, except:
a) Juridical/Legal Tie
b) Active subject
c) Passive subject
d) Consideration

49. It is
from lawful,

a juridical relation arising


voluntary and unilateral acts

based on the principle that no one should


unjustly enrich himself at the expense of
another.
a) Quasi-contract
b) Quasi-delict
c) Cotract
d) Delict
50. The following are the elements of quasidelict, except:

Never Let The Odds Keep You From Pursuing What You Know In Your Heart You Were Meant To Do.Leroy Satchel Paige

Page 140 of 180

Civil Law Q&As (2007-2013)

hectorchristopher@yahoo.com

dbaratbateladot@gmail.com

a) Act or omission

54. It is an international evasion of the

b) Fault/negligence

faithful performance of the obligation.

c) Damage/injury

a) Negligence

d) Pre-existing contract

b) Fraud
c) Delay

51. A debtor is liable for damages in case of

d) Mistake

delay if he is guilty of any of the following,


55. The following are the requisites of

except:

fortuitous event, except:

a) default (mora)
b) mistake

a) Cause is independent of the will

c) negligence (culpa)

of the debtor.

d) breach through contravention of

b)

the tenor thereof

is unforeseeable/unavoidable.

The

event

c) Occurrence renders it absolutely


52. This term refers to a delay on the part

impossible for the debtor to fulfill his

of both the debtor and creditor in reciprocal

obligation

obligations.

impossibility must be absolute not

in

normal

manner;

a) Mora accipiendi

partial, otherwise not force majeure.

b) Mora solvendi

d)

c) Compensation morae

solvendi, except:
a) Obligation pertains to the debtor
determinate,

due,

demandable, and liquidated.


b) Obligation was performed on its
maturity date.
c) There is judicial or extrajudicial
demand by the creditor.
d) Failure of the debtor to comply
with such demand.

to

the

creditor.

53. The following are the requisites of mora

is

contributed

aggravation of the injury to the

d) Solution indibiti

and

Debtor

56. A debtor may still be held liable for loss


or damages even if it was caused by a
fortuitous event in any of the following
instances, except:
a) The debtor is guilty of dolo, malice
or bad faith, has promised the same
thing to two or more persons who do
not have the same interest.
b) The debtor contributed to the
loss.

c) The thing to be delivered is


generic.

Never Let The Odds Keep You From Pursuing What You Know In Your Heart You Were Meant To Do.Leroy Satchel Paige

Page 141 of 180

Civil Law Q&As (2007-2013)

hectorchristopher@yahoo.com

dbaratbateladot@gmail.com

b) No, because the creditor may


d) The creditor is guilty of fraud,

proceed against any one of the

negligence

solidary debtors or some or all of

or

contravened

delay

the

or

tenor

if
of

he
the

obligation.
57.

Buko,

themselves

Fermin
solidarily

and
to

Toti
pay

bound

Ayee

the

amount of P 5,000.00. Suppose Buko paid


the obligation, what is his right as against
his co-debtors?
a)

Buko

cas

ask

for

reimbursement from Fermin and


Toti.
b) Buko can sue Fermin and Toti for
damages.
c) Buko can sue for rescission.
d) Buko can claim a refund from
Ayee.
58.

Buko,

Fermin

and

Toti

bound

themselves solidarily to pay Ayee the sum of


P 10,000.00. When the obligation became
due and demandable, Ayee sued Buko for
the payment of the P 10,000.00. Buko
moved to dismiss on the ground that there
was failure to implead Fermin and Toti who
are indispensable parties. Will the motion
to dismiss prosper? Why?
a) Yes, because Fermin and Toti
should have been impleaded as their
obligation is solidary.

them simultaneously.

c) No, because in solidary obligation


any one of the solidary debtors can

c) No, because a motion to dismiss

pay the entire debt.

is a prohibited pleading.
d) Yes, because Fermin and Toti

d) Yes, because Fermin and Toti will

should also pay their share of the

be unduly enriched at the expense

obligation.

of Buko.

59. Buko, Fermin and Toti are solidarily

60. Buko, Fermin and Toti are solidary

debtors of Ayee. Twelve (12) years after the

debtors

obligation became due and demandable,

300,000.00 which has fallen due. The

Buko paid Ayee and later on asked for

creditor has, however, condoned Fermins

reimbursement

entire share in the debt. Since Toti has

of

Fermins

and

Totis

shares. Is Buko correct? Why?


a) No, because the obligation has
already prescribed.
b) Yes, because the obligation is
solidary.

under

loan

obligation

of

become insolvent, the creditor makes a


demand on Buko to pay the debt. How
much, if any, may Buko be compelled to
pay?
a) P 200.000.00
b) P 300,000.00
c) P 100,000.00

Never Let The Odds Keep You From Pursuing What You Know In Your Heart You Were Meant To Do.Leroy Satchel Paige

Page 142 of 180

Civil Law Q&As (2007-2013)

hectorchristopher@yahoo.com

dbaratbateladot@gmail.com

and also of the same quality if the


latter has been stated.

d) P 150,000.00

c) That the two (2) debts are not


61. Dina bought a car from Jai and
delivered a check in payment of the same.
Has Dina paid the obligation? Why?
a) No, not yet. The delivery of
promissory

notes

payable

to

order, or bills of exchange or


other mercantile documents shall
produce the effect of payment
only when they have been cashed,
or when through the fault of the
creditor they have been impaired.
b) Yes, because a check is a valid
legal tender of payment.
c) It depends. If the check is a
managers check or cashiers check
it will produce the effect of payment.
If

its

an

ordinary

check,

no

payment.
d) Yes, because a check is as good
as cash.
62. The following are the requisites of legal
compensation, except:
a) That each of the obligors is bound
principally and that he be the same
time a principal creditor of the
other.
b) That both debts consist in a sum
of money, or if the things due are
consumable, they be the same kind,

yet due.

also to the natural consequences that flow


d) That they be liquidated and

out of such agreement.


a) Obligatory force of contracts

demandable.

b) Mutuality of contracts
63. Which

of the following

statements

is correct?
a) All contracts are perfected by
mere consent.
b) All contracts are perfected by
delivery of the object.
c) All contracts are required to be in
writing.
d) All contracts are required to
have a valid consideration.
64. It is a principle which holds that parties
are bound not only by what has been

c) Autonomy of contracts
d) Relativity of contracts
65. It is a principle which holds that
contracts must be binding to both parties
and its validity and effectivity can never be
left to the will of one of the parties.
a) Obligatory force of contracts
b) Mutuality of contracts
c) Autonomy of contracts
d) Relativity of contracts
66. It refers to
is binding

the rule that a contract


not
only between

expressly provided for in the contract but


Never Let The Odds Keep You From Pursuing What You Know In Your Heart You Were Meant To Do.Leroy Satchel Paige

Page 143 of 180

Civil Law Q&As (2007-2013)

hectorchristopher@yahoo.com

a)

parties but extends to the heirs, successors


in interest, and assignees of the parties,
provided

that

the

contract

involved

transmissible rights by their nature, or by


stipulation or by law.
a) Obligatory force of contracts
b) Mutuality of contracts
c) Autonomy of contracts
d) Relativity of contracts
67. It is rule which holds that the freedom
of the parties to contract includes the
freedom

to

stipulate,

provided

the

stipulations are not contrary to law, morals,


good customs, public order or public policy.
a) Obligatory force of contracts
b) Mutuality of contracts
c) Autonomy of contracts
d) Relativity of contracts
68. The following are the ways by which
innominate

contracts

are

regulated, except:
a) By the stipulation of the parties.
b)

By the general principles of

quasi-contracts and delicts


c) By the rules governing the most
analogous nominate contracts.
d) By the customs of the place.
69. An offer becomes ineffective on any of
the following grounds, except:

Death,

dbaratbateladot@gmail.com

civil

interdiction,

insanity/insolvency of either party


before acceptance is conveyed.

specific acceptance.
d) Advertisements

b) Acceptance of the offer by the

are only

offeree.

for

invitations

not

of the offer, which becomes counter-

highest/lowest bidder,

offer.

appears otherwise.
Subject

to

make

proposals and the advertiser is

c) Qualified/conditional acceptance

d)

Bidders

matter

becomes

71.

bound

The

to

accept

the
unless it
are solemn

following

illegal/impossible before acceptance


contracts (Contracts which must appear in

is communicated.

writing), except:
70. Which

of the following

statements

a) Donations of real estate or of


movables

is correct?

if

the

value

a) Offers in interrelated contracts

exceeds P 5,000.00.

are perfected upon consent.

b) Stipulation to pay interest in

b) Offers in interrelated contracts

loans.

require a single acceptance.

c) Sale of land through an agent

c) Business
are definite

(authority must be in writing).

advertisements
offers that require

Never Let The Odds Keep You From Pursuing What You Know In Your Heart You Were Meant To Do.Leroy Satchel Paige

Page 144 of 180

Civil Law Q&As (2007-2013)

d)

hectorchristopher@yahoo.com

Construction contract of a74. The following are the characteristics of

building.
72.

The

dbaratbateladot@gmail.com

following

a voidable contract, except:


a) Effective until set aside.
are

rescissible

b) May be assailed/attacked only in

contracts, except:

an action for that purpose.

a) Entered into by guardian

c) Can be confirmed or ratified.

whenever ward suffers damage more

d) Can be assailed only by either

than of value of property.

party.

b) Agreed upon in representation of


absentees, if absentee suffers lesion

75.

The

following

by more than of value of property.

contracts, except:

are

c) Contracts where fraud is

a) Pactum commissorium

committed on

b) Pactum de non alienando

creditor (accion

pauliana).

c) Pactum leonina

d) Contracts entered into by

d) Pacto de retro

void

minors.
76. The borrower in a contract of loan or
73. The following are the requisites before a

mutuum must pay interest to the lender.

contract entered into in fraud of creditors

a) If there is an agreement in

may be rescinded, except:

writing to the effect.

a) There must be credited existing

b) As a matter of course.

prior to the celebration of the

c) If the amount borrowed is very

contract.

large.

b) There must be fraud, or at least,

d) If the lender so demands at the

the intent to commit fraud to the

maturity date.

prejudice of the creditor seeking


rescission.

77. The liability of the school, its

c) The creditor cannot in any legal

administrators and teachers, or the

manner collect his credit (subsidiary

individual, entity or institution engaged in

character of rescission)

child care over the minor child or damage

d) The object of the contract must

caused by the acts or omissions of the

be legally in the possession of a

unemancipated minor while under their

rd

person in good faith.

supervision, instruction or custody shall be:


a) Joint and subsidiary
b) Principal and solidary

Never Let The Odds Keep You From Pursuing What You Know In Your Heart You Were Meant To Do.Leroy Satchel Paige

Page 145 of 180

Civil Law Q&As (2007-2013)

hectorchristopher@yahoo.com

dbaratbateladot@gmail.com

82. An obligation which is based on equity


c) Principal and joint
d) Subsidiary and solidary.
78. The creditor has the right to the fruits
of the thing from the time:
a) the thing is delivered.
b) the obligation to deliver the
things arises.
c) the contract is perfected.
d) the fruits are delivered.
79. If one of the parties to the contract is
without juridical capacity, the contract is:
a) voidable
b) rescissible
c) void
d) unenforceable
80. When both parties to the contract are
minors, the contract is:
a) voidable
b) rescissible
c) void
d) unenforceable
81. When the consent of one of the parties
was vitiated, the contract is:
a) voidable
b) rescissible
c) void
d) unenforceable

and natural law is known as:


a) pure

b) valid because all of the essential


b) quasi-contract

requisites of a contract are present.

c) civil

c) unenforceable because Michael

d) natural

Fermin had no authority but he


sold the car in the name of Mr.

83.

Consent

representation

was
of

given
another

by

one

in

but

without

authority. The contract is:


a) voidable
b) rescissible

Lacas, the owner.


d) rescissible because the contract
caused lesion to Atty. Buko.
85. Which of the following contracts is void?

c) void

a) An oral sale of a parcel of land.

d) unenforceable

b) A sale of land by an agent in a

84. Michael Fermin, without the authority


of Pascual Lacas, owner of a car, sold the
same car in the name of Mr. Lacas to Atty.
Buko. The contract between Atty. Buko and
Mr. Lacas is --a) void because of the absence of

public

instrument

where

his

authority from the principal is


oral.
c) A donation of a wrist watch
worth P 4,500.00.

consent from the owner, Mr. Lacas.


Never Let The Odds Keep You From Pursuing What You Know In Your Heart You Were Meant To Do.Leroy Satchel Paige

Page 146 of 180

Civil Law Q&As (2007-2013)

hectorchristopher@yahoo.com

dbaratbateladot@gmail.com

b) If Aligada refused to deliver the


land, Balane may successfully sue

d) A relatively simulated contract


86. Which of the following expresses a
correct principle of law? Choose the best
answer.
a) Failure to disclose facts when
there is a duty to reveal them, does
not constitute fraud.
b) Violence or intimidation does not
render

contract

annullable

if

employed not by a contracting party


but by a third person.
c) A threat to enforce ones claim
through competent authority, if the
claim is legal or just, does not vitiate
consent.
d)

Absolute

simulation

of

contract always results in a void


contract.

87. Aligada orally offered to sell his twohectare rice land to Balane for P 10Million.
The

offer

was

orally

accepted.

By

agreement, the land was to be delivered


(through execution of a notarized Deed of
Sale) and the price was to be paid exactly
one-month

from

their

oral

agreement.

Which statement is most accurate?


a) If Aligada refuses to deliver the
land on the agreed date despite
payment by Balane, the latter may
not

successfully

sue

because the contract is oral.

Aligada

the contract are not transmissible


by their nature, or by stipulation or

for fulfillment of the obligation even

by provision of law.

if he has not tendered payment of

c) If a contract should contain some

the purchase price.

stipulation

c) The contract between the parties

person,

is rescissible.
d)

The

contract

between

may

of

third

demand

provided

its
he

obligor before its revocation.


d)

In

contracts

creating

real

rights, third persons who come

statements

into possession of the object of

is wrong?
a) Creditors are protected in cases of

the

contracts intended to defraud them.

thereby.

b)

favor

communicated his acceptance to the

by the parties.
of the following

he

fulfillment

the

parties is subject to ratification

88. Which

in

Contracts

take

effect

contract

are

not

bound

only

between the parties, their assign

89.

Which

phrase

most

accurately

and heirs, except in case where the

completes the statement Any third person

rights and obligations arising from

who induces another to violate his contract:

Never Let The Odds Keep You From Pursuing What You Know In Your Heart You Were Meant To Do.Leroy Satchel Paige

Page 147 of 180

Civil Law Q&As (2007-2013)

hectorchristopher@yahoo.com

dbaratbateladot@gmail.com

c) Right to inherit
a) shall be liable for damages only if
he is a party to the same contract.
b) shall be liable for damages to
the other contracting party.
c) shall not be liable for damages to
the other contracting party.
d) shall not be liable for damages if
the parties are in pari delicto.
90. The requisites of succession are as
follows, except:
a) Death of decedent
b) Transmissible estate
c)

Existence

and

capacity

of

successor, designated by decedent


or law
d) Payment of Taxes
91. The characteristics of succession are as
follows, except:
a) It is a legal contract.
b)

Only

property,

rights

and

obligations to the extent of the value


of the inheritance are transmitted.
c) The transmission takes place only
at the time of death.
d) The transmission takes place
either by will or by operation of law.
92. The following rights are extinguished by
death, except:
a) Legal support
b) Parental authority

d) Agency

a) Entirely written;
b) Dated;
93. The attestation clause contains the

c) Signed by testator himself

following, except:

d) Notarized by a notary public.

a) the number of pages used;


b) that the testator signed or caused
another to sign the will and every
page thereof in the presence of the

a) The formalities required by law

b)

c) notary public;
the

disallowance of wills, except:

have not been complied with.

instrumental witnesses;

d)

95. The following are the grounds for

instrumental

witnesses

The

testator

was

insane

or

mentally incapable of making will.

witnessed and signed the will and

c) The will was executed through

all the pages thereof in the presence

force or under duress, or influence

of the testator and one another.

of fear or threats.
d) The will contains an attestation

94.

The

following

are

the

formalities

clause.

required in the execution of holographic


will, except:
Never Let The Odds Keep You From Pursuing What You Know In Your Heart You Were Meant To Do.Leroy Satchel Paige

Page 148 of 180

Civil Law Q&As (2007-2013)

hectorchristopher@yahoo.com

96. It is the omission in the testators will of


one, some or all of the compulsory heirs in
direct line, whether living at the time of
execution of the will or born after the death
of the testator. What principle is being
referred to?
a) reserva troncal
b) preterition
c) fideicommissary
d) disposicion captatoria
97.

Any

disposition

made

upon

the

condition that the heir shall make some


provision in his will in favor of the testator
or of any other person shall be void. Here,
both the condition and the disposition are
void. What principle is being referred to?
a) reserva troncal
b) preterition
c) fideicommissary
d) disposicion captatoria
98.

Which

phrase

most

accurately

completes the statement If at the time the


contract of sale is perfected, the thing
which is the object of the contract has been
entirely lost:
a) the buyer bears the risk of loss.
b) the contract shall be without
any effect.
c) the seller bears the risk of loss.
d) the buyer may withdraw from the
contract.

dbaratbateladot@gmail.com

99. A contract granting a privilege to a


person,

for

which

he

has

paid

authority

is

not

reduced

into

consideration, which gives him the right to

writing.

buy

c) Sale of EGMs car by KRP, a

certain

merchandise

or

specified

property, from another person, at anytime

person stranger to EGM, without

within the agreed period, at a fixed price.

EGMs consent or authority.

What contract is being referred to?

d) Sale of EGMs piece of land by

a) Option Contract

KRP, a person stranger to EGM,

b) Contract to Sell

without EGMs consent or authority.

c) Contract of Sale
d) Lease

2011 Taxation Law Exam


100. Which of the following contracts of
sale is void?

MCQ (November 13, 2011)

a) Sale of EGMs car by KRP, EGMs

(1)When does a declaration of absence of a

agent,

missing person take effect?

whose

authority

is

not

reduced into writing.

(A) Immediately from the issuance of

b) Sale of EGMs piece of land by

the declaration of absence.

KRP,

EGMs

agent,

whose

Never Let The Odds Keep You From Pursuing What You Know In Your Heart You Were Meant To Do.Leroy Satchel Paige

Page 149 of 180

Civil Law Q&As (2007-2013)

hectorchristopher@yahoo.com

dbaratbateladot@gmail.com

(4) Upon the proposal of a third person, a


(B) 3 months after the publication of

without the latters consent. The creditor

the declaration of absence.


(C) 6 months after the publication
of the declaration of absence.
(D) 15 days from the issuance of the
declaration of absence.
(2) The authority that school administrators
exercise over school children under their
supervision,

instruction,

or

custody

is

called
(A) legal parental authority.
(B) substitute parental authority.
(C) ordinary parental authority.
(D) special parental authority.
(3) Can future inheritance be the subject of
a contract of sale?
(A)

No,

since

it

will

put

the

predecessor at the risk of harm from


a tempted buyer, contrary to public
policy.
(B) Yes, since the death of the
decedent is certain to occur.
(C) No, since the seller owns no
inheritance while his predecessor
lives.
(D) Yes, but on the condition that
the amount of the inheritance can
only

be

ascertained

after

the

obligations of the estate have been


paid.

new debtor substituted the original debtor

avoid unjust enrichment


part.

accepted the substitution. Later, however,


the

new debtor became insolvent and

defaulted in his obligation. What is the


effect of the new debtors default upon the
original debtor?
(A) The original debtor is freed of
liability since novation took place
and

this

relieved

him

of

his

on his

(5) Lennie bought a business class ticket


from Alta Airlines. As she checked in, the
manager downgraded her to economy on
the ground that a Congressman had to be
accommodated
Lennie

in

suffered

the
the

business

class.

discomfort

and

embarrassment of the downgrade. She sued

obligation.

the airlines for quasi-delict but Alta Airlines

(B) The original debtor shall pay or

countered

perform the obligation with recourse

governed by a contract between them, no

to the new debtor.

quasi-delict could arise.

(C) The

original

debtor

remains

liable since he gave no consent to


the substitution.
(D) The original debtor shall pay or
perform 50% of the obligation to

that,

since

her

travel

Is the

was

airline

correct?
(A) No, the breach of contract may
in fact be tortious as when it is
tainted as in this case with

Never Let The Odds Keep You From Pursuing What You Know In Your Heart You Were Meant To Do.Leroy Satchel Paige

Page 150 of 180

Civil Law Q&As (2007-2013)

hectorchristopher@yahoo.com

dbaratbateladot@gmail.com

wedding, however, Y suddenly died of heart


arbitrariness, gross bad faith, and
malice.
(B) No, denying Lennie the comfort
and amenities of the business class
as provided in the ticket is a tortious
act.
(C) Yes, since the facts show a
breach of contract, not a quasidelict.
(D)

Yes,

since

quasi-delict

presupposes the absence of a preexisting

contractual

relation

between the parties.


(6)

Which

of

the

following

is

an

indispensable requirement in an action for


"quieting of title" involving real property?
The plaintiff must
(A) be in actual possession of the
property.
(B) be the registered owner of the
property.
(C) have legal or equitable title to
the property.
(D) be the beneficial owner of the
property.
(7) X and Y were to marry in 3 months.
Meantime,

to

express

his

affection,

donated a house and lot to Y, which


donation X wrote in a letter to Y. Y wrote
back, accepting the donation and took
possession of the property. Before the

attack. Can Ys heirs get the property?

the company of handsome boys. What legal


(A) No, since the marriage did not

remedy does Lily have?

take place.

(A) She can file an action for

(B) Yes, since all the requisites of a

annulment of marriage on ground

donation

of fraud.

of

an

immovable

are

present.

(B) She can seek a declaration of

(C) No, since the donation and its

nullity of the marriage based on

acceptance are not in a public

Renes psychological incapacity.


(C) She can go abroad and file for

instrument.

divorce in a country that can grant

(D) Yes, since X freely donated the

it.

property to Y who became its owner.

(D) She has none since she had the


(8) Rene and Lily got married after a brief

opportunity to examine the goods

courtship. After one month, Lily discovered

and freely entered into the marriage.

that while Rene presented himself as a


macho man he was actually gay. He would
not go to bed with her. He kept obscene
magazines of nude men and always sought

(9)

Lucio

executed

simple

deed

of

donation of P50 million on time deposit


with a bank in favor of A, B, C, D, and E,

Never Let The Odds Keep You From Pursuing What You Know In Your Heart You Were Meant To Do.Leroy Satchel Paige

Page 151 of 180

Civil Law Q&As (2007-2013)

hectorchristopher@yahoo.com

Raffys
without indicating the share of each donee.
All the donees accepted the donation in
writing. A, one of the donees, died. Will B,
C, D, and E get As share in the money?
(A) Yes, accretion will automatically
apply to the joint-donees in equal
shares.
(B) Yes, since the donors intention
is to give the whole of P50 million to
the jointdonees in equal shares.
(C) No, A"s share will revert to the
donor

because

accretion

applies

only if the joint-donees are spouses.


(D) No, As share goes to his heirs
since

the

donation

did

not

provide for reversion to donor.

(10) Raul, Ester, and Rufus inherited a 10hectare land from their father. Before the
land could be partitioned, however, Raul
sold his hereditary right to Raffy, a stranger
to the family, for P5 million. Do Ester and
Rufus have a remedy for keeping the land
within their family?
(A) Yes, they may be subrogated to
Raffys right by reimbursing to
him

within

the

required

time

what he paid Raul.


(B) Yes, they may be subrogated to
Raffys right provided they buy him
out before he registers the sale.
(C) No, they can be subrogated to

dbaratbateladot@gmail.com

right

conformity.

only

with

his

(A) Its cause comes from the guilt


(D)

No,

since

impediment

to

was

no

of a spouse in a legal separation

selling

his

case, the innocent-spouse having

there
Raul

inheritance to a stranger.

died.
(B) The truth of its cause is denied

(11) When one exercises a right recognized

and

by law, knowing that he thereby causes an

evidence.

injustice to another, the latter is entitled to

(C) Its cause is not authorized by

recover damages. This is known as the

the law.

principle of

(D) Its cause is not specified.

not

sufficiently

proved

by

(A) res ipsa loquitur.


(B) damnum absque injuria.

(13) Manuel came to Manila and married

(C) vicarious liability.

Marianne. Unknown to Marianne, Manuel

(D) abuse of rights.

had been previously convicted in Palawan of


theft and served time for it. After Marianne

(12) Which of the following is NOT a basis

learned of his previous conviction, she

for rendering a disinheritance defective or

stopped living with him. Can Marianne seek

imperfect?

the annulment of the

Never Let The Odds Keep You From Pursuing What You Know In Your Heart You Were Meant To Do.Leroy Satchel Paige

Page 152 of 180

Civil Law Q&As (2007-2013)

hectorchristopher@yahoo.com

dbaratbateladot@gmail.com

(C) The children born before and


marriage based on Manuels nondisclosure
of his previous crime?
(A) No, since the assumption is that
marriage forgives all past wrongs.
(B) Yes, since the non-disclosure
of that crime is the equivalent of
fraud,

which

is

ground

for

annulment.
(C) No, in case of doubt, the law
must be construed to preserve the
institution of marriage.
(D) No, since Manuel already served
the penalty for his crime.
(14) Arthur and Helen, both Filipinos, got
married and had 2 children. Arthur later
worked in Rome where he acquired Italian
citizenship. He got a divorce from Helen in
Rome but, on returning to the Philippines,
he realized his mistake, asked forgiveness of
his wife, and resumed living with her. They
had 2 more children. What is the status of
their 4 children?
(A) The children born before the
divorce are legitimate but those
born after it are not since Arthur
got the divorce when he had
ceased to be a Filipino.
(B) The divorce rendered illegitimate
the children born before it since the
marriage that begot them had been
nullified.

after the divorce are all legitimate

(16) The liability of the partners, including


since Philippine law does not
recognize divorce.
(D) All the children are legitimate
since they were born of the same
father and mother.

industrial

partners

for

partnership

contracts entered into in its name and for


its account, when all partnership assets
have been exhausted is
(A) Pro-rata.
(B) Joint.

(15) Who can make a donation?


(A) All persons who can enter into
contracts and dispose of their
property.
(B) All persons who are of legal age
and suffer from no civil interdiction.
(C) All persons who can make a last
will and testament.
(D) All persons, whether natural or
artificial, who own property.

(C) Solidary.
(D) Voluntary.
(17) When can a missing person who left
someone to administer his property be
declared an absentee by the court? When
he has been missing for
(A) 2 years from the receipt of the
last news about him.
(B) 7 years from the receipt of the
last news about him.

Never Let The Odds Keep You From Pursuing What You Know In Your Heart You Were Meant To Do.Leroy Satchel Paige

Page 153 of 180

Civil Law Q&As (2007-2013)

hectorchristopher@yahoo.com

dbaratbateladot@gmail.com

given the agent. The agent executed the


(C) 10 years from the receipt of the
last news about him.
(D) 5 years from the receipt of the
last news about him.

(18) Which of the following claims against


the debtor enjoys preference over the others
with respect to his specific immovable
property and real rights?
(A) Unpaid price of real property
sold, upon the immovable property.
(B) Mortgage credits recorded in the
registry

of

property,

upon

the

mortgaged real estate.


(C) Taxes due, upon the land or
building.
(D) Expenses for the preservation
and improvement of property, when
the law authorizes reimbursement,
upon the preserved or improved
immovable.
(19) When bilateral contracts are vitiated
with vices of consent, they are rendered
(A) rescissible.
(B) void.
(C) unenforceable.
(D) voidable.
(20) An agent, authorized by a special
power of attorney to sell a land belonging to
the principal succeeded in selling the same
to a buyer according to the instructions

deed of absolute sale on behalf of his

claimed that it was exempt from execution,


principal two days after the principal died,
an event that neither the agent nor the
buyer knew at the time of the sale. What is
the standing of the sale?
(A) Voidable.
(B) Valid.
(C) Void.
(D) Unenforceable.

being a family home. Is this claim correct?


(A) Yes, because while Bs parents
own the land, they agreed to have
their

daughter

build

her

family

home on it.
(B) No, because there is no judicial
declaration that it is a family home.
(C) No, since the land does not
belong to A and B, it cannot

(21) Spouses A and B leased a piece of land

qualify as a family home.

belonging to B's parents for 25 years. The

(D) Yes, because the A and Bs

spouses built their house on it worth

family actually lives in that house.

P300,000.00. Subsequently, in a case that


C filed against A and B, the court found the

(22) Solomon sold his coconut plantation to

latter liable to C for P200,000.00. When the

Aragon, Inc. for P100 million, payable in

sheriff was attaching their house for the

installments of P10 million per month with

satisfaction of the judgment, A and B

6% interest per annum. Solomon married

Never Let The Odds Keep You From Pursuing What You Know In Your Heart You Were Meant To Do.Leroy Satchel Paige

Page 154 of 180

Civil Law Q&As (2007-2013)

hectorchristopher@yahoo.com

dbaratbateladot@gmail.com

Lorna after 5 months and they chose

(24) When A and B married, they chose

conjugal partnership of gains to govern

conjugal partnership of gains to govern

their

their property relations. After 3 years, B

property

relations.

When

they

married, Aragon had an unpaid balance of

succeeded in getting her marriage to A

P50 million plus interest in Solomons favor.

annulled

To whom will Aragons monthly payments

psychological incapacity. What liquidation

go after the marriage?

procedure will they follow in disposing of

(A) The principal shall go to the


conjugal

partnership

but

on

ground

of

the

latters

their assets?
(A)

the

They

will

interests to Solomon.

governing

the

(B) Both principal and interests

conjugal partnership of gains where

shall go to Solomon since they are

the party who acted in bad faith

his exclusive properties.

forfeits his share in the net profits.

(C) Both principal and interests

(B) Since the marriage has been

shall go to the conjugal partnership

declared

since these become due after the

liquidation of absolute community of

marriage.

property shall be followed.

(D) The

principal

shall

go

to

(C) The

follow

the

liquidation

void,

the

liquidation

rule
of

rule

of

for

co-

since

the

Solomon but the interests to the

ownership

conjugal partnership.

annulment brought their property

applies

relation under the chapter on


(23) X and Y, although not suffering from

property

any impediment, cohabited as husband and


wife

without

the

benefit

of

(D)

Following the birth of their child, the couple

annulled the marriage and issued a decree


of annulment. What is the present status of
the child?
(A) Legitimated.
(B) Illegitimate.
(C) Natural child.
(D) Legitimate.

without

marriage.

marriage.

got married. A year after, however, the court

regimes

The

law

on

liquidation

of

partnerships applies.
(25) X and Y agreed verbally before their
marriage

(a)

on

the

paternity

of

the

illegitimate child of Y and (b) on the


economic regime that will govern X and Ys
property relations. Is the verbal agreement
valid?

(A) No,

because

marriage

settlement to be valid should be


in writing.

Never Let The Odds Keep You From Pursuing What You Know In Your Heart You Were Meant To Do.Leroy Satchel Paige

Page 155 of 180

Civil Law Q&As (2007-2013)

hectorchristopher@yahoo.com

Fidels.
(B)

Yes,

since

ante-nuptial

agreements need not be in writing.


(C)

No,

settlement

because
cannot

marriage

include

an

agreement on the paternity of an


illegitimate child.
(D) Yes, since even if it is not a valid
marriage settlement, it is a valid
verbal contract.
(26) Spouses X and Y have a minor
daughter, Z, who needs support for her
education.

Both

and

Y,

who

are

financially distressed, could not give the


needed support to Z. As it happens, Zs
other relatives are financially capable of
giving that support. From whom may Z first
rightfully demand support? From her
(A) grandfather.
(B) brother.
(C) uncle.
(D) first cousin.
(27) Fidel, a Filipino with fair complexion,
married Gloria. Before the marriage, Gloria
confessed to Fidel that she was two-month
pregnant with the child of a black African
who had left the country for good. When
the child was born, Fidel could not accept it
being too black in complexion. What is the
status of the child?
(A)

Illegitimate,

because

confessed that the child is not

Gloria

dbaratbateladot@gmail.com

(B) Illegitimate, because by the color

(29) In his will, the testator designated X as

of its skin, the child could not

a legatee to receive P2 million for the

possibly be that of Fidel.

purpose of buying an ambulance that the

(C) Legitimate, because the child

residents of his Barangay can use. What

was born within a valid marriage.

kind of institution is this?

(D) Legitimate, because Fidel agreed


to treat the child as his own after
Gloria told him who the father was.
(28) The husbands acts of forcibly ejecting
his wife without just cause from the
conjugal dwelling and refusing to take her
back constitutes
(A) desertion.
(B) recrimination.
(C) constructive abandonment.

(A) a fideicomissary institution.


(B) a modal institution.
(C) a conditional institution.
(D) a collective institution.
(30) X insured himself for P5 million,
designating
beneficiary.

Y,

his
The

wife,

as

his

designation

sole
was

irrevocable. A few years later, X had their


marriage annulled in court on the ground
that Y had an existing prior marriage. X

(D) de facto separation.

Never Let The Odds Keep You From Pursuing What You Know In Your Heart You Were Meant To Do.Leroy Satchel Paige

Page 156 of 180

Civil Law Q&As (2007-2013)

hectorchristopher@yahoo.com

dbaratbateladot@gmail.com

(A) Marlon gets 1/4 and Cecilia gets


3/4.

subsequently died, Is Y entitled to the

(B) Marlon gets 2/3 and Cecilia 1/3.

insurance benefits?
(A) Yes, since the insurance was not
dependent on the marriage.
(B) Yes, since her designation as
beneficiary was irrevocable.
(C) No, Xs designation of Y is
revoked by operation of law upon
the annulment of their marriage
based on Ys fault.
(D)

Yes,

since

revocation,

Xs

without

judicial

designation

of

remains valid and binding.


(31) May a spouse freely donate communal
or conjugal property without the consent of
the other?
(A) Absolutely not, since the spouses
co-own such property.
(B) Yes, for properties that the
family

may

spare,

regardless

of

value.
(C) Yes, provided the donation is
moderate and intended for charity
or family rejoicing.
(D) Yes, in a donation mortis causa
that the donor may still revoke in
his lifetime.
(32) The decedent died intestate leaving an
estate of P10 million. He left the following
heirs: a) Marlon, a legitimate child and b)
Cecilia, the legal spouse. Divide the estate.

(34) A buyer ordered 5,000 apples from the


(C) Marlon gets 1/2 and Cecilia
gets 1/2.
(D) Marlon gets 3/4 and Cecilia 1/4.

seller at P20 per apple. The seller delivered


6,000 apples. What are the rights and
obligations of the buyer?
(A) He can accept all 6,000 apples

(33) Contracts take effect only between the

and pay the seller at P20 per

parties or their assigns and heirs, except

apple.

where the rights and obligations arising

(B) He can accept all 6,000 apples

from the contract are not transmissible by

and pay a lesser price for the 1,000

their nature, by stipulation, or by provision

excess apples.

of law. In the latter case, the assigns or the

(C) He can keep the 6,000 apples

heirs are not bound by the contracts. This

without paying for the 1,000 excess

is known as the principle of

since

the

seller

delivered

them

(A) Relativity of contracts.

anyway.

(B) Freedom to stipulate.

(D)

(C) Mutuality of contracts.

transaction since the seller violated

(D) Obligatory force of contracts.

the terms of their agreement.

He

can

cancel

the

whole

Never Let The Odds Keep You From Pursuing What You Know In Your Heart You Were Meant To Do.Leroy Satchel Paige

Page 157 of 180

Civil Law Q&As (2007-2013)

hectorchristopher@yahoo.com

dbaratbateladot@gmail.com

(37) Acme Cannery produced sardines in


(35) Lino entered into a contract to sell with
Ramon, undertaking to convey to the latter
one of the five lots he owns, without
specifying which lot it was, for the price of
P1 million. Later, the parties could not
agree which of five lots he owned Lino
undertook to sell to Ramon. What is the
standing of the contract?
(A) Unenforceable.
(B) Voidable.
(C) Rescissible.
(D) Void.
(36) Knowing that the car had a hidden
crack in the engine, X sold it to Y without
informing the latter about it. In any event,
the deed of sale expressly stipulated that X
was not liable for hidden defects. Does Y
have the

right

to demand from

X a

reimbursement of what he spent to repair


the engine plus damages?
(A) Yes. X is liable whether or not he
was aware of the hidden defect.
(B) Yes, since the defect was not
hidden; X knew of it but he acted
in bad faith in not disclosing the
fact to Y.
(C) No, because Y is in estoppel,
having changed engine without prior
demand.
(D)

No,

because

waived

warranty against hidden defects.

the

cans known as "Sards." Mylene bought a


can of Sards from a store, ate it, and

and Mylene has not overcome such


suffered

from

poisoning

caused

by

noxious substance found in the sardines.


Mylene filed a case for damages against
Acme. Which of the following defenses will
hold?
(A) The expiry date of the "Sards"
was clearly printed on its can,
still the store sold and Mylene

presumption.
(38) Fernando executed a will, prohibiting
his wife Marina from remarrying after his
death, at the pain of the legacy of P100
Million in her favor becoming a nullity. But
a year after Fernandos death, Marina was
so overwhelmed with love that she married
another man. Is she entitled to the legacy,

bought it.

the amount of which is well within the

(B) Mylene must have detected the

capacity of the disposable free portion of

noxious substance in the sardines

Fernandos estate?

by smell, yet she still ate it.

(A) Yes, since the prohibition against

(C) Acme had no transaction with

remarrying is absolute, it is deemed

Mylene; she bought the "Sards" from

not written.

a store, not directly from Acme.

(B) Yes, because the prohibition is

(D) Acme enjoys the presumption of

inhuman and oppressive and

safeness of its canning procedure


Never Let The Odds Keep You From Pursuing What You Know In Your Heart You Were Meant To Do.Leroy Satchel Paige

Page 158 of 180

Civil Law Q&As (2007-2013)

hectorchristopher@yahoo.com

dbaratbateladot@gmail.com

violates Marinas rights as a free

(A) It

woman.

since it was not given to the bride

(C) No, because the nullity of the

or groom.

prohibition also nullifies the legacy.


(D) No, since such prohibition is

is

an

ordinary

donation

(B) It is donation propter nuptias


since it was given with the marriage

authorized by law and is not

in mind.

repressive; she could remarry but

(C) It is an indirect donation propter

must give up the money.

nuptias

since

the

bride

would

eventually inherit the property from


(39) X, the owner, constituted a 10-year

her parents.

usufruct on his land as well as on the

(D) It is a remunatory donation.

building standing on it in Ys favor. After


flood totally destroyed the building 5 years

(41) X and Y, both Filipinos, were married

later, X told Y that an act of God terminated

and resided in Spain although they intend

the usufruct and that he should vacate the

to return to the Philippines at some future

land. Is X, the owner of the land, correct?

time. They have not executed any marriage

(A)

No,

since

the

building

was

settlements.

What

law

governs

their

property relations?

destroyed through no fault of Y.


(B) No, since Y still has the right

(A)

to use the land and the materials

Spanish law and Philippine law.

left on it.

(B) Philippine law since they are

(C) Yes, since Y cannot use the land

both Filipinos.

without the building.

(C) No regime of property relations

(D) Yes, since the destruction of the

will apply to them.

building

(D) Spanish law since they live in

without

the

Xs

fault

They

may

choose

between

Spain.

terminated the usufruct.


(40) In gratitude, the grooms parents made

(42) Birth determines personality. Death

a donation of a property in writing to the

extinguishes it. Under what circumstances

brides

may the personality of a deceased person

parents

shortly

childrens

wedding.

The

accepted.

What

the

donation?

is

before

their

donation

was

nature

of

the

continue to exist?
(A) In case of re-appearance of a
missing person presumed dead.

Never Let The Odds Keep You From Pursuing What You Know In Your Heart You Were Meant To Do.Leroy Satchel Paige

Page 159 of 180

Civil Law Q&As (2007-2013)

hectorchristopher@yahoo.com

dbaratbateladot@gmail.com

(A) the brother or sister who needs


(B) In protecting the works of a
deceased under intellectual property
laws.
(C)

In

case

presumptive

of
death

declaration

of

of a missing

spouse.
(D) In the settlement of the estate
of a deceased person.

(43) Six tenants sued X, the landowner, for


willfully denying them water for their farms,
which water happened to flow from land
under Xs control, his intention being to
force them to leave his properties. Is X
liable for his act and why?
(A) No, because the tenants must be
content with waiting for rainfall for
their farms.
(B) No, since X owns both the land
and the water.
(C) Yes, because the tenants farms
have the natural right of access to
water wherever it is located.
(D) Yes, since X willfully caused
injury to his tenants contrary to
morals, good customs or public
policy.

(44)

Illegitimate

brothers

and

sisters,

whether of full or half-blood, are bound to


support each other, EXCEPT when

support lives in another place.


(B) such brothers and sisters are not
recognized by their father.

(A) No, the child was 7 years old and


(C) the brother or sister in need
stops

schooling

without

valid

reason.
(D) the need for support of a
brother or sister, already of age, is
due to the latter's fault.

(45) Virgilio owned a bare and simple

knew the dangers that the pool


offered.
(B)

Yes,

being

an

attractive

nuisance, Virgilio had the duty to


prevent children from coming near
it.
(C) No, since the pool was bare
and had no enticing or alluring

swimming pool in his garden. MB, a 7-year

gadgets, floats, or devices in it

old child, surreptitiously entered the garden

that would attract a 7-year old

and merrily romped around the ledges of

child.

the pool. He accidentally tripped, fell into

(D) Yes, since Virgilio did not cover

the pool, and drowned. MBs parents sued

the swimming pool while not in use

Virgilio for damages arising from their

to prevent children from falling into

childs death, premised on the principle of

it.

"attractive nuisance". Is Virgilio liable for


the death of MB?
Never Let The Odds Keep You From Pursuing What You Know In Your Heart You Were Meant To Do.Leroy Satchel Paige

Page 160 of 180

Civil Law Q&As (2007-2013)

hectorchristopher@yahoo.com

dbaratbateladot@gmail.com

(B) No, but Rex is entitled to recover


(46) The term of a 5-year lease contract
between X the lessor and Y the lessee,
where rents were paid from month to
month, came to an end. Still, Y continued
using the property with Xs consent. In
such a case, it is understood that they
impliedly renewed the lease
(A) from month to month under
the same conditions as to the
rest.
(B) under the same terms and
conditions as before.
(C) under the same terms except the
rent which they or the court must
fix.
(D) for only a year, with the rent
raised by 10% pursuant to the
rental control law.
(47)

Rex,

valuable

lot to

philanthropist,
the

donated

municipality on

a
the

condition that it will build a public school on


such lot within 2 years from its acceptance of
the

donation.

The

municipality

properly

accepted the donation but did not yet build


the public school after 2 years. Can Rex
revoke the donation?

(A) Yes,

since

the

donation

is

subject to a resolutory condition


which was not fulfilled.

the value of the land from the


municipality.
(C) No, the transfer of ownership has
been completed.

Without his consent, his friend Boyong paid


(D) Yes, the donation is not deemed
made until the suspensive condition

the whole loan. Since Asiong benefited from


the payment, can Boyong compel the bank
to subrogate him in its right as mortgagee

has been fulfilled.

of Asiong's land?
(48)

Illegitimate

children,

those

not

(A) No, but the bank can foreclose

recognized by their biological fathers, shall

and pay Boyong back.

use the surname of their

(B) No, since Boyong paid for

(A) biological father subject to no

Asiongs

condition.

approval.

(B) mother or biological father, at

(C) Yes, since a change of creditor

the mothers discretion.

took place by novation with the

(C) mother.

banks consent.

(D)

biological

father

unless

he

judicially opposes it.

loan

without

his

(D) Yes, since it is but right that


Boyong be able to get back his
money and, if not, to foreclose the

(49) Asiong borrowed P1 million from a

mortgage in the manner of the

bank, secured by a mortgage on his land.

bank.

Never Let The Odds Keep You From Pursuing What You Know In Your Heart You Were Meant To Do.Leroy Satchel Paige

Page 161 of 180

Civil Law Q&As (2007-2013)

hectorchristopher@yahoo.com

dbaratbateladot@gmail.com

(C) Yes, since the payment covers


(50) Congress passed a law imposing taxes
on income earned out of a particular
activity that was not previously taxed. The
law, however, taxed incomes already earned
within the fiscal year when the law took
effect. Is the law valid?
(A) No, because laws are intended to
be prospective, not retroactive.
(B) No, the law is arbitrary in that it
taxes income that has already been
spent.
(C) Yes, since tax laws are the
lifeblood of the nation.
(D) Yes, tax laws are an exception;
they

can

be

given

retroactive

effect.

(51)

Rudolf

Rodrigo

borrowed

and

Fernando

P1

million

who

from

acted

as

solidary creditors. When the loan matured,


Rodrigo wrote a letter to Rudolf, demanding
payment of the loan directly to him. Before
Rudolf could comply, Fernando went to see
him personally to collect and he paid him.
Did Rudolf make a valid payment?
(A) No, since Rudolf should have
split the payment between Rodrigo
and Fernando.
(B) No, since Rodrigo, the other
solidary creditor, already made a
prior demand for payment from
Rudolf.

the whole obligation.

governed by the regime of absolute


(D) Yes,

since

Fernando

was

solidary creditor, payment to him

community of property.
(D) They are superseded by the
Family Code which has retroactive

extinguished the obligation.

effect.
(52) What happens to the property regimes

(53) The testator executed a will following

that were subsisting under the New Civil

the formalities required by the law on

Code when the Family Code took effect?

succession without designating any heir.

(A) The original property regimes


are

immutable

and

remain

effective.
(B) Those enjoying specific regimes
under the New Civil Code may adopt
the regime of absolute community of
property under the Family Code.
(C) Those that married under the
New Civil Code but did not choose

The only testamentary disposition in the


will is the recognition of the testator's
illegitimate child with a popular actress. Is
the will valid?
(A) Yes, since in recognizing his
illegitimate child, the testator has
made him his heir.
(B) No, because the non-designation
of heirs defeats the purpose of a will.

any of its regimes shall now be

Never Let The Odds Keep You From Pursuing What You Know In Your Heart You Were Meant To Do.Leroy Satchel Paige

Page 162 of 180

Civil Law Q&As (2007-2013)

hectorchristopher@yahoo.com

dbaratbateladot@gmail.com

(B) Void, because the couple did not


(C) No, the will comes to life only
when the proper heirs are instituted.
(D) Yes,

the

illegitimate

recognition
heir

is

of

an

an

ample

reason for a will.

(54) A left B, his wife, in the Philippines to


work in Egypt but died in that country after
a years continuous stay. Two months after
As death, B gave birth to a child, claiming
it is As child. Who can assail the legitimacy
of the child?
(A) As other heirs apart from B.
(B) The State which has interest in
the welfare of overseas contract
workers.
(C) Any one who is outraged by Bs
claim.
(D) No one since A died.
(55) QR and TS who had a marriage license
requested a newly appointed Judge in
Manila to marry them on the beach of
Boracay.

Since

the

Judge

maintained

Boracay as his residence, he agreed. The


sponsors were all public officials. What is
the status of the marriage.
(A) Valid,

since

the

improper

venue is merely an irregularity;


all

the

elements

marriage are present.

of

valid

get local permit for a beach wedding.

Y, came to the Philippines and married Z.


(C) Voidable, because the Judge
acted

beyond

his

territorial

Can X be held liable for bigamy?


(A) No since Xs marriage to Y is void

jurisdiction and is administratively

ab initio or did not exist.

liable for the same.

(B) No since X acted in good faith,

(D) Void, because the Judge did not

conscious that public policy did not

solemnize the marriage within the

approve of marriage between first

premises of his court.

cousins.
(C) Yes since he married Z without

(56) X and Y, Filipinos, got married in Los

first

Angeles, USA, using a marriage license

declaration

issued by the Philippine consul in Los

marriage to Y.

Angeles, acting as Civil Registrar. X and Y


did not know that they were first cousins
because their mothers, who were sisters,
were

separated

when

they

were

quite

securing
of

a
nullity

judicial
of

his

(D) Yes since his first marriage to Y


in Los Angeles is valid.
(57) Allan bought Billys property through

young. Since X did not want to continue

Carlos, an agent empowered with a special

with the relation when he heard of it, he left

power of attorney (SPA) to sell the same.

Never Let The Odds Keep You From Pursuing What You Know In Your Heart You Were Meant To Do.Leroy Satchel Paige

Page 163 of 180

Civil Law Q&As (2007-2013)

hectorchristopher@yahoo.com

dbaratbateladot@gmail.com

consumables,
When Allan was ready to pay as scheduled,
Billy called, directing Allan to pay directly to
him. On learning of this, Carlos, Billy's
agent, told Allan to pay through him as his
SPA

provided

and

to

protect

his

commission. Faced with two claimants,


Allan consigned the payment in court. Billy
protested, contending that the consignation
is ineffective since no tender of payment
was made to him. Is he correct?
(A) No, since consignation without
tender of payment is allowed in
the face of the conflicting claims
on the plaintiff.
(B) Yes, as owner of the property
sold, Billy can demand payment
directly to himself.
(C)

Yes,

since

Allan

made

no

announcement of the tender.


(D) Yes, a tender of payment is
required for a valid consignation.
(58) X sold Y 100 sacks of rice that Y was to
pick up from Xs rice mill on a particular
date. Y did not, however, appear on the
agreed date to take delivery of the rice.
After one week, X automatically rescinded
the sale without notarial notice to Y. Is the
rescission valid?
(A) Yes, automatic rescission is
allowed
character

since,
of

having

the

movables

and

deteriorate.

rice

can

easily

(A) No, efforts at a compromise will


(B) No, the buyer is entitled to a

only deepen the wifes anguish.

customary 30-day extension of his

(B) No, since legal separation like

obligation to take delivery of the

validity of marriage is not subject

goods.

to

(C) No, since there was no express

purposes of filing.

agreement

(C) Yes, to avoid a family feud that is

regarding

automatic

rescission.
(D)

No,

compromise

agreement

for

hurtful to everyone.
the

seller

should

first

(D) Yes, since the dispute could have

determine that Y was not justified in

been

failing to appear.

agreeing to legal separation.

settled

with

the

parties

(59) The wife filed a case of legal separation


against her husband on the ground of

(60) An Australian living in the Philippines

sexual infidelity

acquired shares of stock worth P10 million

without previously exerting earnest efforts

in food manufacturing companies. He died

to come to a compromise with him. The

in Manila, leaving a legal wife and a child in

judge dismissed the case for having been

Australia and a live-in partner with whom

filed without complying with a condition


precedent. Is the dismissal proper?
Never Let The Odds Keep You From Pursuing What You Know In Your Heart You Were Meant To Do.Leroy Satchel Paige

Page 164 of 180

Civil Law Q&As (2007-2013)

hectorchristopher@yahoo.com

dbaratbateladot@gmail.com

(B) No, since the sale cannot under


he had two children in Manila. He also left
a will, done according to Philippine laws,
leaving all his properties to his live-in
partner and their children. What law will
govern the validity of the disposition in the
will?
(A) Australia law since his legal wife
and legitimate child are Australians
and domiciled in Australia.
(B) Australian

law

since

the

intrinsic validity of the provisions


of a will is governed

by the

decedents national law.


(C) Philippine law since the decedent
died in Manila and he executed his
will according to such law.
(D)

Philippine

law

since

the

decedents properties are in the


Philippines.
(61) X bought a land from Y, paying him
cash. Since they were friends, they did not
execute any document of sale. After 7 years,
the heirs of X asked Y to execute a deed of
absolute sale to formalize the verbal sale to
their father. Unwilling to do so, Xs heirs filed
an action for specific performance against Y.
Will their action prosper?

(A) No, after more than 6 years,


the action to enforce the verbal
agreement has already elapsed.

the Statute of Frauds be enforced.


(C) Yes, since X bought the land and
paid Y for it.

proceedings

to

(D) Yes, after full payment, the

administrator

whom

action became imprescriptible.

appoint.

new
it

will

(C) Cicero automatically becomes


(62) A court declared Ricardo, an old

administrator

bachelor, an absentee and appointed Cicero

until judicially relieved.

administrator of his property. After a year, it

(D) Ciceros alienations of Ricardo's

was discovered that Ricardo had died

property will be set aside.

of

Ricardos

estate

abroad. What is the effect of the fact of his


death

on

the

administration

of

his

property?

(63) Baldo, a rejected suitor, intimidated


Judy into marrying him. While she wanted

(A) With

Ricardo

an

to question the validity of their marriage

absentee but a deceased person,

two years after the intimidation ceased,

Cicero will cease to be administrator

Judy decided in the meantime to freely

of his properties.

cohabit with Baldo. After more than 5 years

(B) The administration shall be

following their wedding, Judy wants to file a

given

case for annulment of marriage against

by

jurisdiction

no

longer

the

court

over

the

having
intestate

Never Let The Odds Keep You From Pursuing What You Know In Your Heart You Were Meant To Do.Leroy Satchel Paige

Page 165 of 180

Civil Law Q&As (2007-2013)

hectorchristopher@yahoo.com

dbaratbateladot@gmail.com

(65) In the order of intestate succession


Baldo on ground of lack of consent. Will her

last intestate heirs or heir who will inherit if

action prosper?
(A) Yes, the action for annulment is
imprescriptible.
(B) No, since the marriage was
merely voidable and Judy ratified
it by freely cohabiting with Baldo
after the force and intimidation
had ceased.
(C) No, since the action prescribed 5
years

from

the

date

of

the

celebration of the marriage.


(D) Yes, because the marriage was
celebrated without Judy's consent
freely given.
(64) Is the wife who leaves her husband
without just cause entitled to support?
(A)

No,

because

the

wife

must

always be submissive and respectful


to the husband.
(B) Yes. The marriage not having
been

dissolved,

the

husband

continues to have an obligation to


support his wife.
(C) No,

where the decedent is legitimate, who is the

because in

leaving

the

conjugal home without just cause,


she forfeits her right to support.
(D) Yes, since the right to receive
support
condition.

is

not

subject

to

any

(A) Neither solidary nor joint since


they cannot waive the defense of

all heirs in the higher level are disqualified

fortuitous event to which they are

or unable to inherit?

entitled.

(A) Nephews and nieces.


(B) Brothers and sisters.

(B)

Solidary

or

joint

upon

the

(C) State.

discretion of Sam.

(D) Other collateral relatives up to

(C) Solidary since Roy and Carlos

the 5th degree of consanguinity.

failed to perform their obligation to


deliver the motor boat.

(66) Roy and Carlos both undertook a

(D) Joint since the conversion of

contract to deliver to Sam in Manila a boat

their liability to one of indemnity

docked in Subic. Before they could deliver

for damages made it joint.

it, however, the boat sank in a storm. The


contract provides that fortuitous event shall

(67) Joanne married James, a person with

not exempt Roy and Carlos from their

no known relatives. Through James' hard

obligation. Owing to the loss of the motor

work, he and his wife Joane prospered.

boat, such obligation is deemed converted

When

James

died,

his

estate

alone

into one of indemnity for damages. Is the


liability of Roy and Carlos joint or solidary?
Never Let The Odds Keep You From Pursuing What You Know In Your Heart You Were Meant To Do.Leroy Satchel Paige

Page 166 of 180

Civil Law Q&As (2007-2013)

hectorchristopher@yahoo.com

(D) When
amounted to P100 million. If, in his will,
James designates Joanne as his only heir,
what will be the free portion of his estate.
(A) Joanne gets all; estate has no
free portion left.
(B) Joanne gets 1/2; the other
half is free portion.
(C) Joanne gets 1/3; the remaining
2/3 is free portion.
(D) Joanne gets 1/4; the remaining
3/4 is free portion.
(68) A warranty inherent in a contract of
sale, whether or not mentioned in it, is
known as the
(A) warranty on quality.
(B) warranty against hidden defects.
(C) warranty against eviction.
(D) warranty in merchantability.
(69) The doctrine of stare decisis prescribes
adherence

to

precedents

in

order

to

promote the stability of the law. But the


doctrine can be abandoned
(A) When adherence to it would
result in the Governments loss of its
case.
(B) When the application of the
doctrine

would

cause

great

prejudice to a foreign national.


(C) When necessary to promote the
passage of a new law.

dbaratbateladot@gmail.com

the

precedent

has

ceased to be beneficial and useful.

(C) Yes, since they executed their


(70) Ric and Josie, Filipinos, have been
sweethearts for 5 years. While working in a
European country where the execution of
joint wills are allowed, the two of them
executed a joint holographic will where they
named each other as sole heir of the other
in case either of them dies. Unfortunately,
Ric died a year later. Can Josie have the
joint will successfully probated in the
Philippines?
(A) Yes, in the highest interest of
comity of nations and to honor the
wishes of the deceased.
(B) No,

since

Philippine

law

prohibits the execution of joint


wills and such law is binding on

joint will out of mutual love and


care,

values

that

the

generally

accepted principles of international


law accepts.
(D) Yes, since it is valid in the
country

where

it

was

executed,

applying the principle of "lex loci


celebrationis."
(71) ML inherited from his father P5 million
in legitime but he waived it in a public
instrument in favor of his sister QY who
accepted the waiver in writing. But as it
happened, ML borrowed P6 million from PF
before the waiver. PF objected to the waiver
and filed an action for its rescission on the
ground that he had the right to MLs P5

Ric and Josie even abroad.


Never Let The Odds Keep You From Pursuing What You Know In Your Heart You Were Meant To Do.Leroy Satchel Paige

Page 167 of 180

Civil Law Q&As (2007-2013)

hectorchristopher@yahoo.com

dbaratbateladot@gmail.com

is essentially void for containing an


million legitime as partial settlement of
what ML owed him since ML has proved to
be insolvent. Does PF, as creditor, have the
right to rescind the waiver?
(A) No, because the waiver in favor
of his sister QY amounts to a
donation and she already accepted
it.
(B) Yes,

because

the

waiver

is

prejudicial to the interest of a


third person whose interest is
recognized by law.
(C) No, PF must wait for ML to
become solvent and, thereafter, sue
him for the unpaid loan.
(D) Yes, because a legitime cannot
be waived in favor of a specific heir;
it must be divided among all the
other heirs.
(72) While engaged to be married, Arnold
and

Josephine

agreed

in

public

instrument to adopt out the economic


regime of absolute community of property.
Arnold

acknowledged

in

the

same

instrument that Josephines daughter


Mary,

is

his

illegitimate

child.

But

Josephine died before the marriage could


take place. Does the marriage settlement
have any significance?
(A)

None,

since

the

instrument

containing the marriage settlement

unrelated matter.

Philippines and married Leonora. What is


(B) Yes,

insofar

acknowledged

as

Mary

Arnold
as

his

illegitimate child.
(C) None, since the marriage did not
take place.
(D) Yes, if they acquired properties

the status of this second marriage?


(A) Void, because he did not cause
the

judicial

issuance

of

declaration of the nullity of his


first marriage to Jenny before
marrying Leonora.

while living together as husband

(B) Valid, because Joseph's marriage

and wife.

to Jenny is void, he being only 17

(73) Joseph, a 17-year old Filipino, married

years of age when he married her.

Jenny, a 21-year old American in Illinois,

(C) Valid, because his marriage to

USA, where the marriage was valid. Their

Leonora has all the elements of a

parents gave full consent to the marriage of

valid marriage.

their children. After three years, Joseph

(D) Void, because Joseph is still

filed a petition in the USA to promptly

considered married to Jenny since

divorce Jenny and this was granted. When

the Philippines does not recognize

Joseph turned 25 years, he returned to the

divorce.

Never Let The Odds Keep You From Pursuing What You Know In Your Heart You Were Meant To Do.Leroy Satchel Paige

Page 168 of 180

Civil Law Q&As (2007-2013)

hectorchristopher@yahoo.com

dbaratbateladot@gmail.com

(C) unless the court first directs


(74) T died intestate, leaving an estate of

mediation of the parties.

P9,000,000. He left as heirs three legitimate

(D)

children, namely, A, B, and C. A has two

conducted by a public prosecutor.

children, D and E. Before he died, A


irrevocably repudiated his inheritance from
T in a public instrument filed with the
court. How much, if any, will D and E, as
As children, get from Ts estate?
(A) Each of D and E will get
P1,500,000

by

representation

since

right
their

of
father

repudiated his inheritance.


(B) Each of D and E will get
P2,225,000 because they will inherit
from the estate equally with B and
C.
(C) D and E will get none because
of the repudiation; "B" and "C"
will get As share by right of
accretion.
(D) Each of D and E will get
P2,000,000 because the law gives
them some advantage due to the
demise of "A".
(75) No decree of legal separation can be
issued
(A) unless the childrens welfare is
attended to first.
(B) without

prior

efforts

at

reconciliation shown to be futile.

without

prior

investigation

with the title needed to effect the


(76) X, who

was

abroad,

phoned his

brother, Y, authorizing him to sell Xs parcel


of land in Pasay. X sent the title to Y by
courier service. Acting for his brother, Y
executed a notarized deed of absolute sale
of the land to Z after receiving payment.
What is the status of the sale?
(A) Valid, since a notarized deed of
absolute

sale

covered

the

transaction and full payment was


made.
(B) Void,

since

should

have

authorized agent Y in writing to


sell the land.

sale.
(D) Valid, since the buyer could file
an action to compel X to execute a
deed of sale.
(77) In a true pacto de retro sale, the title
and ownership of the property sold are
immediately vested in the vendee a retro
subject only to the resolutory condition of
repurchase by the vendor a retro within the
stipulated period. This is known as
(A) equitable mortgage.
(B) conventional redemption.
(C) legal redemption.
(D) equity of redemption.

(C) Valid, since Y was truly his

(78) A natural obligation under the New

brother Xs agent and entrusted

Civil Code of the Philippines is one which

Never Let The Odds Keep You From Pursuing What You Know In Your Heart You Were Meant To Do.Leroy Satchel Paige

Page 169 of 180

Civil Law Q&As (2007-2013)

hectorchristopher@yahoo.com

dbaratbateladot@gmail.com

only upon the wifes acceptance


(A)

the

obligor

has

moral

obligation to do, otherwise entitling


the obligee to damages.
(B) refers to an obligation in writing
to do or not to do.
(C) the obligee may enforce through
the court if violated by the obligor.
(D) cannot be judicially enforced
but

authorizes

the

obligee

to

retain the obligors payment or


performance.

(79)

The

husband

administration

of

the

assumed
familys

sole
mango

plantation since his wife worked abroad.


Subsequently, without his wifes knowledge,
the husband entered into an antichretic
transaction

with

company,

possession

and

management

giving
of

it
the

plantation with power to harvest and sell


the fruits and to apply the proceeds to the
payment of a loan he got. What is the
standing of the contract?
(A) It is void in the absence of the
wifes consent.
(B) It is void absent an authorization
from the court.
(C) The transaction is void and can
neither be ratified by the wife nor
authorized by the court.
(D) It is considered a continuing
offer by the parties, perfected

or the courts authorization.

(81) Anne owed Bessy P1 million due on


(80)

When

the

donor

gives

donations

without reserving sufficient funds for his


support

or

for

the

support

of

his

(A) Rescissible, since it results in


economic lesion of more than 25%
of the value of his properties.
(B) Voidable, since his consent to
the donation is vitiated by mindless
kindness.
(C) Void, since it amounts to wanton
expenditure beyond his means.
(D) Reducible to the extent that
donations

impaired

the

support due to himself and his


dependents.

due date. Bessy sent a demand letter to


Anne giving her 5 days from receipt within
which to pay. Two days after receipt of the

dependents, his donations are

the

October 1, 2011 but failed to pay her on

letter, Anne personally offered to pay Bessy


in manager's check but the latter refused to
accept the same. The 5 days lapsed. May
Annes

obligation

be

considered

extinguished?
(A) Yes, since Bessys refusal of the
managers check, which is presumed
funded, amounts to a satisfaction of
the obligation.
(B) No, since tender of payment
even in cash, if refused, will not
discharge the obligation without
proper consignation in court.

Never Let The Odds Keep You From Pursuing What You Know In Your Heart You Were Meant To Do.Leroy Satchel Paige

Page 170 of 180

Civil Law Q&As (2007-2013)

(C)

Yes,

since

hectorchristopher@yahoo.com

Anne

tendered

dbaratbateladot@gmail.com

(83) The owner of a thing cannot use it in a

payment of the full amount due.

way that will injure the right of a third

(D) No, since a managers check is

person. Thus, every building or land is

not considered legal tender in the

subject to the easement which prohibits its

Philippines.

proprietor or possessor from committing


nuisance like noise, jarring, offensive odor,

(82) The residents of a subdivision have

and smoke. This principle is known as

been using an open strip of land as passage

(A) Jus vindicandi.

to the highway for over 30 years. The owner

(B) Sic utere tuo ut alienum non

of that land decided, however, to close it in

laedas.

preparation for building his house on it.

(C) Jus dispondendi.

The residents protested, claiming that they

(D) Jus abutendi.

became

owners

of

the

land

through

acquisitive prescription, having been in

(84) Janice and Jennifer are sisters. Janice

possession of the same in the concept of

sued

owners,

and

business partner for recovery of property

continuously for more than 30 years. Is this

with damages. The complaint did not allege

claim correct?

that Janice exerted earnest efforts to come

publicly,

peacefully,

Jennifer

and

Laura,

Jennifers

(A) No, the residents have not

to a compromise with the defendants and

been in continuous possession of

that

the land since they merely passed

dismissed the complaint outright for failure

through

to comply with a condition precedent. Is the

it

in

going

to

the

highway.
(B) No, the owner did not abandon
his right to the property; he merely

such

efforts

failed.

The

judge

dismissal in order?
(A) No, since Laura is a stranger to
the sisters, Janice has no moral

tolerated his neighbors use of it for

obligation to settle with her.

passage.

(B) Yes, since court should promote

(C) Yes, residents of the subdivision

amicable

have become owners by acquisitive

relatives.

prescription.

(C) Yes, since members of the same

(D) Yes, community ownership by

family, as parties to the suit, are

prescription prevails over private

required to exert earnest efforts to

claims.

settlement

among

settle their disputes before coming


to court.

Never Let The Odds Keep You From Pursuing What You Know In Your Heart You Were Meant To Do.Leroy Satchel Paige

Page 171 of 180

Civil Law Q&As (2007-2013)

hectorchristopher@yahoo.com

dbaratbateladot@gmail.com

(D) No, the family council, which

(87) When does the regime of conjugal

would

partnership of gains begin to exist?

ordinarily

mediate

the

dispute, has been eliminated under

(A) At the moment the parties

the Family Code.

take and declare each other as


husband

(85) X borrowed money from a bank,


secured by a mortgage on the land of Y, his
close friend. When the loan matured, Y
offered to pay the bank but it refused since
Y was not the borrower. Is the banks action
correct?
(A) Yes, since X, the true borrower,
did not give his consent to Ys offer
to pay.
(B) No, since anybody can discharge
Xs obligation to his benefit.
(C) No, since Y, the owner of the
collateral, has an interest in the
payment of the obligation.

obligation to the bank.

property

after

performance
mortgage

of

but

redeem

the

mortgaged

his

default

the

conditions

before

the

sale

in

the

of

the

of

the

mortgaged property or confirmation of the


sale by the court, is known as
(A) accion publiciana.
(B) equity of redemption.
(C) pacto de retro.
(D) right of redemption.

before

officiating officer.
(B) At the time the spouses acquire
properties through joint efforts.
(C) On the date the future spouses
executed their marriage settlements
because this is the starting point of
their marital relationship.
(D) On the date agreed upon by the
future spouses in their marriage
settlements since their agreement is
the law between them.
(88) Josie, 18, married Dante, 25, without

lived with him. After a year, Josie returned


to her parents home, complained of the

(86) The right of a mortgagor in a judicial


to

wife

her parents knowledge and consent, and

(D) Yes, since it was X who has an

foreclosure

and

unbearable battering she was getting from


Dante, and expressed a desire to have her
marriage with him annulled. Who may
bring the action?
(A) Dante.
(B) Her parents.
(C) Josie herself.
(D) The State.
(89) X, a married man, cohabited with Y, an
unmarried woman. Their relation bore them

BB, a baby boy. Subsequently, after X


became a widower, he married Y. Was BB
legitimated by that marriage?

Never Let The Odds Keep You From Pursuing What You Know In Your Heart You Were Meant To Do.Leroy Satchel Paige

Page 172 of 180

Civil Law Q&As (2007-2013)

hectorchristopher@yahoo.com

dbaratbateladot@gmail.com

(A) Yes, since his parents are now

(D) Yes, as long as they leave

lawfully married.

sufficient property for themselves

(B) Yes, since he is an innocent

and for their dependents.

party and the marriage rectified the


wrong done him.

(92) X owed Y P1.5 million. In his will, X

(C) No, since once illegitimate, a

gave Y legacy of P1 million but the will

child

provided that this legacy is to be set off

shall

always

remain

illegitimate.

against the P1.5 million X owed Y. After the

(D) No, since his parents were not

set off, X still owed Y P500,000. Can Y still

qualified

collect this amount?

to

marry

each

other

(A) Yes, because the designation

when he was conceived.

of Y as legatee created a new and


(90) The presence of a vice of consent

separate

vitiates the consent of a party in a contract

between them, that of testator-

and this renders the contract

legatee.

relationship

(A) Rescissible.

(B) It depends upon the discretion of

(B) Unenforceable.

the probate court if a claim is filed

(C) Voidable.

in the testate proceedings.

(D) Void.
(91)

juridical

Can

properties

(C) No, because the intention of the

common-law

spouses

donate

of substantial value to one

another?
(A) No, they are only allowed to
give moderate gifts to each other

testator in giving the legacy is to


abrogate his entire obligation to Y.
(D)

No,

because

had

no

instruction in his will to deliver


more than the legacy of P1 million to
Y.

during family rejoicing.


(B) No, they cannot give anything of

(93) Josie owned a lot worth P5 million

value

prevent

prior to her marriage to Rey. Subsequently,

placing their legitimate relatives at a

their conjugal partnership spent P3 million

disadvantage.

for the construction of a house on the lot.

(C) Yes, unlike the case of legally

The construction resulted in an increase in

married spouses, such donations

the value of the house and lot to P9 million.

are not prohibited.

Who owns the house and the lot?

to

each

other

to

Never Let The Odds Keep You From Pursuing What You Know In Your Heart You Were Meant To Do.Leroy Satchel Paige

Page 173 of 180

Civil Law Q&As (2007-2013)

hectorchristopher@yahoo.com

dbaratbateladot@gmail.com

(B) When the title holder concealed


(A)

Josie

and

the

conjugal

partnership of gains will own both


on a 50-50 basis.
(B) Josie will own both since the
value

of

the

house

and

the

increase in the propertys value is


less than her lots value; but she
is

to

reimburse

conjugal

partnership expenses.
(C) Josie still owns the lot, it being
her

exclusive

house

property,

belongs

to

the

but

the

conjugal

partnership.
(D) The house and lot shall both
belong to the conjugal partnership,
with Josie entitled to reimbursement
for the value of the lot.

(94)

An

action

for

reconveyance

of

registered piece of land may be brought


against the owner appearing on the title
based on a claim that the latter merely
holds such title in trust for the plaintiff.
The action prescribes, however, within 10
years from the registration of the deed or
the date of the issuance of the certificate of
title of the property as long as the trust had
not been repudiated. What is the exception
to this 10-year prescriptive period?
(A) When the plaintiff had no notice
of the deed or the issuance of the
certificate of title.

the matter from the plaintiff.

marriage after they became 21 of


(C) When fortuitous circumstances

age;

prevented the plaintiff from filing

convalidated the marriage.

the case sooner.

(B) No, since Lindas parents made

(D) When

the

plaintiff

is

in

their

cohabitation

also

no allegations that earnest efforts


have been made to come to a

possession of the property.

compromise with Conrad and Linda


(95) Conrad and Linda, both 20 years old,

and which efforts failed.

applied for a marriage license, making it

(C)

appear that they

voidable, the couple being below 21

were over 25. They

Yes,

since

the

marriage

is

married without their parents knowledge

years of age when they married.

before an unsuspecting judge. After the

(D) Yes, since Lindas parents never

couple has been in cohabitation for 6 years,

gave their consent to the marriage.

Lindas parents filed an action to annul the


marriage on ground of lack of parental

(96) Pepito executed a will that he and 3

consent. Will the case prosper?

attesting witnesses signed following the

(A) No, since only the couple can


question

the

validity

of

their

formalities of law, except that the Notary


Public failed to come. Two days later, the

Never Let The Odds Keep You From Pursuing What You Know In Your Heart You Were Meant To Do.Leroy Satchel Paige

Page 174 of 180

Civil Law Q&As (2007-2013)

hectorchristopher@yahoo.com

dbaratbateladot@gmail.com

(A) Conjugal since they earned the


Notary Public notarized the will in his law
office where all signatories to the will
acknowledged that the testator signed the
will in the presence of the witnesses and
that the latter themselves signed the will in
the presence of the testator and of one
another. Was the will validly notarized?
(A) No, since it was not notarized on
the occasion when the signatories
affixed their signatures on the will.
(B) Yes, since the Notary Public
has to be present only when the
signatories acknowledged the acts
required of them in relation to
the will.
(C) Yes, but the defect in the mere
notarization of the will is not fatal to
its execution.
(D) No, since the notary public did
not require the signatories to sign
their respective attestations again.
(97) Venecio and Ester lived as common-law
spouses since both have been married to
other persons from whom they had been
separated

in

Hardworking

fact
and

for

several

years.

bright,

each

earned

incomes from their respective professions


and enterprises. What is the nature of their
incomes?

same while living as husband and


wife.

(C) 4 years from the perfection of the


donation.

(B) Separate since their property

(D) Such action does not prescribe.

relations with their legal spouses


are still subsisting.
(C) Co-ownership since they agreed
to work for their mutual benefit.
(D) Communal since they earned the
same as common-law spouses.
(98) What is the prescriptive period for filing
an action for revocation of a donation based
on acts of ingratitude of the donee?
(A) 5 years from the perfection of the
donation.
(B) 1 year from the perfection of
the donation.

(99)

Before

Karen

married

Karl,

she

inherited P5 million from her deceased


mother which amount she brought into the
marriage. She later used part of the money
to buy a new Mercedes Benz in her name,
which Karen and her husband used as a
family car. Is the car a conjugal or Karens
exclusive property?
(A) It is conjugal property since the
spouses use it as a family car.
(B) It is Karens exclusive property
since it is in her name.
(C) It is conjugal property having
been bought during the marriage.

Never Let The Odds Keep You From Pursuing What You Know In Your Heart You Were Meant To Do.Leroy Satchel Paige

Page 175 of 180

Civil Law Q&As (2007-2013)

hectorchristopher@yahoo.com

dbaratbateladot@gmail.com

stairway eventually gave way and collapsed,


(D) It

is

Karens

exclusive

property since she bought it with


her own money.

(100) Because of Xs gross negligence, Y


suffered

injuries

that

resulted

in

the

abortion of the foetus she carried. Y sued X


for, among other damages, P1 million for
the death of a family member. Is Y entitled
to indemnity for the death of the foetus she
carried?
(A) Yes, since the foetus is already
regarded as a child from conception,
though unborn.
(B) No, since Xs would not have
known

that

the

accident

would

result in Ys abortion.
(C) No,

since

birth

determines

personality, the accident did not


result in the death of a person.
(D) Yes, since the mother believed in
her heart that she lost a child.

2010 Civil Law Exam MCQ


(September 12, 2010)
No.II. Multiple choice.
(A). A had a 4-storey building which was
constructed by Engineer B. After five years,
the

building developed cracks

and its

resulting to injuries to some lessees. Who

1659 NCC, the proprietor of a building or


should the lessees sue for damages? (1%)

structure is responsible for the damages

(1). A, the owner

resulting

from

its

total

or

partial

collapse, if it is due to lack of necessary

(2). B, the engineer

repairs.

(3). both A & B

Under Article 1723, NCC, the engineer


or architect who drew up the plans and

SUGGESTED ANSWER:

specifications for a building is liable for

3. Both A & B.

damage if 15 years from the completion

The lessee may proceed against A for

of

breach of contract, and against B for tort

collapse by a reason of a defect by those

or statutory liability. Under Article 1654

plans and specifications, or due to the

(2) of the New Civil Code, the lessor is

defects in the ground. This liability

obliged to make all the necessary repairs

maybe enforced against the architect or

in order to keep the leased property

engineer even by a third party who has

suitable for the use to which it has been

no privity of contract with the architect

devoted.

or engineer under Article 2192, NCC.

Consequently,

under

Article

the

structure

the

same

should

Never Let The Odds Keep You From Pursuing What You Know In Your Heart You Were Meant To Do.Leroy Satchel Paige

Page 176 of 180

Civil Law Q&As (2007-2013)

hectorchristopher@yahoo.com

dbaratbateladot@gmail.com

problem are clearly those resulting from


defects in the construction plans or

ALTERNATIVE ANSWER:

specifications.

No.1. A , the owner .


The lessee can sue only the lessor for
breach of contract under Article 1659 in
relation to Article 1654, NCC. The lessee
cannot sue the architect or the engineer
because there was no privity of contracts
between them. When sued, however, the
lessor may file a third party claim
against the architect or the engineer.
ANOTHER ALTERNATIVE ANSWER:
No. 2. B, the Engineer .
Under

Article

1723

the

engineer

or

architect who drew up the plans and


specifications for a building is liable for
damages if within 15 years from the
completion of the structure, the same
should collapse by reason of a defect in
those plans and specifications, or due to
the defects in the ground. Under Article
2192 (NCC), however, if the damages
should be the result of any of the defects
in the construction mentioned in Art
1723, NCC, the third person suffering
damages may proceed only against the
engineer

or

architect

or

contractor

within the period fixed therein. The


damages suffered by the lessee in the

No. 4. None of the above.


(B) O, owner of Lot A, learning that
Japanese soldiers may have buried gold
and other treasures at the adjoining vacant
Lot B belonging to spouses X & Y, excavated
in Lot B where she succeeded in unearthing

The general rule is that the treasure


shall belong to the spouses X and Y, the
owner of Lot B. Under Article 438 (NCC),
the exception is that when the discovery

gold and precious stones. How will the

of a hidden treasure is made on the

treasures found by O be divided? (1%)

property of another and by chance, onehalf thereof shall belong to the owner of

(1). 100% to O as finder

the

land

and

the

other

one-half

is

(2). 50% to O and 50% to the spouses X

allowed to the finder. In the problem, the

and Y

finding

of the

treasure was

not

by

chance because O knew that the treasure


(3). 50% to O and 50% to the state
(4). None of the above.

was in Lot B. While a trespasser is also


not entitled to any share, and there is no
indication in the problem whether or not

SUGGESTED ANSWER:

O was a trespasser, O is not entitled to a

Never Let The Odds Keep You From Pursuing What You Know In Your Heart You Were Meant To Do.Leroy Satchel Paige

Page 177 of 180

Civil Law Q&As (2007-2013)

hectorchristopher@yahoo.com

dbaratbateladot@gmail.com

No. 2 and No.4 are both false. The


share because the finding was not by
chance.

(C) A executed a Deed of Donation in favor


of B, a bachelor, covering a parcel of land
valued at P1 million. B was, however, out of
the country at the time. For the donation to
be valid, (1%)
(1). B may e-mail A accepting the donation.
(2). The donation may be accepted by Bs
father with whom he lives.
(3). B can accept the donation anytime
convenient to him.
(4). Bs mother who has a general power of
attorney may accept the donation for him.
(5). None of the above is sufficient to make
Bs acceptance valid
SUGGESTED ANSWER:
No. 5 None of the above is sufficient to
make B's acceptance valid .
Since

the

donation

covered

an

immovable property, the donation and


the

acceptance

must

be

in

public

document and e-mail is not a public


document. Hence, No.1 is false.

acceptance by the donees father alone

(D) A executed a 5-page notarial will before a


or

mother

alone,

even

in

public

document, is not sufficient because the

notary public and three witnesses. All of them


signed each and every page of the will.

father and mother did not have a special

One of the witnesses was B, the father of

power of attorney for the purpose. Under

one of the legatees to the will. What is the

Article 745 (NCC), the donee must accept

effect of B being a witness to the will? (1%)

the donation personally, or through an


authorized person with a special power of
attorney for the purpose; otherwise, the
donation shall be void.

(1). The will is invalidated


(2). The will is valid and effective
(3). The legacy given to Bs child is not valid

No.3 is also false. B cannot accept the


donation anytime at his convenience.

SUGGESTED ANSWER:

Under Article 749 NCC, the donee may

No. 3. The legacy given to B's child is not

accept the donation only during the

valid.

lifetime of the donor.

The validity of the will is not affected by


the legacy in favor of the son of an

Never Let The Odds Keep You From Pursuing What You Know In Your Heart You Were Meant To Do.Leroy Satchel Paige

Page 178 of 180

Civil Law Q&As (2007-2013)

hectorchristopher@yahoo.com

(E). letters a and b


attesting witness to the will. However,
the said legacy is void under Article 823
NCC.
ALTERNATIVE ANSWER:
No. 2 .The will is valid and effective.
Under Article 823 ( NCC ),the legacy
given

in

favor

of

the

son

of

an

instrumental witness to a will has no


effect on the validity of the will. Hence,
the will is valid and effective.

2007 Civil Law Exam MCQ


(September 09, 2007)
No.IX. Multiple choice: Choose the right
answer. (2% each)
(1). The parties to a bailment are the:
(A). bailor;
(B). bailee;
(C) comodatario;
(D). all the above;
(E). letters a and b
SUGGESTED ANSWER:

dbaratbateladot@gmail.com

ALTERNATIVE ANSWER:
(D). all the above
(2). A deposit made in compliance with a
legal obligation is:
(A). an extrajudicial deposit;
(B). a voluntary deposit;

(C). a necessary deposit


(3). A contract of antichresis is always:
(A). a written contract;
(B). a contract, with a stipulation that the
debt will be paid through receipt of the
fruits of an immovable;
(C). Involves the payment of interests, if

(C). a necessary deposit;

owing;

(D). a deposit with a warehouseman;

(D). All of the above;

(E). letters a and b

(E). Letters a and b

SUGGESTED ANSWER:

SUGGESTED ANSWER:

Never Let The Odds Keep You From Pursuing What You Know In Your Heart You Were Meant To Do.Leroy Satchel Paige

Page 179 of 180

Civil Law Q&As (2007-2013)

hectorchristopher@yahoo.com

dbaratbateladot@gmail.com

(C). 3/5 of the number of creditors


should agree to the settlement;

(D). All of the above;


(4). An, assignee in a proceeding under the
Insolvency Law does not have the duty of:
(A). suing to recover the properties of the
state of the insolvent debtor;
(B). selling property of the insolvent debtor;
(C). ensuring that a debtor corporation
operate

the

business

efficiently

and

effectively while the proceedings are


pending;
(D). collecting and discharging debts owed
to the insolvent debtor.
SUGGESTED ANSWER:
(C). ensuring that a debtor corporation
operate

the

business

efficiently

and

effectively while the proceedings are


pending;
(5). In order to obtain approval of the
proposed settlement of the debtor in an
insolvency proceeding.
(A). the court must initiate the proposal
(B). 2/3 of the number of creditors should
agree to the settlement;

full credit for the two items regardless of


(D).

1/3

of

the

total

debts

must

be

their answers.]

represented by the approving creditors;


(E). Letters a and b
SUGGESTED ANSWER:

References:

(C). 3/5 of the number of creditors

not included within the coverage of Civil


Law but Commercial Law. It is therefore
suggested that the examinees be given

to

Bar

Examination

Questions by the UP LAW COMPLEX


(2007, 2009, 2010)

should agree to the settlement;


[Note: Items 4&5 on Insolvency Law are

Answers

UP LAW REVIEW

PHILIPPINE ASSOCIATION OF LAW


SCHOOLS (2008)
lawphil.net

Never Let The Odds Keep You From Pursuing What You Know In Your Heart You Were Meant To Do.Leroy Satchel Paige

Page 180 of 180

Anda mungkin juga menyukai